* Your assessment is very important for improving the workof artificial intelligence, which forms the content of this project
Download 4176 metodological instructions - Сумський державний університет
Hypothyroidism wikipedia , lookup
Metabolic syndrome wikipedia , lookup
Hypoglycemia wikipedia , lookup
Hyperthyroidism wikipedia , lookup
Graves' disease wikipedia , lookup
Epidemiology of metabolic syndrome wikipedia , lookup
Diabetes mellitus type 1 wikipedia , lookup
Epigenetics of diabetes Type 2 wikipedia , lookup
Diabetes mellitus wikipedia , lookup
Diabetes management wikipedia , lookup
Diabetes mellitus type 2 wikipedia , lookup
Diabetic ketoacidosis wikipedia , lookup
Ministry of Education and Science of Ukraine Sumy State University 4176 METODOLOGICAL INSTRUCTIONS for the practical course “ENDOCRINOLOGY” for the foreign students of the specialty 222 “Medicine” of the full-time course of study Sumy Sumy State University 2016 Metodological instructions for the practical course “Endocriniliogy” for the foreign students of the specialty 222 “Medicine” of the full-time course of study / compiler N. V. Demikhova. – Sumy : Sumy State University, 2016. – 208 p. Department of the Family and Social Medicine 2 Contents PREFACE ……..……………...……....................... 4 Unit 1. Diabetes mellitus………………………..…. 5 References for unit 1 ………………………………. 24 Unit 2. Thyreoid gland diseases………………… References for unit 2 …………………………… 24 38 Unit 3. Adrenal gland diseases…………………… 40 References for unit 3 …………………………….. 50 Unit 4. Hypothalamic and pituitary diseases…….. 50 References for unit 4 …………………………… 54 Unit 5. Diseases of the reproductive system…… References for unit 5 …………………………… 55 63 Methodological Instruction to Module………… 65 Tests and Assignments for Self-assessment…….. Tests for unit 1 ................................................... Tests for unit 2 ................................................... Tests for unit 3 ................................................... Tests for unit 4 ................................................... 3 73 73 140 169 189 PREFACE The methodological instructions for the practical course “Endocrinology” have been worked out for the fourth-year foreign students of the specialty 222 “Medicine” of the fulltime study course, aiming to facilitate quality and efficiency of obligatory and individual work and self-study foreseen by the curriculum of the Family and Social Medicine Department. The methodological instructions comprise set of questions to lecture and various tasks on different topics. The methodological instructions consist of five units. 4 Unit 1. Diabetes Mellitus Methodological instruction to practicl class №1 Theme: Diabetes mellitus type 1, 2: differentiation diagnostic, clinical assessment, investigations. Hours: 2. Working place: classroom, endocrine department wards of the Sumy Region Hospital. Aim: - to learn classification and pathogenic particularities of some forms of DM; - to learn pathomorphologic changes in organs and systems in patients with DM; - to learn signs, symptoms and clinical diagnostic criteria of DM; - to learn laboratory and instrumental diagnostic criteria of DM. Professional Motivation. Type 1 diabetes is the form of the disease due primarily to β-cell destruction. Thi s usually leads to a type of diabetes in which insulin is required for survival. Individuals with type 1 diabetes are metabolically normal before the disease is clinicall y manifest, but the process of β -cell destruction can be detected earlier by the prese nce of certain autoantibodies. Type 1 diabetes usually is characterized by the presence of anti -GAD, anti-islet cell, or anti-insulin antibodies, which reflects the autoimmune processes that have led to β -cell destruction. Individuals who have one of more of 5 these antibodies can be subclassified as having type 1A, immune-mediated type 1 diabetes. Type 2 diabetes is the most common form of diabetes. It is characterized by disorders of insulin action and insulin secretion, either of which may be the predominant feature. Usually, both are present at the time diabetes becomes clinically manifest. Although the specific etiology of this form of diabetes is not known, autoimmune destruction of the β-cells does not occur. Patients with type 2 diabetes usually have i nsulin resistance and relative, rather than absolute, insulin deficiency. At the time of diagnosis of diabetes, and often throughout their lifetimes, these patients do not need insulin treatment to survive, although ultimately many require it for gl ycemic control. This form of diabetes is associated with progressive β -cell failure with increasing duration of diabetes The currently recommended classification and criteria for diabetes differ from those recommended in earlier years. The major changes in classi fication have been to move toward an etiologically based classification because of increasing knowledge about some of the specific underlying causes of diabetes. This trend is likely to continue. The terms insulin-dependent and non-insulindependent diabetes no longer appear in the classification. These terms were often misused, frequently reflecting the type of treatment that the patients were receiving rather than their actual type of diabetes. In addition, there is now greater certainty that type 1 diabe tes is due primarily to 6 pancreatic β-cell destruction, and this frequently can be documented by the presence of specific types of antibodies, as well as insulinopenia. The class of malnutrition -related diabetes is no longer recognized, as this was heterogenous, and one of its forms, fibrocalculous pancreatic diabetes, is clearly secondary to a specific form of pancreatic disease. The other form of diabetes formerly described as “malnutrition related” is now generally considered to be type 2 diabetes that occurs in malnourished persons. Knowledge of specific forms of diabetes attributable to mutations in specific genes has expanded enormously, and such cases can now be classified very specifically if they are investigated in detail. Importantly, this informat ion has led to circumstances in which susceptibility to the development of some types of diabetes can be identified before hypergl ycemia is evident. The introduction of clinical staging has been prompted by the recognition that all forms of diabetes pass through a number of stages associated with different degrees of metabolic dysfunction. Furthermore, it now has been established that progression from normal glucose tolerance to overt diabetes can be halted or delayed and that reversion to less severe states of dysglycemia is possible. On the other hand, hypergl ycemia may be present, but specific information or knowledge enabling precise etiologic classification may not be available for many patients. Such individuals can be classified according to clinical stage in the absence of specific 7 knowledge of causation. Basic Level of knowledge and skills. Students must know: - Risk factors for DM. - Diagnostic criteria for gestational diabetes, impaired fasting glucose (IFG), impaired glucose tolerance (IGT). - Differentiation diagnostic of DM. Diabetes and abnormalities in glucose-stimulated insulin secretion. - Clinical classification of DM: clinical classes, stages of development, stages of severity and compensation. - Manifestation particularity of Type 1 DM. - Temporal model for development of type 1 diabetes.Clinical fitches. - Particularity of Type 2 DM development and clinic. Students should be able to: 1. examine the patient with DM; 2. estimate oral glucose tolerance test; 3. do differential diagnosis of DM; 4. evaluate dates of additional methods of investigations for diagnosis of DM. 5. make the basic clinical diagnosis of DM. 6. make plan of examination of patient with DM Organization structure of lesson: I. Overview of topical problem of DM. 10 min. II. Estimate basic level of knowledge – 10 min. III. Practical work – 40 min. Method of practical work 1. Work with patients in the wards of endocrinology department. 2. Examination of patients with DM: do history taking, palpation, percussion, auscultation. 8 3. Working out symptoms in syndromes, emphasize the main syndrome. 4. Make diagnosis plan of investigations. 5. Estimate dates of additional methods of investigations. 6. Working out the diagnosis of DM 7. Making and substantiation of the basis clinical diagnosis. Practical skills: students should acquire and use practical skills according to the matriculs of practical skills of the subject. ІV. Control of students’ knowledge and skills. 1. Feed back of the practical work. – 10 min. 2 Feed back of theoretic questions. – 10 min. V. Conclusion. Different, items. – 10 min. Illustrations. Work with CDs, floopy discs, audi- and video in the computers’ class-room. Methodological Instruction to Lesson № 2. Theme: Long-term (late) complications of diabetes mellitus: classification and diagnostic criteria. Clinical assessment, investigations and management. Diabetic foot: classification and diagnostic criteria. Clinical assessment, investigations. Management of chronic complication. Diabetes mellitus and pregnancy Hours: 2 h. Working place: classroom, endocrine department wards of the Sumy Region Hospital. Aim: 9 - to learn classification of DM chronic complication; - to learn pathogenic particularities of DM chronic complication development; - to learn signs, symptoms and diagnostic criteria of DM chronic complication; - to learn investigation methods of DM chronic complication; - to learn DM chronic complication management and prevention. Professional Motivation. Both Type 1 and Type 2 diabetes are severe chronic diseases that carry risks of serious chronic complications associated with considerable morbidity and mortality. The number of people with diabetes in the world is growing because the world population is growing, and it is growing older, and lifestyles are changing. The increase is greatest in developing countries, because this is where these changes are most dramatic. The prognosis of diabetes depends largely on the quality of care given, the drugs and technology available, and on the education of people with diabetes, their families and their health care providers. Investment in this will reap rewards in fewer and less severe complications, and longer and better lives The microvascular complications are specific for diabetes, and the most ominous are retinopathy, which is a common cause of blindness; nephropathy, which is a common cause of renal failure; and neuropathy, which can cause risk of foot ulcers and amputation. Also included are Charcot joints, and features of autonomic dysfunction, including sexual dysfunction. People with diabetes are at much increased risk 10 for arteriosclerotic diseases, which can lead to coronary heart disease (myocardial infarction, angina pectoris and heart failure), cerebrovascular disease leading to stroke, and intermittent claudication and ischaemic gangrene of the lower extremities. Certain hand abnormalities (carpal tunnel syndrome, Dupuytren's contracture), capsulitis in the shoulder, thick skin and limited joint mobility are also more common in diabetes. The microvascular complications are best prevented by as near-normalisation of blood glucose as possible. Early specific interventions can prevent or delay their progression, e.g. laser therapy of the retina at the optimal stage can prevent blindness. The atherosclerotic complications need a much wider approach: smoking control, dietary changes to prevent hyperlipidaemia, increased physical activity and, when necessary, antihypertensive therapy. In addition to technology and drugs, education of persons with diabetes and their families is most important for success in reaching these goals. Basic Level of knowledge and skills. Students must know: - Etiopatogenetic model for development of DM chronic complication. - Diagnostic criteria for DM chronic complication. - Differentiation diagnostic of DM chronic complication - Clinical classification of DM chronic complications (Diabetic Retinopathy, Diabetic Nephropathy, Diabetic Polyneuropathy/Mononeuropathy, Autonomic Neuropathy, Cardiovascular complication, Diabetic Foot): clinical classes, stages of development. - Clinical fitches, glycemic control, long-term complications treatment. Students should be able to: 11 1. examine of the patient with DM chronic complication; 2. estimate data of additional examination methods; 3. do differential diagnosis of DM chronic complication; 4. make plan of examination of patient with DM chronic complication; 5. make the DM clinical complication diagnosis; 6. make plan of DM chronic complication treatment. Organization structure of lesson: I. Overview of topical problem of DM. 10 min. II. Estimate basic level of knowledge – 10 min. III. Practical work – 40 min. Method of practical work 1. Work with patients in the wards of endocrinology department. 2. Examination of patients with chronic complication of DM: do history taking, palpation, percussion, auscultation, specific examination of foot. 3. Making provisional diagnosis. 4. Making diagnosis plan of investigations. 5. Estimate dates of additional methods of investigations. 6. Making and substantiation of the basis clinical diagnosis. 7. Making and substantiation of the DM clinical complication diagnosis. 8. Making plan of long-term complications treatment. 12 Practical skills: students should acquire and use practical skills according to the matriculs of practical skills of the subject. ІV. Control of students’ knowledge and skills. 1. Feed back of the practical work. – 10 min. 2. Feed back of theoretic questions. – 10 min. V. Conclusion. Different, items. – 10 min. Illustrations. Work with CDs, floopy discs, audi- and video in the computers’ class-room. Methodological Instructions to practical class 3 Theme: Management of type 2 diabetes mellitus. The diet. The oral hypoglycemic agents. Hours: 2 h. Working place: classroom, endocrine department wards of the Sumy Region Hospital. Aim: - to learn principles of metabolic disorders on Type 2 DM ; - to learn the main principles of diet therapy, exercise program and oral hypoglycemic agents administration; - to learn classification oral hypoglycemic preparations; - to learn pharmacokinetics of oral hypoglycemic preparations; - to learn complication of oral hypoglycemic preparations. Professional Motivation. Combination therapy of oral antidiabetic agents (OAD) with insulin is being increasingly considered in the treatment of 13 Type 2 diabetes. Although the strategies of antihyperglycaemic therapy have not changed significantly over the past few years, there are now two major components driving the increased application of combination therapy: a) the commitment of physicians to good glycaemic targets and b) the novel availability of a basal insulin with a 16- to 24-hour plateau profile. An abundance of clinical trials have shown that the fasting blood glucose levels and overall metabolic control obtained in a given patient with Type 2 diabetes are dependent on the targets set in the study. Also in primary care it has been shown that metabolic control strongly depends on the belief of physicians as to which would be the right targets. To achieve these targets the conventional strategy of diet and exercise and eventually OAD have to be employed in most cases. In Europe, metformin and sulphonylureas are the best studied first line drugs in monotherapy, but other options are alpha-glucosidase inhibitors or repaglinide. When glycaemic goals are not met by this regimen, a combination of OAD including nateglinide may be applied. However, practical experience shows that the need for more than two different OAD in a given patient calls for a combination therapy with insulin in order to meet the target. The complexity of multiple risk reduction that is intrinsic to the modern day management of Type 2 diabetes has resulted in sub-optimal implementation of prevention and treatment measures. The challenge of integrating guidelines and protocols into everyday treatment of a complex disease like diabetes is immense. This makes the need for populationbased systems of risk reduction even more pressing. The last two decades have been the era of large-scale clinical trials and molecular genetics. The next few years need to be the era of research into delivering effective healthcare. The effective use 14 of information technology is one instrument that can underpin quality care by providing accurate and timely information on the results of individual patient treatment in real time, and facilitate the tracking of patients over time for purposes of continuous improvement of care for designated populations or sub-populations. It is also invaluable for purposes of epidemiological research and can provide robust information in regard to quality of care and outcome data. Only if the treatment of diabetes is viewed from the public health as well as the individual perspective, are we likely to succeed in our efforts to deliver consistent evidence-based diabetes care. Type 2 diabetes is a disease primarily affecting pancreatic beta-cell function. The modern concept about treatment of Type 2 diabetes is that drugs should be initiated early on in the natural course of disease, as soon as HbA-|c increases above 6.5% despite diet and physical exercise. Metformin and a rapid-acting insulin secretagogue (nateglinide) are the drugs of choice in initial treatment of Type 2 diabetes. If HbA-|C increases over 6.5% despite oral drugs, insulin has to be initiated. The first step is evening administration of insulin glargine. However, in most cases a rapid-acting insulin analogue at meals is needed as well. Diet and metformin should be continued to prevent excessive increase in body weight. Although blood glucose control is but one part of the wider metabolic disturbance found in most people with Type 2 diabetes, the understanding that any level of hyperglycaemia above the normal range is associated with increased risk of arterial damage is driving earlier and earlier consideration of initiation of insulin therapy. Accordingly, target levels for people with Type 2 diabetes are now below those for people with Type 1 diabetes, a reversal of perceived attitudes from 15 past decades. Basic Level of knowledge and skills. Students must know: - Patophysiology of Type 2 DM. - Classification and basis characteristics of oral hypoglycemic preparations. - Principles of oral hypoglycemic therapy. - Nutritional recomendation. Students should be able to: 1. make plan of administrate oral hypoglycemic preparations; 2. use principles of glycaemic control; 3. administrate hypocaloric diets. Organization structure of lesson: I. Overview of topical problem of treatment of Type 2 DM. - 10 min. II. Estimate basis level of knowledge – 10 min. III. Practical work – 40 min. Method of practical work 1. Examine patients with Type 2 DM. 2. Estimate dates of additional methods of investigations. 3. Making and substantiation of the clinical diagnosis. 4. Making the plan of administrate oral hypoglycemic preparations. 5. Correct the caloric intake and exercises. Practical skills: students should do practical skills according to the matriculs of practical skills of the subject. ІV. Control of students’ knowledges and skills. 1. Discussion of the practical work. – 10 min. 16 2. Discussion of theoretic questions. – 10 min. V. Conclusion. Organization question. – 10 min. Illustrations. Work with compacktdiscs, floopy discs, audi- and video in the computers class-room. Methodological instructions to practical class 4 Theme: The main principles of Type 1 diabetes mellitus therapy. Insulin therapy and its complications. Hours: 2 h. Working place: classroom, endocrine department wards of the Sumy Region Hospital. Aim: - to learn principles of physiological insulin secretion; - to learn principles of insulin replacement therapy; - to learn classification of insulin; - pharmacokinetics of insulin preparations ; - to learn schemes of insulin regimens; - to learn complication of insulin therapy. Professional Motivation. The real milestone in diabetes care obviously is the discovery of insulin and the start of treatment with insulin, a little more than 80 years ago. Since then we have witnessed great progress in diabetes care, indeed also related to insulin therapy, where new modifications of the molecule make treatment not only easier, but also more efficient, as discussed in our meeting today. The DCCTand UKPDS studies have definitely established that in Type 1 as well as in Type 2 diabetes, long-term near- 17 normoglycaemia strongly protects against onset and/or progression of microangiopathic complications. Therefore, implementation of strategies to maintain long-term near-normoglycaemia as early as possible, is of key importance in the management of both types of diabetes. In Type 1 diabetes, rapid-acting insulin analogues should be given at each meal and basal insulin substituted either as evening administration of the long-acting insulin analogue glargine or continuous subcutaneous insulin infusion. Physiological insulin secretion is of two types: continuous basal insulin secretion and incremental prandial insulin secretion, controlling meal-related glucose excursions. Designing insulin treatment programmes that mimic the physiological insulin secretion profile has been a vexing problem for clinicians and researchers. This has led to a number of advances designed to more closely approximate an ideal insulin absorption pattern. The ability of patients to self-monitor blood glucose has been a major development, which is constantly evolving in sophistication for patient safety and convenience. Advances in islet cell transplantation and the potential of stem cells may essentially result in patients' independence from exogenous insulin administration. Basic Level of knowledge and skills. Students must know: 1. Physiological insulin secretion model. 2. Classification and basis characteristics of insulins. 3. Methods of insulin injection. 4. Methods of coordination the caloric intake with the appropriate amount of insulin. Students should be able to: 18 a. do insulin injection; b. make plan of administrate and correct of insulin regimens; c. use insulin penfill ; d. use glucometer; e. administrate medical nutrition therapy (MNT). Organization structure of lesson: I. Overview of topical problem of DM. 10 min. II. Estimate basis level of knowledge – 10 min. III. Practical work – 40 min. Method of practical work 1. Examine patients with Type 1 DM. 2. Test the glucose blood level. 3. Estimate dates of additional methods of investigations. 4. Making and substantiation of the clinical diagnosis. 5. Making the plan of administrate insulin regimens. 6. Correct the caloric intake with the appropriate amount of insulin. Practical skills: students should do practical skills according to the matriculs of practical skills of the subject. ІV. Control of students’ knowledges and skills. 1. Discussion of the practical work. – 10 min. 2. Discussion of theoretic questions. – 10 min. V. Conclusion. Organization question. – 10 min. Illustrations. Work with compacktdiscs, floopy discs, audi- and video in the computers class-room. 19 Methodological instructions to practical class 5 Theme: Regimens of insulin therapy. Complications of insulin therapy. Curation of the patient to fill in a case history. Hours: 2 h. Working place: classroom, endocrine department wards of the Sumy Region Hospital. Aim: Curation of the patient to fill in a case history. Professional Motivation. Because endocrinology interfaces with numerous physiologic systems, there is no standard endocrine history and examination. Moreover, because most glands are relatively inaccessible, the examination usually focuses on the manifestations of hormone excess or deficiency, as well as direct examination of palpable glands, such as the thyroid and gonads. For these reasons, it is important to evaluate patients in the context of their presenting symptoms, review of systems, family and social history, and exposure to medications that may affect the endocrine system. Astute clinical skills are required to detect subtle symptoms and signs suggestive of underlying endocrine disease. For example, a patient with Cushing's syndrome may manifest specific findings, such as central fat redistribution, striae, and proximal muscle weakness, in addition to features seen commonly in the general population, such as obesity, plethora, hypertension, and glucose intolerance. Similarly, the insidious onset of hypothyroidism—with mental slowing, fatigue, dry skin, and other features—can be difficult to distinguish from similar, nonspecific findings in the general population. Clinical judgment, based on knowledge of disease prevalence and pathophysiology, is required to decide when to embark on 20 more extensive evaluation of these disorders. Laboratory testing plays an essential role in endocrinology by allowing quantitative assessment of hormone levels and dynamics. Radiologic imaging tests, such as computed tomography (CT) scan, magnetic resonance imaging (MRI), thyroid scan, and ultrasound, are also used for the diagnosis of endocrine disorders. However, these tests are generally employed only after a hormonal abnormality has been established by biochemical testing. Methodological instructions to practical class 6 Theme: Pathogenesis, diagnosis and treatment of hyperglycemic and hypoglycemic coma in patients with diabetes mellitus. Defence of a case history. Hours: 2 h. Working place: classroom, endocrine department wards of the Sumy Region Hospital. Aim: - to learn etiology, pathogenesis of acute complication of DM; - to learn diagnostic criteria, clinical manifistation and management of diabetic ketoacidosis (DKA); - to learn diagnostic criteria, clinical manifistation and management of nonketonic hyperglycemic hyperosmolar coma (NKHHC); - to learn diagnostic criteria, clinical manifistation and management of lactoacidosis (LA); - to learn diagnostic criteria, clinical manifistation and management of hypoglycemic coma (HC). 21 Professional Motivation. DKA is the most frequent endocrine emergency seen by the primary care physician. Mortality rates from 6 – 10 % have been reported. All the abnormalities associated with DKA can be traced to an absolute or relative insulin lack, which develops over the period of several hours or days. The symptoms and physical signs of DKA usually develop over 24 hours. DKA may be the initial symptom complex that leads to a diagnosis of type 1 DM, but more frequently it occurs in individuals with established diabetes. Nausea and vomiting are often prominent, and their presence in an individual with diabetes warrants laboratory evaluation for DKA. Abdominal pain may be severe and can resemble acute pancreatitis or ruptured viscous. Acute disorders are associated with potentially serious complications if not promptly diagnosed and treated. Using management strategies, early DKA can be prevented or detected and treated appropriately on an outpatient basis. Basic Level of knowledge and skills. Basic Level. 1. Etiology and pathogenesis of DM acute complication. Pathogenesis of insulin lack. 2. Laboratory values in DKA, hyperglycemic hyperosmolar state (HHS), LA, HC. 3. Clinical features of DKA, HHS, LA, HC. 4. Management strategies of DM patients with coma. 5. Differentional diagnostic of DM acute complication. 22 Students should be able to: 1. examine patients with DM acute complication 2. confirm diagnosis of DKA, HHS, LA, HC; 3. make plan of DM acute complication administrate; 4. use principles of clinical-laboratory monitoring; Organization structure of lesson: I. Overview of topical problem of DM acute complication. 10 min. II. Estimate basis level of knowledge – 10 min. III. Practical work – 40 min. Method of practical work 1. Examine patients with DM acute complication. 2. Confirm diagnosis. Assess patient. Estimate dates of investigations. 3. Making the plan of administrate: replace fluids, administer regular insulin. 4. Measure capillary glucose every 1–2 h; measure electrolytes. 5. Monitor blood pressure, pulse, respirations, mental status, fluid intake. Practical skills: students should do practical skills according to the matriculs of practical skills of the subject. ІV. Control of students’ knowledges and skills. 1. Discussion of the practical work. – 10 min. 2. Discussion of theoretic questions. – 10 min. V. Conclusion. Organization question. – 10 min. Illustrations. Work with compacktdiscs, floopy discs, audi- and video in the computers class-room. 23 References for unit 1: 1. Manual of Endocrinology and Metabolism (Second Edition)/ Norman Lavin. – Little, Brown and Company.- Boston-New York-Toronto-London, 1994. - P. 111 – 142, 173 - 180.. 2. Endocrinology (A Logical Approach for Clinicians (Second Edition)). William Jubiz.-New York: WC Graw-Hill Book, 1985. - P. 38 – 42, 144 –164, 198 – 205. 3. Short Textbook of Medical Diagnosis and Management (Third Edition). Mohammad Inam Danish. – Pakistan, 2002. – P.459 – 462, 504 – 505. Unit 2. Thyreoid gland disease Methodological instructions to practical class 8 Theme: Hyperthyroidism. Graves' disease. Classification, etiopathogenesis, diagnostic criteria, treatment. Hyperhtyroid crisis. Hours: 2 h. Working place: classroom, endocrine department wards of the Sumy Region Hospital. Aim: - to learn anatomy, physiology (localization, normal sizes and weight) of thyroid gland, - to learn regulation of thyroid gland function, - to learn biological effects of thyroid hormones’ action, - to learn hormone excess causes; - to learn etiology, pathogenesis and classification of the hyperthyroidism; - to learn signs, symptoms and diagnostic criteria of hyperthyroidism, 24 - to learn diagnostic criteria and treatment Graves' disease, - to learn diagnostic and management of hyperhtyroid crisis, - to learn medical preparations which can be used for patients with hyperthyroidism. Professional Motivation. Most cases of hyperthyroidism are caused by Grave’s diseases, multinodular goitre or autonomously functioning thyroid nodule (toxic adenoma). Graves' disease accounts for 60 to 80% of thyrotoxicosis, but the prevalence varies among populations, depending mainly on iodine intake (high iodine intake is associated with an increased prevalence of Graves' disease). Grave’s disease is distinguished clinically from other forms of hyperthyroidism by the presence of diffuse thyroid enlargement, ophthalmopathy and rarely pretibial myxoedema. It can occur at any age but is unusual before puberty and most commonly affects the 30-50 year-old age group. Clinical features generally worsen without treatment; mortality was 10 to 30% before the introduction of satisfactory therapy. Some patients with mild Graves' disease experience spontaneous relapses and remissions. Rarely, there may be fluctuation between hypo- and hyperthyroidism due to changes in the functional activity of TSH-R antibodies. Life-threatening increase in the severity of the clinical features of hyperthyroidism leads to hyperthyroid crisis. It is a medical emergency and, despite early recognition and treatment, the mortality rate is high. 25 Basic Level of knowledge and skills. Students must know: 1. Thyroid hormone secretion and action mechanisms. 2. Etiologic classification of hyperthyroidism. 3. Clinical classification of hyperthyroidism: stages of development, goiter degree. 4. Clinical features of hyperthyroidism. 5. Criteria for the diagnosis of hyperthyroidism. 6. Main principles of thyreostatic therapy. 7. Grave’s disease: etiopathogenesis, autoimmune factors, environmental factors, immunological markers, differential diagnosis and management. 8. Hyperthyroidism complications. 9. Hyperhtyroid crisis. Students should be able to: 1. Examine the patient with hyperthyroidism. 2. Emphasize clinical sings of hyperthyroidism. 3. Evaluate dates of clinical methods of investigations for diagnosis of hyperthyroidism. 4. Make provisional diagnosis of hyperthyroidism. 5. Make the plan of examination of a patient with hyperthyroidism. 6. Estimate dates of additional methods of investigations. 7. Make the plan of hyperhtyroid crisis management. Organization structure of lesson: I. Overview of topical problem of DM. 10 min. II. Emphasize basic level of knowledge – 10 min. III. Practical work – 40 min. Method of practical work 26 1. Work with patient in the wards of endocrinology department. 2. Examination of patients with hyperthyroidism: do history taking, palpation, percussion, auscultation. 3. Work out symptoms in syndromes, emphasize on the main syndrome. 4. Making diagnosis plan of investigations. 5. Making and substantiation of the basic clinical diagnosis. 6. Making the plan of the hyperthyroidism treatment. Practical skills: students should acquire and use practical skills according to the matriculs of practical skills of the subject. ІV. Control of students’ knowledge and skills. 1. Feed back of the practical work. – 10 min. 2. Feed back of theoretical questions. – 10 min. V. Conclusion. Different, items. – 10 min. Illustrations. Work with CDs, floopy discs, audi- and video in the computers’ class-room. Methodological instructions to practical class 9 Theme: Hypothyroidism: diagnostic criteria, treatment. Thyroiditis: classification, differentiation diagnostic, clinical assessment, investigations and management. Myxoedema coma. Hours: 2 h. Working place: classroom, endocrine department wards of the Sumy Region Hospital. Aim: 27 - to learn hormone deficiency causes; - to learn etiology, pathogenesis and classification of the hypothyroidism; - to learn signs, symptoms and diagnostic criteria of hypothyroidism and chronic complications, - to learn classification, diagnostic criteria and treatment of the thyroiditis, - to learn diagnostic and management of myxoedema coma. Professional Motivation. The prevalence of primary hypothyroidism is 1:100 but increases to 5:100 if patients with subclinical hypothyroidism (normal T4, raised TSH) are included. The female: male is approximately 6:1. There are various causes of primary hypothyroidism, but spontaneous atrophic hypothyroidism, thyroid failure following I131 or surgical treatment of hyperthyroidism, and the hypothyroidism of Hashimoto’s thyroiditis account for over 90% of cases in those parts of the world which are not significantly iodine-deficient. In some patients there is a history of Graves' disease treated with antithyroid drugs 10-20 years earlier and. very occasionally, patients with this form of hypothyroidism develop Graves' disease. As with any of the immunologically mediated thyroid disorders, patients are at risk of developing other organ-specific autoimmune conditions such as type 1 diabetes mellitus, pernicious anaemia and Addison's disease, and autoimmune disease is not uncommon in first- and seconddegree relatives This is the most common cause of goitrous hypothyroidism. It typically affects 20-60-year-old women who present with a small or moderately sized diffuse goitre 28 which is characteristically firm or rubbery in consistency. Thyroid status depends upon the relative degrees of lymphocytic infiltration, fibrosis and follicular cell hyperplasia within the gland but 25% of patients are hypothyroid at presentation. 1. 2. 3. 4. 5. 6. 7. 8. 9. 1. 2. 3. 4. 5. 6. 7. Basic Level of knowledge and skills. Students must know: Thyroid hormone secretion and action mechanisms. Etiologic classification of hypothyroidism. Clinical classification of hypothyroidism. Clinical features of hypohyroidism. Criteria for the diagnosis of hypothyroidism Mane principles of replacement therapy. Thyroiditis: classification, etiopathogenesis, differential diagnosis and management. Hypohyroidism complications. Myxoedema coma. Students should be able to: Examination of the patient with hypothyroidism. Emphasize clinical sings of hypothyroidism. Evaluate dates of clinical methods of investigations for diagnosis of hyporthyroidism. Make the plan of examination of patient with hypothyroidism. Estimate dates of additional methods of investigations. Make diagnosis of patients with hypothyroidism. Make the plan of myxoedema coma management. Organization structure of lesson: I. Overview of topical problem of the hypothyroidism. 10 min. II. Estimate basis level of knowledge – 10 min. III. Practical work – 40 min. 29 Method of practical work 8. Work near the bed of patient in the wards of endocrinology department. 9. Examine patients with hyperthyroidism: do history taking, palpation, percussion, auscultation. 10. Combine symptoms in syndromes, emphasize the main syndrome. 11. Make diagnosis plan of investigations. 12. Differential the diagnosis of hyperthyroidism. 13. Making and substantiation of the basis clinical diagnosis. 14. Make the plan of the hyperthyroidism treatment. Practical skills: students should do practical skills according to the matriculs of practical skills of the subject. ІV. Control of students’ knowledges and skills. 1. Discussion of the practical work. – 10 min. 2. Discussion of theoretic questions. – 10 min. V. Conclusion. Organization question. – 10 min. Illustrations. Work with compacktdiscs, floopy discs, audi- and video in the computers class-room. Methodological instructions to practical class 10 Theme: Nodule goiter. Diseases of the parathyroid glands. Hours: 2. Working place: lection hall of the Sumy Region Hospital. Aim: - to learn anatomy and physiology of parathyroid glands. 30 - to learn main functions of сalcitonin and parathyroid hormone. - to learn biological effects of calcium on organism. - to learn causes parathyroid hormone excess and deficientcy; - to learn etiology, pathogenesis of the hypoparathyroidism, hyperparathyroidism; - to learn signs, symptoms and diagnostic criteria of hypoparathyroidism, hyperparathyroidism; - to learn iodine deficiency: clinical assessment, investigations, differentiation diagnostic and management; - to learn thyroid nodules, malignant tumors. Professional Motivation. Patients with primary hyperparathyroidism may have a chronic, non-specific history. Their symptoms are brought to mind by the adage 'bones, stones and abdominal groans'. However, about 50% of patients with primary hyperparathyroidism are asymptomatic. In others, symptoms may go unrecognized until patients present with renal calculi (5% of first stone formers and 15% of recurrent stone formers have primary hyperparathyroidism), with or without impaired renal function, or acute dehydration and profound hypercalcaemia. Hypertension is common in hyperparathyroidism. Parathyroid tumours are almost never palpable. The most common cause of hypoparathyroidism is damage to the parathyroid glands (or their blood supply) during thyroid surgery, although this complication is only permanent in 1% of thyroidectomies. Transient hypocalcaemia develops in 10% of patients 12—36 hours following subtotal thyroidectomy 31 for Graves' disease. Idiopathic hypoparathyroidism may develop at any age, and is sometimes associated with autoimmune disease of the adrenal, thyroid or ovary, especially in young people. Simple multinodular goiter is rare before middle age. The patient may have been aware of the goitre for many years, perhaps slowly increasing in size. Rarely, medical advice may have been sought because of painful swelling lasting a few days caused by haemorrhage into a nodule or cyst. The goitre is nodular or tabulated on palpation and may extend retrosternally. Very large goitres may cause mediastinal compression with stridor, dysphagia and obstruction of the superior vena cava. Hoarseness due to recurrent laryngeal nerve palsy can occur but is far more suggestive of thyroid carcinoma. Primary thyroid malignancy is rare, accounting for less than 1% of all carcinomas, and has a prevalence of 25 per million. With the exception of medullary carcinoma, thyroid cancer is always more common in females. Papillary carcinoma is the most common of the malignant thyroid tumours and account for 90% of irradiation-induced thyroid cancer. Metastases are most often found in bone, lung and brain. Even for patients with distant metastases at presentation, the 10-year survival is approximately 40%. Myxoedema coma is a medical emergency in which there is a depressed level of consciousness, usually in an elderly patient who appears myxoedematous. The mortality rate is 50% and survival depends upon early recognition and treatment of hypothyroidism and other factors contributing to the altered consciousness level, e.g. drugs such as phenothiazines, cardiac failure, pneumonia, dilution hyponatraemia, hypoxaemia and hypercapnia due to 32 hypoventilation. Mane questions of the lection: 1. Hypoparathyroidism: classification, etiopathogenesis, differential diagnosis and management. 2. Hyperparathyroidism: classification, etiopathogenesis, differential diagnosis and management. 3. Iodine deficiency: clinical assessment, investigations and management. 4. Thyroid nodules: classification, pathogenesis, differential diagnosis and management. 5. Malignant tumors: clinical assessment, investigations, differentiation diagnostic and management. Technical equipment: multi-media presentation. Methodological instructions to practical class 11 Theme: Hypoparathyroidism. Hyperparathyroidism. Diagnostic criteria. Clinical assessment, investigations and management. Iodine deficiency. Thyroid nodules. Malignant tumors. Clinical assessment, investigations, differentiation diagnostic and management. Hours: 2 h. Working place: classroom, endocrine department wards of the Sumy Region Hospital. Aim: - to learn anatomy and physiology of parathyroid glands. - to learn main functions of сalcitonin and parathyroid hormone. 33 - to learn biological effects of calcium on organism. - to learn causes parathyroid hormone excess and deficientcy; - to learn etiology, pathogenesis of the hypoparathyroidism, hyperparathyroidism; - to learn signs, symptoms and diagnostic criteria of hypoparathyroidism, hyperparathyroidism; - to learn iodine deficiency: clinical assessment, investigations, differentiation diagnostic and management; - to learn thyroid nodules, malignant tumors. Professional Motivation. Investigation of calcium metabolism is usually straightforward. Most laboratories measure total calcium in serum. Differential diagnosis of disorders of calcium metabolism requires measurement of phosphate, alkaline phosphatase and sometimes PTH (for which the blood sample has to be taken to the laboratory 'on ice' and centrifuged rapidly). Hvpercalcaemia is one of the most common biochemical abnormalities. It is detected most frequently during routine , biochemical analysis in asymptomatic patients. However, it can present with chronic symptoms, and occasionally patients present as acute emergencies with severe hvpercalcaemia and dehydration. Hvpercalcaemia is one of the most common biochemical abnormalities. It is detected most frequently during routine, biochemical analysis in asymptomatic patients. However, it can present with chronic symptoms, and occasionally patients present as acute emergencies with severe hvpercalcaemia and dehydration. Patients with primary hyperparathyroidism may have a 34 chronic, non-specific history. Their symptoms are brought to mind by the adage 'bones, stones and abdominal groans'. However, about 50% of patients with primary hyperparathyroidism are asymptomatic. In others, symptoms may go unrecognized until patients present with renal calculi (5% of first stone formers and 15% of recurrent stone formers have primary hyperparathyroidism), with or without impaired renal function, or acute dehydration and profound hypercalcaemia. Hypertension is common in hyperparathyroidism. Parathyroid tumours are almost never palpable. A family history of renal tract stones and/or neck surgery raises the possibility of multiple endocrine neoplasia. Familial hypocalciuric hypercalcaemia is a rare but important catch for the unwary. This autosomal dominant disorder is associated with a defective calcium receptor in the parathyroid gland, but is almost always asymptomatic and uncomplicated. Occasionally, these patients have had their parathyroid glands removed unnecessarily. Hypocalcaemia is much less common than hypercalcaemia. Although almost all laboratories routinely report total serum calcium concentrations, it is the ionised concentration which is biologically important. The most common cause of hypocalcaemia is a low serum albumin with normal ionised calcium concentration. The most common cause of hypoparathyroidism is damage to the parathyroid glands (or their blood supply) during thyroid surgery, although this complication is only permanent in 1% of thyroidectomies. Transient hypocalcaemia develops in 10% of patients 12—36 hours following subtotal thyroidectomy for Graves' disease. Idiopathic hypoparathyroidism may develop at any age, 35 and is sometimes associated with autoimmune disease of the adrenal, thyroid or ovary, especially in young people. In certain parts of the world, such as the Andes, the Himalayas and central Africa, where there is dietary iodine deficiency, thyroid enlargement is common (more than 10% of the population) and is known as endemic goitre. Most patients are euthyroid and have normal or raised TSH levels. In general the more severe the iodine deficiency, the greater the incidence of hypothyroidism. Simple multinodular goiter is rare before middle age. The patient may have been aware of a goitre for many years, perhaps slowly increasing in size. Rarely, medical advice may have been sought because of painful swelling lasting a few days caused by haemorrhage into a nodule or cyst. The goitre is nodular or tabulated on palpation and may extend retrosternally. Very large goitres may cause mediastinal compression with stridor, dysphagia and obstruction of the superior vena cava. Hoarseness due to recurrent laryngeal nerve palsy can occur but is far more suggestive of thyroid carcinoma. Primary thyroid malignancy is rare, accounting for less than 1% of all carcinomas, and has a prevalence of 25 per million. With the exception of medullary carcinoma, thyroid cancer is always more common in females. Papillary carcinoma is the most common of the malignant thyroid tumours and account for 90% of irradiation-induced thyroid cancer. Metastases are most often found in bone, lung and brain. Even for patients with distant metastases at presentation, the 10-year survival is approximately 40%. Myxoedema coma is a medical emergency in which there is a depressed level of consciousness, usually in an elderly patient who appears myxoedematous. The mortality 36 rate is 50% and survival depends upon early recognition and treatment of hypothyroidism and other factors contributing to the altered consciousness level, e.g. drugs such as phenothiazines, cardiac failure, pneumonia, dilution hyponatraemia, hypoxaemia and hypercapnia due to hypoventilation. Basic Level of knowledge and skills. Students must know: 10. Hypoparathyroidism: classification, etiopathogenesis, differential diagnosis and management. 11. Hyperparathyroidism: classification, etiopathogenesis, differential diagnosis and management. 12. Iodine deficiency: clinical assessment, investigations and management. 13. Thyroid nodules: classification, pathogenesis, differential diagnosis and management. 14. Malignant tumors: clinical assessment, investigations, differentiation diagnostic and management. Students should be able to: 8. Examination of the patient with hypothyroidism. 9. Emphasize clinical sings of hypothyroidism. 10. Evaluate dates of clinical methods of investigations for diagnosis of hyporthyroidism. 11. Make the plan of examination of patient with hypothyroidism. 12. Estimate dates of additional methods of investigations. 13. Make diagnosis of patients with hypothyroidism. 14. Make the plan of myxoedema coma management. Organization structure of lesson: I. Overview of topical problem of DM. 10 min. 37 II. Estimate basis level of knowledge – 10 min. III. Practical work – 40 min. Method of practical work 15. Work near the bed of patient in the wards of endocrinology department. 16. Examine patients: do history taking, palpation, percussion, auscultation. 17. Combine symptoms in syndromes, emphasize the main syndrome. 18. Make diagnosis plan of investigations. 19. Making and substantiation of the basis clinical diagnosis. 20. Make the plan of the treatment. Practical skills: students should do practical skills according to the matriculs of practical skills of the subject. ІV. Control of students’ knowledges and skills. 1. Discussion of the practical work. – 10 min. 2. Discussion of theoretic questions. – 10 min. V. Conclusion. Organization question. – 10 min. Illustrations. Work with compacktdiscs, floopy discs, audi- and video in the computers class-room. References for unit 2: 1. Weetman A.P: Graves' disease. N Engl J Med 343:1236, 2000 2. Schwarnz K.M. et al: Dermopathy of Graves' disease (pretibial myxedema): Long-term outcome. J Clin Endocrinol Metab 87:438, 2002 38 3. Manual of Endocrinology and Metabolism (Second Edition)/ Norman Lavin. – Little, Brown and Company.- Boston-New York-Toronto-London, 1994. - P. 111 – 142, 173 - 180.. 4. Endocrinology (A Logical Approach for Clinicians (Second Edition)). William Jubiz.-New York: WC Graw-Hill Book, 1985. - P. 38 – 42, 144 –164, 198 – 205. 5. Short Textbook of Medical Diagnosis and Management (Third Edition). Mohammad Inam Danish. – Pakistan, 2002. – P.459 – 462, 504 – 505. 6. Best, C. H., and Taylor, N. B. (1966): The Thyroid gland, Chapter 75. In: The Physiological Basis of Medical Practice, 8th Edition, pp. 1529-1554. Williams & Wilkins Co., Baltimore. 7. Ladenson P.W. et al: American Thyroid Association guidelines for detection of thyroid dysfunction. Arch Intern Med 160:1573, 2000 8. Braga M. et al: Efficacy of ultrasound-guided fine-needle aspiration biopsy in the diagnosis of complex thyroid nodules. J Clin Endocrinol Metab 86:4089, 2001 9. Degroot L.J. et al: Thyroid gland (Part III), in Endocrinology, 4th ed, LJ DeGroot, JL Jameson (eds). Philadelphia, Saunders, 2000 10. Gharib H: Changing concepts in the diagnosis and management of thyroid nodules. Endocrinol Metab Clin North Am 26:777, 1997 11. Klein I., Ojamaa K: Thyroid hormone and the cardiovascular system. N Engl J Med 344:501, 2001 12. Rapoport B., Mclachlan S.M.: Thyroid autoimmunity. J Clin Invest 108:1253, 2001 39 Unit 3. Adrenal gland diseases Methodological instructions to practical class 12 Topic: Hypofunction of the adrenal cortex. Classification of adrenocortical insufficiency. Clinical assessment, investigations and management of the chronic adrenocortical insufficiency. Acute adrenocortical insufficiency. Congenital adrenal hyperplasia. Adrenal tumours. Phaeochromocytoma. Aldosteronism. Clinical assessment, diagnostic criteria, treatment. Hours: 2 h. Working place: classroom, endocrine department wards of the Sumy Region Hospital. Aim: - to learn anatomy, physiology (localization, normal sizes and weight) of the of the adrenal, - to learn regulation of endocrine function, - to learn biological effects of glucocorticoid hormones’ action, - to learn etiology, pathogenesis and classification of adrenocortical insufficiency; - to learn signs, symptoms and diagnostic criteria of the chronic adrenocortical insufficiency; - to learn diagnostic and management of congenital adrenal hyperplasia, adrenal tumours, - to learn diagnostic criteria and management of phaeochromocytoma; - to learn diagnostic criteria and treatment of aldosteronism; - to learn diagnostic and management of acute adrenocortical insufficiency. 40 Professional Motivation. Adrenal insufficiency results from inadequate secretion of cortisol and/or aldosterone. It is potentially fatal and notoriously variable in its presentation. A high index of suspicion is therefore required in patients with unexplained fatigue, hyponatraemia or hypotension. The most common is ACTH deficiency (i.e. secondary adrenocortical failure), usually because of inappropriate withdrawal of chronic glucocorticoid therapy or a pituitary tumour. Congenital adrenal hyperplasias and Addison's disease (i.e. primary adrenocortical failure) are rare, although in areas where AIDS and tuberculosis are common associated Addison's disease is increasing in prevalence. In Addison's disease, either glucocorticoid or mineralocorticoid deficiency may come first, but eventually all patients fail to secrete both classes of corticosteroid. Similar features occur in different combinations with other causes of adrenocortical insufficiency. Patients may present with chronic features and/or in acute circulatory shock. With a chronic presentation, initial symptoms are often misdiagnosed (e.g. as chronic fatigue syndrome or depression). Adrenocortical insufficiency should also be considered in patients with hyponatraemia, even in the absence of symptoms. Vitiligo occurs in 10-20% of patients with autoimmune Addison's disease. Features of an acute adrenal crisis include circulatory shock with severe hypotension, hyponatraemia, hyperkalaemia and, in some instances, hypoglycaemia and hypercalcaemia. Muscle cramps, nausea, vomiting, diarrhoea and unexplained fever may be present. The crisis is often precipitated by intercurrent disease, surgery or infection. 41 Patients with adrenocortical insufficiency always need glucocorticoid replacement therapy and usually, but not always, mineralocorticoid. Other treatments depend on the underlying cause. Most often, causes of excessive activation of mineralocorticoid receptors results from enhanced secretion of renin (secondary hyperaldosteronism) in response to inadequate renal perfusion (e.g. in heart failure, hypoalbuminaemia or renal artery stenosis). Secondary hyperaldosteronism is not dealt with here. Less commonly, mineralocorticoid excess occurs in the face of suppressed renin secretion (primary hyperaldosteronism and rare disorders of mineralocorticoid action). These disorders are usually diagnosed in patients presenting with hypertension. Indications to test for primary hyperaldosteronism in hypertensive patients include hypokalaemia (including hypokalaemia induced by thiazide diuretics), poor control of blood pressure with conventional therapy, or presentation at a young age. The prevalence of primary hyperaldosteronism is controversial. If only hypertensive patients with hypokalaemia are investigated, then fewer than 1% of patients with hypertension will be found to have primary hyperaldosteronism. Around half of these have an adrenal adenoma secreting aldosterone (Conn's syndrome). However, recent studies in which hypertensive patients have been screened using aldosterone/renin ratios (see below) suggest that the prevalence may be as high as 5%. Most of these 'extra' patients have bilateral adrenal hyperplasia rather than Conn's syndrome, and many have normal plasma potassium. Although spironolactone would be the antihypertensive agent of choice in such patients, it remains to be determined whether investigation of all hypertensive patients for bilateral adrenal 42 hyperplasia is worth while. Excessive secretion of catecholamines can be confirmed by measuring the hormones (adrenaline, noradrenaline and dopamine) in plasma or their metabolites (e.g. vallinylmandelic acid, VMA; conjugated metanephrine and normetanephrine) in urine. However, catecholamine secretion is usually paroxysmal and sometimes the paroxysms are infrequent; in a patient with classical symptoms, phaeochromocytoma can only be excluded if the 24-hour urinary catecholamine excretion is normal on a day on which symptoms have occurred. Increased urinary catecholamine excretion occurs in stressed patients (e.g. after myocardial infarction or major surgery) and is induced by some drugs (notably β-blockers and antidepressants). For this reason, a suppression test may be valuable. Normal adrenomedullary secretion is suppressed by administration of drugs which interfere with sympathetic outflow, such as clonidine or pentolinium. In phaeochromocytoma these drugs do not suppress plasma catecholamines. Provocative tests of catecholamine release should not be used. Congenital adrenal hyperplasia: defects in the cortisol biosynthetic pathway result in impaired negative feedback and increased ACTH secretion. ACTH then stimulates the production of steroids up to the enzyme block. This produces adrenal hyperplasia and a combination of clinical features which depend on the severity and site of the defect in biosynthesis. All of these enzyme abnormalities are inherited as autosomal recessive traits. There is therefore a 1:4 chance that the sibling of an affected child will also have the disease, but a low risk of passing the disease to the next generation. The enzyme block leads to insufficiency of hormones 'distal' to the block (glucocorticoids and mineralocorticoids), 43 and impaired negative feedback suppression of ACTH leading to accumulation of precursor hormones 'proximal' to the block which 'spill over' into the adrenal androgen biosynthetic pathway. The severity of the mutation in the 21hydroxylase (21 OHase) gene determines which features are present. The most severely affected 'classical' patients present in infancy (salt-wasting in boys; ambiguous genitalia in girls). The least severely affected 'late-onset' patients present as adults (hirsutism in women). (DHEA = dehydroepiandrosterone) most common enzyme defect is 21-hydroxylase deficiency. In about one-third of cases this defect is severe. In the other two-thirds, mineralocorticoid secretion is not affected but there may be features of cortisol insufficiency and/or androgen excess. 1. 2. 3. 4. 5. 6. Basic Level of knowledge and skills. Students must know: Glucocorticoid hormones’ secretion and action mechanisms. Etiologic classification of chronic adrenocortical insufficiency. Clinical features of chronic adrenocortical insufficiency. Criteria for the diagnosis of phaeochromocytoma, aldosteronism, congenital adrenal hyperplasia. Main principles of adrenal tumours therapy. Acute adrenocortical insufficiency: etiopathogenesis, differential diagnosis and management. Students should be able to: 1. Examine the patient with chronic adrenocortical insufficiency. 2. Emphasize clinical sings of adrenocortical insufficiency. 3. Evaluate dates of clinical methods of investigations. 44 4. Make provisional diagnosis. 5. Make the plan of examination of a patient. 6. Estimate dates of additional methods of investigations. 7. Make the plan of chronic adrenocortical insufficienc management. 8. Make the plan of diagnosis and management of acute adrenocortical insufficiency. Organization structure of lesson: I. Overview of problem of adrenocortical insufficiency and adrenal tumours 10 min. II. Emphasize basic level of knowledge –10 min. III. Practical work –40 min. Methods of practical work: Work with patient in the wards of endocrinology department. Examination of patients with adrenocortical insufficiency or adrenal tumours: do history taking, palpation, percussion, auscultation. Work out symptoms in syndromes, emphasize on the main syndrome. Making diagnosis plan of investigations. Making and substantiation of the basic clinical diagnosis. Making the plan of the adrenocortical insufficiency and adrenal tumours diseases treatment. Practical skills: students should acquire and use practical skills according to the matriculs of practical skills of the subject. ІV. Control of students’ knowledge and skills. 1. Feed back of the practical work. – 10 min. 2. Feed back of theoretical questions. – 10 min. 45 V. Conclusion. Different, items. – 10 min. Illustrations. Work with CDs, floopy discs, audi- and video in the computers’ class-room. Methodological instructions to practical class 13 Topic: Hyperfunction of the adrenal cortex. Classification. Cushing’s syndrome. Hypothalamic syndrome of pubertal period. Clinical assessment, differentiation diagnostic and treatment. Hours: 2 h. Working place: classroom, endocrine department wards of the Sumy Region Hospital. Aim: - to learn biological effects of glucocorticoid hormones’ exess, - to learn etiology, pathogenesis and classification of adrenocortical hyperfunction; - to learn signs, symptoms and diagnostic criteria of the Cushing’s syndrome; - to learn diagnostic and management of Cushing’s syndrome, - to learn diagnostic criteria and management of hypothalamic syndrome of pubertal period. Professional Motivation. Amongst endogenous causes, pituitary-dependent cortisol excess (by convention, called Cushing's disease) accounts for ∼80% of cases. Both Cushing's disease and adrenal tumour are four times more common in women than men. In contrast, 46 ectopic ACTH syndrome (often due to a small-cell carcinoma of the bronchus) is more common in men. Many of the diverse manifestations of glucocorticoid excess are not specific to Cushing's syndrome and, because spontaneous Cushing's syndrome is rare, the positive predictive value of any one feature alone is low. Moreover, some common disorders can be confused with Cushing's syndrome because they are associated with alterations in cortisol secretion: for example, obesity and depression. Features which have the best predictive value in favour of Cushing's syndrome in an obese patient are bruising, myopathy and hypertension. Any clinical suspicion of cortisol excess is best resolved by further investigation. Some clinical features are more common in ectopic ACTH syndrome. Unlike pituitary tumours secreting ACTH, ectopic tumours have no residual negative feedback sensitivity to cortisol, and both ACTH and cortisol levels are usually higher than with other causes. Very high ACTH levels are associated with marked pigmentation. Very high cortisol levels overcome the barrier of 11β-HSD in the kidney and cause hypokalaemic alkalosis. Hypokalaemia aggravates both myopathy and hyperglycaemia (by inhibiting insulin secretion). When the tumour secreting ACTH is malignant (e.g. pancreatic or small-cell lung carcinomas), then the onset is usually rapid and may be associated with cachexia. For these reasons, the classical features of Cushing's syndrome are less common in ectopic ACTH syndrome, and if present suggest that a benign tumour (e.g. bronchial carcinoid) is responsible. In Cushing's disease the pituitary tumour is almost always a microadenoma (< 10 mm in diameter); hence other features of a pituitary macroadenoma (hypopituitarism, visual failure or disconnection hyperprolactinaemia) are rare. 47 Once the presence of Cushing's syndrome is established, measurement of plasma ACTH is key in establishing the differential diagnosis. In the presence of excess cortisol secretion, an undetectable ACTH indicates an adrenal tumour while any detectable ACTH is pathological. Tests to discriminate pituitary from ectopic sources of ACTH rely on the fact that pituitary tumours, but not ectopic tumours, retain some features of normal regulation of ACTH secretion. Thus, in Cushing's disease ACTH secretion is suppressed by dexamethasone, albeit at a higher dose than in health, and ACTH is stimulated by corticotrophin-releasing hormone (CRH). This is essential, as untreated Cushing's syndrome has a 50% 5-year mortality. Most patients are treated surgically with medical therapy given for a few weeks prior to operation. The type of surgery depends on the cause 1. 2. 3. 4. 5. Basic Level of knowledge and skills. Students must know: Action mechanisms of glucocorticoid hormones’ exess. Etiologic classification of chronic adrenocortical hyperfunction. Clinical features of chronic adrenocortical hyperfunction. Criteria for the diagnosis of Cushing’s syndrome, hypothalamic syndrome of pubertal period. Main principles of therapy adrenocortical hyperfunction. Students should be able to: 1. Examine the patient with adrenocortical hyperfunction. 2. Emphasize clinical sings of adrenocortical hyperfunction. 48 3. Evaluate dates of clinical methods of investigations. 4. Make provisional diagnosis. 5. Make the plan of examination of a patient. 6. Estimate dates of additional methods of investigations. 7. Make the plan of chronic adrenocortical hyperfunction management. Organization structure of lesson: I. Overview of problem of adrenocortical insufficiency and adrenal tumours 10 min. II. Emphasize basic level of knowledge – 10 min. III. Practical work – 40 min. Method of practical work 1. Work with patient in the wards of endocrinology department. 2. Examination of patients with Cushing’s syndrome or hypothalamic syndrome of pubertal period: do history taking, palpation, percussion, auscultation. 3. Work out symptoms in syndromes, emphasize on the main syndrome. 4. Making diagnosis plan of investigations. 5. Making and substantiation of the basic clinical diagnosis. 6. Making the plan of Cushing’s syndrome or hypothalamic syndrome of pubertal period treatment. Practical skills: students should acquire and use practical skills according to the matriculs of practical skills of the subject. ІV. Control of students’ knowledge and skills. 1. Feed back of the practical work. – 10 min. 2. Feed back of theoretical questions. – 10 min. 49 V. Conclusion. Different, items. – 10 min. Illustrations. Work with CDs, floopy discs, audi- and video in the computers’ class-room. References for unit 3: 1. Manual of Endocrinology and Metabolism (Second Edition)/ Norman Lavin. – Little, Brown and Company.- Boston-New York-Toronto-London, 1994. - P. 111 – 142, 173 - 180.. 2. Endocrinology (A Logical Approach for Clinicians (Second Edition)). William Jubiz.-New York: WC Graw-Hill Book, 1985. - P. 38 – 42, 144 –164, 198 – 205. 3. Short Textbook of Medical Diagnosis and Management (Third Edition). Mohammad Inam Danish. – Pakistan, 2002. – P.459 – 462, 504 – 505. Unit 4. Hypothalamic and pituitary diseases. Methodological instructions to practical class 14 Theme: Hypothalamic and pituitary diseases. Hypopituitarism. Obesity. Acromegaly. Gigantism. Pituitary dwarfism. Diabetes insipidus. Syndrome of inappropriate ADH secretion. Prolactinoma. Clinical assessment, investigations and management. Hours: 2 h. Working place: classroom, endocrine department wards of the Sumy Region Hospital. Aim: 50 - to learn anatomy, physiology (localization, normal sizes and weight) of the pituitary gland, - to learn regulation of endocrine function, - to learn biological effects of thyroid hormones’ action, - to learn hormone excess causes; - to learn etiology, pathogenesis and classification of the hypopituitarism; - to learn signs, symptoms and diagnostic criteria of hypopituitarism, obesity; - to learn diagnostic and management diabetes insipidus, syndrome of inappropriate ADH secretion, - to learn diagnostic criteria and management pituitary dwarfism; - to learn diagnostic criteria and treatment of acromegaly, gigantism; - to learn diagnostic and management of prolactinoma. Professional Motivation. The anterior pituitary is often referred to as the “master gland” because, together with the hypothalamus, it orchestrates the complex regulatory functions of multiple other endocrine glands. Pituitary tumors cause characteristic hormone excess syndromes. Hormone deficiency may be inherited or acquired. Fortunately, efficacious treatments exist for the various pituitary hormone excess and deficiency syndromes. Nonetheless, these diagnoses are often elusive, emphasizing the importance of recognizing subtle clinical manifestations and performing the correct laboratory diagnostic tests. The anterior pituitary is often referred to as the “master gland” because, together with the hypothalamus, it orchestrates the complex regulatory functions of multiple other endocrine 51 glands. The anterior pituitary gland produces six major hormones: (1) prolactin (PRL), (2) growth hormone (GH), (3) adrenocorticotropin hormone (ACTH), (4) luteinizing hormone (LH), (5) follicle-stimulating hormone (FSH), and (6) thyroidstimulating hormone (TSH). Pituitary hormones are secreted in a pulsatile manner, reflecting stimulation by an array of specific hypothalamic releasing factors. Each of these pituitary hormones elicits specific responses in peripheral target tissues. The hormonal products of these peripheral glands, in turn, exert feedback control at the level of the hypothalamus and pituitary to modulate pituitary function. Pituitary tumors cause characteristic hormone excess syndromes. Hormone deficiency may be inherited or acquired. Fortunately, efficacious treatments exist for the various pituitary hormone excess and deficiency syndromes. Nonetheless, these diagnoses are often elusive, emphasizing the importance of recognizing subtle clinical manifestations and performing the correct laboratory diagnostic tests. Diseases of the hypothalamus and pituitary are rare, with an annual incidence of ∼1:50 000. They are usually diagnosed in patients presenting with a classical syndrome of hormone excess (e.g. acromegaly or prolactinoma), hormone deficiency (e.g. hypopituitarism, isolated secondary hypogonadism or adrenal insufficiency) or a space-occupying lesion (headache and/or visual disturbance). The pituitary plays a central role in several major endocrine axes, so that investigation and treatment involve several other glands. The reader may usefully use the following section in part as revision of disorders of each gland which have been described above. 52 1. 2. 3. 4. 5. - Basic Level of knowledge and skills. Students must know: Pituitary hormone secretion and action mechanisms. Etiologic classification of hypopituitarism. Clinical features of hypopituitarism, diabetes insipidus, syndrome of inappropriate ADH secretion, Criteria for the diagnosis of pituitary dwarfism, acromegaly, gigantism, prolactinoma. Main principles of therapy. Hypopituitarism: etiopathogenesis, differential diagnosis and management. Students should be able to: Examine the patient with hypothalamic and pituitary diseases. Emphasize clinical sings of hypothalamic and pituitary diseases. Evaluate dates of clinical methods of investigations. Make provisional diagnosis. Make the plan of examination of a patient. Estimate dates of additional methods of investigations. Make the plan of hypothalamic and pituitary diseases management. Organization structure of lesson: I. Overview of topical problem of hypothalamic and pituitary diseases. - 10 min. II. Emphasize basic level of knowledge – 10 min. III. Practical work – 40 min. Method of practical work Work with patient in the wards of endocrinology department. Examination of patients with hypothalamic and pituitary diseases: do history taking, palpation, percussion, auscultation. 53 Work out symptoms in syndromes, emphasize on the main syndrome. Making diagnosis plan of investigations. Making and substantiation of the basic clinical diagnosis. Making the plan of the hypothalamic and pituitary diseases treatment. Practical skills: students should acquire and use practical skills according to the matriculs of practical skills of the subject. ІV. Control of students’ knowledge and skills. 1. Feed back of the practical work. – 10 min. 2. Feed back of theoretical questions. – 10 min. V. Conclusion. Different, items. – 10 min. Illustrations. Work with CDs, floopy discs, audi- and video in the computers’ class-room. References for unit 4: 1. Manual of Endocrinology and Metabolism (Second Edition)/ Norman Lavin. – Little, Brown and Company.- Boston-New York-Toronto-London, 1994. - P. 111 – 142, 173 - 180.. 2. Endocrinology (A Logical Approach for Clinicians (Second Edition)). William Jubiz.-New York: WC Graw-Hill Book, 1985. - P. 38 – 42, 144 –164, 198 – 205. 3. Short Textbook of Medical Diagnosis and Management (Third Edition). Mohammad Inam Danish. – Pakistan, 2002. – P.459 – 462, 504 – 505. 54 Unit 5. Diseases of the Reproductive system. Methodological instructions to practical class 14 Topic: Diseases of the Reproductive system. Delayed puberty. Sexual precocity. Disorders of sexual differentiation. Primary and secondary hypogonadism. The hermaphroditism. The menopause. Diagnosis and the differential diagnosis, management. Hours: 2 h. Working place: classroom, endocrine department wards of the Sumy Region Hospital. Aim: - to learn biological effects of sex hormones, - to learn male and female reproductive physiology, and the normal menstrual cycle, - to learn etiology, pathogenesis and classification of sexual differentiation disorders; - to learn etiology, pathogenesis and classification primary and secondary hypogonadism; - to learn signs, symptoms and diagnostic criteria of the hermaphroditism; - to learn diagnostic and management of menopause. Professional Motivation. Clinical practice in reproductive medicine is shared between several specialties, including gynaecology, urology, psychiatry and endocrinology. The following section focuses on aspects that are commonly managed by endocrinologists. The clinical features of primary (failure of the testis) and secondary (failure of the hypothalamus or anterior pituitary) hypogonadism are identical. These include loss of libido, lethargy with muscle weakness, and decreased frequency of shaving. Patients commonly present with gynaecomastia, 55 erectile impotence, infertility or delayed puberty. Male hypogonadism is confirmed by demonstrating a low serum testosterone level. The distinction between primary and secondary hypogonadism is made by measurement of random LH and FSH. Patients with hypogonadotrophic hypogonadism (i.e. secondary hypogonadism) should be investigated as described for pituitary disease. Patients with hypergonadotrophic hypogonadism (i.e. primary hypogonadism) should have the testes examined for cryptorchidism or tumours, measurement of serum ferritin (to exclude haemochromatosis) and a karyotype (to identify Klinefelter's syndrome, i.e. 47, XXY). If there is no obvious cause, then no further investigations are necessary. Psychotherapy which includes the sexual partner is most useful for psychological problems. Neuropathy and vascular disease are unlikely to improve, but several treatments are available. First-line therapy is usually with oral sildenafil, a phosphodiesterase inhibitor which potentiates the vasodilator action of nitric oxide on cyclic guanosine monophosphate (cGMP). Coadministration of sildenafil with nitric oxide donors ('nitrate' drugs) is contraindicated because of the risk of severe hypotension. Caution should also be exercised in patients with chronic disease including ischaemic heart disease, principally because the unaccustomed stress of sexual activity may precipitate cardiac ischaemia or dysrhythmia. Other treatments for impotence include self-administered intracavernosal injection or urethral gel administration of prostaglandin E1; vacuum devices which achieve an erection which is maintained by a tourniquet around the base of the penis; and prosthetic implants, either of a fixed rod or of an inflatable reservoir. Many patients elect not to use these methods, but unfortunately even more are unaware of their 56 availability. Around 10% of couples have difficulty in conceiving children. This is attributable in roughly equal thirds to infertility in the female, infertility in the male, and idiopathic cases. So, although it is common for women to present with this problem, early assessment of both partners is essential to avoid unnecessary investigations and delay. This should include establishing that the couple are having intercourse when the woman is likely to be fertile. Further assessment of women includes a menstrual history. Oligomenorrhoea suggests that the cycles are anovulatory. This can be confirmed by measurement of serum progesterone 21 days after the start of the last menstrual period (ovulation indicated by level >15 nmol/l). Subsequent tests are similar to those for secondary amenorrhoea above. If the woman has regular menses, and no abnormality is found in the man, then further gynaecological investigation may be required. The male should be examined for a varicocele or other testicular abnormality. A semen analysis should be performed. If he has oligospermia, then blood should be taken for prolactin, testosterone, FSH and LH and these interpreted as described for male hypogonadism above. If the only biochemical abnormality is a high FSH, then an irreversible failure of spermatogenesis is likely (the FSH rises because of lack of β-inhibin). Testicular biopsy is rarely indicated. In patients with gonadotrophin deficiency, fertility can be induced over several months. This is usually performed once, and sperm stored for subsequent artificial insemination. Basic Level of knowledge and skills. Students must know: 1. Action mechanisms of sex hormones. 57 2. Normal male and female reproductive physiology, and the normal menstrual cycle. 3. Etiologic classification of primary and secondary hypogonadism. 4. Clinical features of sexual differentiation disorders. 5. Criteria for the diagnosis of sexual differentiation disorders, menopause. 6. Main principles of therapy sexual dysfunction. Students should be able to: 1. Examine the patient with hypogonadism or menopause. 2. Emphasize clinical sings of hypogonadism or menopause. 3. Evaluate dates of clinical methods of investigations. 4. Make provisional diagnosis. 5. Make the plan of examination of a patient. 6. Estimate dates of additional methods of investigations. 7. Make the plan of hypogonadism and menopause management. Organization structure of lesson: I. Overview of problem of hypogonadism and menopause - 10 min. II. Emphasize basic level of knowledge – 10 min. III. Practical work – 40 min. Method of practical work 1. Work with patient in the wards of endocrinology department. 2. Examination of patients with of hypogonadism and menopause: do history taking, palpation, percussion, auscultation. 58 3. Work out symptoms in syndromes, emphasize on the main syndrome. 4. Making diagnosis plan of investigations. 5. Making and substantiation of the basic clinical diagnosis. 6. Making the plan of of hypogonadism and menopause treatment. Practical skills: students should acquire and use practical skills according to the matriculs of practical skills of the subject. ІV. Control of students’ knowledge and skills. 1. Feed back of the practical work. – 10 min. 2. Feed back of theoretical questions. – 10 min. V. Conclusion. Different, items. – 10 min. Illustrations. Work with CDs, floopy discs, audi- and video in the computers’ class-room. Methodological instructions to practical class 15 Theme: Principles of endocrine system organization. Diabetes mellitus definition. Clinical features and diagnosis of diabetes mellitus. Hours: 2 h. Working place: classroom, endocrine department wards of the Sumy Region Hospital. Aim: - to learn hormone secretion, action, maintenance of homeostasis; - to learn principles of hormonal feedback regulatory systems; 59 - to learn paracrine and autocrine control; - to learn causes hormone excess and deficiency, hormone resistance; - to learn etiology, pathogenesis of the diabetes mellitus (DM); - to learn signs, symptoms and diagnostic criteria of DM. Professional Motivation. An understanding of principles of endocrine system organization is important to diagnosis and treatment. Endocrinology concerns the synthesis, secretion and action of hormones. These are chemical messengers which have diverse molecular structures, are released from endocrine glands and coordinate the activities of many different cells. Endocrine disease therefore has a wide range of manifestation affecting many other major organs. Most endocrine glands are controlled by hormones released from the pituitary gland. These integrated endocrine systems are called “axes”, but some major endocrine glands are not controlled by the pituitary. The classical model of endocrine function involves hormones which are synthesised in endocrine glands, released into the circulation, and act at sites distant from those secretions. However, additional levels of complex regulation have now been recognised. A wide variety of molecules act as hormones. The clinical implications of this complexity of hormone action are only now being appreciated. The worldwide prevalence of DM has risen dramatically over the past two decades. Likewise, prevalence rates of impaired fasting glucose (IFG) are also increasing. Although the prevalence of both type 1 and type 2 DM is increasing worldwide, the prevalence of type 2 DM is expected to rise more 60 rapidly in the future because of increasing obesity and reduced activity levels. DM increases with aging. In 2000, the prevalence of DM was estimated to be 0.19% in people <20 years old and 8.6% in people >20 years old . In individuals >65 years the prevalence of DM was 20.1%. The prevalence is similar in men and women throughout most age ranges but is slightly greater in men >60 years. There is considerable geographic variation in the incidence of both type 1 and type 2 DM. Scandinavia has the highest incidence of type 1 DM (e.g., in Finland, the incidence is 35/100,000 per year). The Pacific Rim has a much lower rate (in Japan and China, the incidence is 1 to 3/100,000 per year) of type 1 DM; Northern Europe and the Uni ted States share an intermediate rate (8 to 17/100,000 per year). Much of the increased risk of type 1 DM is believed to reflect the frequency of high -risk HLA alleles among ethnic groups in different geographic locations. The prevalence of type 2 DM and i ts harbinger, impaired glucose tolerance (IGT), is highest in certain Pacific islands, intermediate in countries such as India and the United States, and relatively low in Russia and China. This variability is likely due to genetic, behavioral, and environ mental factors. DM prevalence also varies among different ethnic populations within a given country. Gestational diabetes mellitus (GDM) occurs in approximately 4% of pregnancies; most women revert to normal glucose tolerance post -partum but have a substantial risk (30 to 60%) of developing DM later in life. 61 Basic Level of knowledge and skills. Students must know: 1. Hormone secretion and action mechanisms 2. Main principles of hormonal regulatory systems 3. Insulin biosynthesis, secretion, and action. Regulation of glucose blood level. 4. Epidemiology. Screening type 1, 2 DM. 5. Criteria for the diagnosis of DM. Diabetes and abnormalities in glucose-stimulated insulin secretion. 6. Etiologic classification of DM. 7. Etiopathogenesis type 1 DM. autoimmune factors, environmental factors, immunological markers. 8. Etiopathogenesis type 2 DM. Metabolic changes during the development of type 2 diabetes mellitus. Insulin resistance. Impaired insulin secretion. Increased hepatic glucose production. 9. Genetically defined, monogenetic forms of DM. Students should be able to: 1. Examine the patient with DM. 2. Call syndromes for DM. 3. Evaluate dates of clinical methods of investigations for diagnosis of DM. 4. Make provisional diagnosis of DM. 5. Make the plan of examination of patient with DM Organization structure of lesson: I. Overview of topical problem of DM. II. Estimate basic level of knowledge – III. Practical work – Method of practical work 62 10 min. 10 min. 40 min. Work with patients in the wards of endocrinology department. Examination of patients with DM: do history taking, palpation, percussion, auscultation. Working out symptoms in syndromes, emphasize on the main syndrome. Making the provisional diagnosis. Making diagnosis plan of investigations. Practical skills: students should acquire and use practical skills according to the matriculs of practical skills of the subject. ІV. Control of students’ knowledge and skills. 1. Feed back of the practical work. – 10 min. 2. Feed back of theoretical questions. – 10 min. V. Conclusion. Different, items. – 10 min. Illustrations. Work with CDs, floopy discs, audi- and video in the computers’ class-room. References for unit 5. 1. Endocrinology (A Logical Approach for Clinicians (Second Edition)). William Jubiz.-New York: WC Graw-Hill Book, 1985. - P. 38 – 42, 144 –164, 198 – 205. 2. Short Textbook of Medical Diagnosis and Management (Third Edition). Mohammad Inam Danish. – Pakistan, 2002. – P.459 – 462, 504 – 505. 3. Manual of Endocrinology and Metabolism (Second Edition)/ Norman Lavin. – Little, Brown and Company.- Boston-New York-Toronto-London, 1994. - P. 111 – 142, 173 - 180. 63 4. Endocrinology (A Logical Approach for Clinicians (Second Edition)). William Jubiz.-New York: WC Graw-Hill Book, 1985. - P. 38 – 42, 144 –164, 198 – 205. 5. Short Textbook of Medical Diagnosis and Management (Third Edition). Mohammad Inam Danish. – Pakistan, 2002. – P.459 – 462, 504 – 505. 64 Methodological Instruction to Module. Theme: Control of the students’ knowledges and skills on DM. Hours: 2 h. Working place: classroom. Aim: to control students’ knowledges, confirm skills on diabetes mellitus, DM long-term and acute complication. PROGRAME FOR THE ISSUE OF DIFFERENTIATED CREDITS ON ENDOCRINOLOGY FOR 4th COURSE STUDENTS FIRST CATEGORY 1. Acromegaly. Etiology, pathogenesis, clinical manifestations, diagnosis and treatment. 2. Cushing’s Disease. Etiology, pathogenesis, clinical manifestations, diagnosis and treatment. 3. Diabetes insipidus. Etiology, pathogenesis, clinical manifestations, diagnosis and treatment. 4. Hypopituitarism in children. Differential diagnosis of growth retardation in children. 5. Treatment of endocrine -induced growth disorders in children. 6. Iodine deficiency conditions and diseases caused by lack of iodine. 7. Diffuse toxic goiter. Etiology, pathogenesis, clinical manifestations, diagnosis and treatment. 8. Hypothyroidism. Etiology, pathogenesis, classification, clinical picture, diagnosis and treatment. 9. Peculiarities of hypothyroidism, congenital hypothyroidism, 65 hypothyroidism in the elderly, clinical course of atypical forms of hypothyroidism. 10. Acute thyroiditis. Etiology, clinical manifestations, differential diagnosis, treatment. 11. Chronic thyroiditis. The etiology, classification, diagnosis, treatment. 12. Cancer of the thyroid gland. Classification. The role of the Chernobyl accident in the occurrence of thyroid cancer. 13. Hyperparathyroidism. Etiology, pathogenesis, clinical manifestations, diagnosis and treatment. 14. Hypoparathyroidism. Etiology, clinical manifestations, diagnosis and treatment. 15. Diabetes. Determining the prevalence of the disease. Diagnosis of diabetes. 16. The etiology of diabetes. Classification of glucose disorders. 17. The clinical course of diabetes. Differences clinical course depending on type of diabetes. 18. Classification of chronic complications of diabetes. Prevention of chronic complications of diabetes. 19. Glycated Hemoglobin. Diagnostic value of glycated hemoglobin. 20. Indications for glucose tolerance test . Evaluation of the test. 21. Diet therapy of diabetes. The glycemic index of foods. The value of food glycemic in the development of chronic complications of diabetes. 22. Hypoglycemic sulfonamides. Classification of drugs. The mechanism of hypoglycemic sulfonamides impact . 23. Prescribe treatment with hypoglycemic sulfonamides. Side effect of medications. Features of hypoglycemic conditions caused by sulfonamides. 66 24. Biguanide. Prescribe for the treatment of diabetes. Indications and contraindications for use, adverse reactions of Biguanide. 25. Postprandial stimulators of insulin secretion. Classification. Methods of use. 26. Thiazolidinedione’s in the treatment of diabetes. Indications, adverse reactions to drugs, the control treatment. 27. Indications for insulin therapy in patients with diabetes. Prescribe insulin therapy for patient newly diagnosed of diabetes. Mode of insulin therapy. Performance indicators insulin treatment. 28. Complications of insulin therapy. Hypoglycemic states. 29.Causes and urgent measures for hypoglycemia. Preventive measures for hypoglycemia. 30. Causes of decompensation of diabetes. Diabetic ketonuria, ketoacidosis. Diagnosis and treatment. 31. Diabetic coma. Causes. Diagnosis and treatment. 32. Chronic adrenal insufficiency. Etiology, pathogenesis, clinical manifestations, diagnosis and treatment. 33. Cushing’s Syndrome. Etiology, pathogenesis , clinical manifestations, diagnosis and treatment. 34. Pheochromocytoma. Etiology, clinical features, diagnosis and treatment. 35. Congenital adrenal hyperplasia. Clinical manifestations, diagnosis, treatment. 36. Cryptorchidism, etiology, pathogenesis, diagnosis and treatment. 37. Syndrome Turner. Clinical manifestations, diagnosis and treatment. 38. Klinefelter syndrome. Clinical manifestations, diagnosis and treatment. 67 39. Disorders of sexual development. Premature sexual development, delayed sexual development. Clinical manifestations, diagnosis and treatment. 40. Obesity. Definition , etiology , classification of obesity by body mass index. Treatment. 41. Obesity in children. Hypothalamic -pituitary obesity. Etiology . Clinical manifestations . Differential diagnosis . Treatment. SECOND CATEGORY 1. Different types of hormones and their place of establishment in the body. 2. Classification hormone chemical structure. 3. Basic functions of hormones, their genomic and epigenomic effects. 4. The synthesis of hormones and their secretion, transport in the blood and metabolism. 5. Mechanisms of action of hormones. 6. Regulation of endocrine function. 7. Basic mechanisms of endocrine diseases. 7. Anatomical and physiological data on the pituitary and hypothalamus. 8. Classification of hypothalamic- pituitary disease. 9. Hyperprolactinemia syndrome. Etiology, pathogenesis, clinical manifestations, diagnosis and treatment. 10. Hypopituitarism . Etiology, pathogenesis, clinical manifestations, diagnosis and treatment. 11. Anatomical and physiological data on the thyroid gland. 12. The concept of goiter. The reasons for the increase in thyroid size. 13. Laboratory and instrumental methods of examination of 68 thyroid gland. Indications , contraindications for use, the diagnostic value of each method . 14. The methods of iodine prophylaxis . 15. The impact of man-made pollution of the environment on the development of thyroid cancer. 16. Nodules in the thyroid gland. Etiology. Clinic differential diagnosis . 17. Indications and contraindications for surgical treatment of thyroid cancer. 18. Differential diagnosis of hyperthyroidism syndrome. 19. Thyrotoxicosis crisis . Causes , clinical manifestations, diagnosis and treatment. 20. The definition and classification of thyroiditis . 21. Autoimmune thyroiditis. Classification , clinical course, diffusion. diagnosis and treatment. 22. The active detection of thyroid cancer , treatment, replacment therapy. 23. Anatomical and physiological characteristics of the parathyroid glands and their role in the regulation of calcium homeostasis. 24. Metabolic osteopathy in endocrine diseases. The etiology and pathogenesis , differential diagnosis , treatment and prevention. 25. Anatomical and physiological data on endocrine part of the pancreas. 26. The use of sweeteners in the diet of diabetics. Classification of sweeteners . The use of dietary fiber in the diets of patients with diabetes. 27. The physical load in patients with diabetes. Purpose graduated exercise based on blood glucose. 28. Hlyukomodulyatoriv use in the treatment of diabetes mellitus. 69 29. Use of herbal products in the treatment of diabetes mellitus. 30. Classification of oral hypoglycemic drugs. 31. Diabetes and pregnancy. Contraindications to pregnancy in patients with diabetes. Terms of diabetes in pregnancy. 32. Classification of insulin drugs, duration of action. 33. Coma in patients with diabetes. Hyperosmolar ( nonacidotic ) coma. Lactic acidosis and coma. Causes. Diagnosis. Urgent measures. Prevention. 34. Diabetic angiopathy of lower extremities. Stages of development, diagnosis, preventive measures for angiopathy. 35. Diabetic polyneuropathy. Manifestations of autonomic diabetic neuropathy. 36. Diabetic foot syndrome . Diffusion. diagnosis of neuropathic and ischemic foot lesions form. 37. Labour and medical- social examination of patients with diabetes. 38. The anatomic features of the adrenal glands, the adrenal hormones each layer . 39. The physiological action of adrenal hormones . 40. Acute adrenal insufficiency. Etiology, clinical manifestations, diagnosis and treatment. 41. Addison crises. Causes, clinical manifestations, diagnosis, emergency care. 42. Classification hormonal active tumors of the adrenal cortical layer. 43. The morphological structure of the endocrine testis and ovary. Physiological effects of sex hormones. 44. Classification of disorders of puberty. 45. Menopause. Pathological course of menopause. Clinical manifestations in women and men. Treatment of pathological manifestations of menopause. 70 46. The impact of obesity on the occurrence of lesions of the organs and systems of the human body. 47. Differential diagnosis of clinical forms of obesity. 48. The metabolic syndrome. Diagnosis. Treatment. 49. Autoimmune Polyglandular Syndrome. Definition, clinical manifestations, diagnosis and treatment. 50. Multiple endocrine neoplasia syndrome, definitions, diagnosis, treatment. THIRD CATEGORY 1. Physiology of the pineal gland and its functional role in the human. 2. Classification and clinical manifestations of the pineal gland pathology. 3. Anatomy and physiological role of the thymus. 4. Immunological failure is caused by thymic hypoplasia. 5. Myasthenia gravis. The clinical manifistation. Features of myasthenia gravis in children. Diagnosis and treatment of myasthenia gravis. 6. Tumors of the thymus. Clinical course . Diagnosis . Treatment. 7. Principles of the thyroid surgery. 8. Complications after thyroid surgery. 9. Indications for surgical treatment of nodular forms of goiter . 10. Surgical treatment of adrenal tumors, preoperative preparation, postoperative period, the rehabilitation of patients after adrenalectomy. 71 LIST of practical skills that should have STUDENT 1. Assess glucose tolerance test. 2. Assess glycemic and glycosuria profile. 3. Determine the type of diabetes, its clinical course and condition of compensation. 4. Determine blood glucose and acetone in the urine method. Determine blood glucose rapid method. 5. Prescribe dietary treatment of diabetes. 6. Prescribing of oral hypoglycemic agents. 7. Identify secondary sulfanilamide resistance and, to be able to treat it. 8. Rehabilitation of endocrine patients. 9. Assign mode of insulin therapy in diabetes. 10. Assign treatment of patients with ketoacidosis. 11. Assign scheme of treatment for diabetic coma. 12. Able to use syringe pen. 13. Develop a plan for self-management of diabetic patients. 14. Determine the degree of goiter. 15. Estimate data and Doppler ultrasonography of the thyroid gland. 16. Evaluate the results of radioisotope examination and thermography thyroid gland. 17. Assess the state of thyroid system according to radioimmunoassay and enzyme immunoassay tests. 18. Assign treatment of patients with toxic goiter. 19. Evaluate the results of ECG and reflex meter to characterize the function of the thyroid gland. 20. Diagnose hypothyroidism. 21. Assess the condition of the adrenal glands of the clinical data, the results of hormonal tests, ultrasound, arteriography, 72 CT , MRI . 22. Prescribe treatment of Addison crisis. 23. Evaluate X-ray and data CT scan, MRI of the skull. 24. Determine the morphotype. 25. Determine the degree of obesity and BMI. 26. Determine the degree somatosexual development. 27. Define “bone age" according to radiography. 28. Estimate data chromosomal analysis. 29. Identify types of disorders of sexual differentiation. 30. Diagnose menopause treatment regimen and schedule pathological menopause. 31. Determine the extent of disability of patients with endocrine disorders. Tests and Assignments for Self-assessment. UNIT 1 A. 1. Patient L., 29 y-r-old, takes prednisolon in treatment of SLE during last 4 years. She is complains of weight gain, polyuria, thirst. Plasma glucose concentration is 12,4 mmol/l. Put diagnosis, please. A. Diabetes mellitus type 1 B. Diabetes mellitus type 2 *C. Steroid diabetes D. Impaired glucose tolerance E. Impaired fasting glycemia 2. Diabetic K., 31 y-r-old, is pregnant. What laboratory data we have to control in pregnant diabetes, EXCEPT fast glycemia? The level of postprandial glucemia. 73 B. C. D. E. B. C. D. E. F. A. B. C. D. E. A. B. C. D. E. The level of glucosurea. *The level of glicolize Hb The level of the insulin in the blood. The level of cholesterol. 3. Patient B., 40 years, has the type 1 of DM for 19 years. Choose the degree of severity of the diabetes mellitus in a case of the level of fasting glucose – 6,8 mmol/l, aglucosurea, the level of creatinine 0,36 mmol/l. Mild. Moderate. *Severe Stabile. Progressive. 4. 30. In patient R., 19 years old, was found first time glucosurea 5 g/l, the level of the glucemia is 5,5 mmol/l. What investigation can exclude diabetes mellitus? Daily fluctuation of glucemia level. *Glucose tolerance test. The level of the insulin in the blood. The level of postprandial glucemia. Daily glucosurea 5. 30. In patient R., 19 years old, was found first time glucosurea 5 g/l, the level of the glucemia is 5,5 mmol/l. What investigation can exclude diabetes mellitus? Daily fluctuation of glucemia level. *Glucose tolerance test. The level of the insulin in the blood. The level of postprandial glucemia. Daily glucosurea 6. A 28-year-old man is diagnosed with diabetes mellitus. Which of the following findings would point most 74 A. B. C. D. E. A. B. C. D. E. convincingly to type 1 rather than type 2 diabetes? He is not obese. Severe polydipsia and polyuria preceded the diagnosis, *Ketoacidosis was present at the time of diagnosis. The serum C-peptide level was normal at the time of diagnosis. Glucose levels rose above 32 mmol/l (600 mg/dL) before treatment was started. 7. A 31-year-old female is diagnosed with diabetes mellitus. Type 1 diabetes mellitus is usually associated with which one of the following? Onset in middle age Obesity Insulin resistance *Requirement for insulin therapy Absence of autoimmune manifestations 8. A 37-year-old male is diagnosed with diabetes mellitus Very high serum level of anti-islet cell antibodies is most closely associated with: *Type 1 diabetes mellitus Type 2 diabetes mellitus Impaired fasting glucose Impaired glucose tolerance Diabetic ketoacidosis 9. A 47 y-r-old overweight man complains of paradontosis. The most sensitive test for diabetes mellitus: Fasting serum glucose level Random serum glucose level *Oral glucose tolerance test Serum level of hemoglobin A1C Urine glucose concentration 10. 62. A 14-yr-old girl presents with 3 weeks of weight loss, polyuria and polydipsia. 75 A. B. C. D. E. Prescribe investigations, please. Fasting blood glucose* Urinary ketones T3, T4 and TSH levels HbAlc levels C-peptide levels 11. Interpret GTT. Fasting glucose level is 7.1 mmol/1; 2 hours after glucose ingestion 14.7 mmol/1 in plasma. A. Diabetes mellitus B. Impairment of carbohydrate tolerance C. Normal D. Necessary to repeat test E. Additional laboratory investigations are indicated A. B. C. D. 12. Woman 62 years, which is ill by Diabetes Mellitus type 2, complains about a trophic ulcer on the leg. The origin of ulcer binds by carrying of uncomfortable shoe. At a review the feet are dry and brilliant; the pulsation on the back arteries of foot is satisfactory. The ulcer on the right foot is unsickly. What is your previous diagnosis? Diabetes mellitus type 2, diabetic angiopathy, foot ulcers Diabetes mellitus type 2, diabetic foot, neuro-ischemic form Diabetes mellitus type 2, diabetic foot, ischemic form Diabetes mellitus type 2, diabetic neuropathy 13. Woman 64 years, which is ill by Diabetes Mellitus type 2, treats by Maninil in a dose 5 mg in the morning. The fasting level of glucose is 6.5 mmol/L. Advised by the oculist. The conclusion is “diabetic retinopathy”. What is the subsequent method of treatment? 76 A. Maninil in a dose 5 mg B. insulin of short action C. insulin of the prolonged action D. insulin of short and prolonged action E. combined therapy 14.Patient N., 56 years old. Duration of Diabetes Mellitus type 2-10 years. Regularly receives glibenclamide 0,005 1 tablet 3 times per day. In the last three years, patient was not examined by endocrinologist. Foot was chilly, and pulse was faint. Fasting blood glucose- 15,0 mmol/1. Glucose in urine - 2,0 % Substantiate diagnosis. A. Diabetes mellitus type 2, inadequate control B. Diabetes insipidus C. Diabetes mellitus type 2, adequate control D. Diabetes mellitus type 1, adequate control E. Diabetes mellitus type 2, inadequate control, ketoacidosis 15. Interpret GTT. Glycemia: I trial - 5.4 mmol/1, II trial - 7.8 mmol/1, III trial - 5.5 mmol/1 A. Normal B. Impairment of carbohydrate tolerance C. Diabetes mellitus D. Necessary to order additional laboratory tests. E. Additional laboratory investigations are indicate 16. Patient 56 years old is ill by Diabetes Mellitus type 2, complains about dizziness, darkening in the eyes at the getting up. These phenomena accompany perspiration on the face, neck, thorax. The reception of the meal does not give the 77 a. b. c. d. e. facilitation. He ills by diabetes for 42 years. The last year he adopted Mannil in a dose 1 or 2 pills per day. Compensation of the disease is satisfactory. The indicated complaints coincided with resistance tachycardia. AP – 140/95 mm.hg., standing – 90/60 mm.hg. Your previous diagnosis? Diabetes Mellitus type 1. Diabetic vegetative neuropathy of a heart Diabetes Mellitus type 1, climacteric syndrome. Dismetabolic cardiopathy Diabetes Mellitus type 1. Diabetic miocardiopathy Diabetes Mellitus type 1,atherosclerotic cardiosclerosis Non of this variant. 17. Patient with persistent dermatomyositis examined glucose and appearance such results: the fasting level – 5,55 mmol/L and 6,68 mmol/L, during the day 7,85;9,11;11,13 mmol/L. Your conclusion. A. Diabetes mellitus B. Impairment of carbohydrate tolerance C. Normal D. Necessary to order additional laboratory tests. E. Needed an add examination 18. Which of the statement given below is correct relatively to GTT? A. Correlates with determination of potential abnormalities of glucose tolerance B. Helpful to choose the most appropriate treatment C. Used to differentiate type of diabetes D. Indicate stage of diabetes E. Useful in the seeking of early diabetic complications 78 19. A patient K., 66 years old, which complains on the headache, eyestrain, loss of appetite. He is ill during 11 years on the diabetes mellitus 1. During the examination: pale, swell face, heart increase in the left sight on 1,5 sm. Pulse – 70/min, rhythmical, pressure 180/100 mm.hg.. Stomach is soft, liver doesn’t increased. Pasternackyj symptom’s is negative with both sight. Urination – accelerating, mainly in the night. Analysis of the blood: Hb 98 g/l, er. 3* 1012 /L.lk. 8,2 *109 /l, urea 15,8 mM/l. Cholesterin 7,6 mM/l, albumen – 81 g/l, creatynine 177 mcM/l, glucose 8,2 mM/l. Analysis of urin : specific gravity 1010, albumen 0,6 g/l, er. 1-3, l. 10-15, glucose 2 g/l, ECG: hypertrophy of the left ventricle. Your preliminary diagnosis? A. Diabetes mellitus 2, hard form in the stage of decompensation. Diabetic nephropathy. B. Diabetes mellitus 2, high pressure disease, stage 3. C. Chronic glomerulonephritis. D. Diabetes mellitus 2, hard form in the stage of compensation. Chronic pyelonephritis, high pressure disease. E. Non of this variant. 20. A patient W., which is ill for 15 years by Diabetes Mellitus type 2, during last year determine weakness, hypertonia, swelling of lower limbs and face. In the general analis of urine: albumen – 0,99 g/L, glucose 11 mmol/L, leukocyte 3 – 5, creatunine, urea are norma. The fasting level of glucose – 11,5 mmol/L. Established the most suitable diagnose? A. Diabetes Mellitus type 2, adequate control, diabetic nephropathy, persistent proteinuria 79 B. Diabetes Mellitus type 2, inadequate control, diabetic nephropathy, persistent proteinuria C. Diabetes Mellitus type 2, adequate control, diabetic nephropathy, renal impairment D. Diabetes Mellitus type 2, inadequate control, diabetic nephropathy, microalbuminuria E. Non of this variant 21. In case of what disease is the number of insulin receptors reduced: a. acromegaly b. insulinoma c. diabetes mellitus, type 2 d. adiposity e. thyrotoxicosis 22. In the pathogenesis of the insulinoresistance the following factors play part: a. hyperlipidemia b. Antibodies to insulin c. hormone of growth d. cortisol e. antibodies to receptors of insulin 23. Mark what characterizes the diabetes mellitus of middle degree of gravity: a. the necessity to be treated with insulin b. the compensation of diabetes mellitus only with a diet c. the necessity of taking tabletlike sugar lowering preparations d. absence of angiopathy e. presence of stable complications of diabetes mellitus 80 24. A light form of the diabetes mellitus is characterized by: a. absence of ketoacidosis b. the necessity of injecting nearly 10 units of insulin c. presence of hepatopathy d. the necessity of taking of tabletlike sugar reducing preparations e. the compensation of the diabetes mellitus is achieved only by a diet 25. Mark the characteristic symptoms of the first type of diabetes mellitus: a. young age b. rapid beginning of the disease c. inclination to ketoacidosis d. growing thin on the background of the increased appetite e. all mentioned above 26. The patient is 54 years old, the diabetes mellitus was suspected and a test on the tolerance to glucose was prescribed: on an empty stomach – 7,0 mmol/litre, in 2 hours – 11,5 mmol/litre. Evaluate the results of tolerance test to glucose. a. breaking tolerance to glucose b. obvious diabetes mellitus c. normal test d. breaking glycemia on the empty stomach e. repeated examination is necessary 27. A woman, 46 years old, has been suffering from diabetes mellitus for 6 years. She is being treated with peroral sugar reducing preparations. At present, being examined, does 81 not put forward any complaints. Which methods can be used to evaluate the compensation of the diabetes mellitus? a. determining the glycemic profile b. determining the level of glucose in blood on the empty stomach c. making a tolerance test to hydrocarbonas d. Determining of the glycosylated haemoglobin e. determining glucose level in the blood during 24 hours 28. The patient of 32 years old, with the diabetes mellitus revealed for the first time, is keeping up glycemia on the empty stomach less than 6 mmol/litre, after the meal less than 9 mmol/litre by means of the diet. The patient refrains from insulin therapy. What investigation must be carried out for receiving differential diagnostics being the diabetes mellitus type1 or type 2? a. assigning glycosylated haemoglobin b. assigning antibodies to the cells of the pancreas insular c. test of glucose tolerance d. assigning the level of glucagon e. assigning glycemia on the empty stomach 29. The patient, 48 years old has been treated from the hypertension. Being hospitalized complained of dryness in the mouth, polydipsia, increased appetite, polyuria. His father suffered from the diabetes mellitus. In his blood the level of glucose – 14,6 mmol/litre was detected. Your preliminary diagnosis is: a. breaking glucose tolerance b. nonmellitus diabetes c. diabetes mellitus, type 1 82 d. renal glucosuria e. diabetes mellitus type 2 30. A 36 years old woman complains of dryness in the mouth, thirst, polyuria losing 5 kg of weight during half a year. Has been ill for 7 months. Her skin and mucus membranes are dry. Glucosuria is 20 gr/litre, glycemia on the empty stomach is up to 10 mmol/litre. Urine reaction to acetone is negative. Which of the investigations will be most reasonable to define the type of the diabetes mellitus? a. determining С-peptide level b. glycemic profile c. determining glycosylated haemoglobin d. determining HLA-antigens e. glucose tolerance test 31. Patient, 45 years, complaints about dry mouth, thirst, polyuria. Estimate the results of glucose tolerance test: fasting - 7,2mmol/l, after 1 hour - 13,2mmol/l, after 2 hrs 11,5mmol/l. A. Evident diabetes mellitus B. Impaired glucose tolerance C. Impaired fasting glycemia D. It's need repeated examination E. It's need to conduct a glucose-prednizolone test. The right answer: A. 32.A patient, 56 years, was hospitalized in endocrinol ogy department with complaints about dry mouth, thirst, polyuria. Estimate the results of glucose tolerance test: fasting 5,8mmol/l, after 1 hour - 9,6mmol/l, after 2hrs - 7,7 mmol/l. A. Impaired fasting glycemia 83 B. Impaired glucose tolerance C. Evident diabetes mellitus D. It's need repeated examination. E. It's need to conduct a glucose-prednizolone test The right answer: A. 33. A patient, 38 years, came to ambulatory with complaints about the weight loss, general weakness, dry mouth, polyuria. Estimate the results of glucose tolerance test: fasting - 5,8 mmol/l, after 1 hour - 9,8mmol/l, after 2 hrs 8,3mmol/l. A. Impaired glucose tolerance B. Impaired fasting glycemia C. Obvious saccharine diabetes D. Necessary repeated examination E. It is necessary to conduct a glucose-prednisolone test The right answer: A. 34. Man, 45 years., has diabetes mellitus type 2. He intakes amaryl 3mg a day. Which methods of treatment efficacy control are the most adequately represent the state of compensation in ambulatory terms? A. Assessment of glycated haemoglobin B. Definition of glycemic profile C. Definition of fasting glucose D. Examination of glucose tolerance test E. Definition of blood lipoproteins The right answer: A. 35. Youth, 18 years, is ill with diabetes mellitus of moderate severity during 2 years and he uses insulinotherapy. He appealed to endocrinologist with the purpose of advice 84 about the choice of future profession. What from these professions aren't contra-indicated for this patient? A. Teacher at school B. Work at badtime, business trips C. Height works D. Machinist E. Pilot, driver The right answer: A. 36. A man, 26 years, complains about thirst, polyuria, general weakness, weight loss. Objectively: a skin is dry, red cheeks, breathing is vesicular. Cardiac tones are sounding. Tongue is dry. The symptoms of irritation of peritoneum are not present. What examination is the most informing for establishment of diagnosis? A. Glucose blood test B. Common blood count C. Common analysis of urine D. Analysis of urine by Zemnitsky E. Blood test on hepatic probesВірні відповіді : A The right answer: A. 37. Patient, 36 years, appealed to district internist with complaints about general weakness, weight loss to 5 kg for the last 3 months, strong thirst. Laboratory data: fasting glucose 9,2mmol/l, glucosuria 1,5%, ketonuria is absent. What examination is need for determination of type of diabetes? A. Definition of С-peptide B. Common blood count C. Definition of glycated haemoglobin D. Level of postprandial glucose E. Blood test on hepatic probes 85 The right answer: A. 38. The most typical sign of diabetes mellitus type 1 is: A. Presence of antibodies to pancreatic islet tissue B. Association with HLA-antigens C. Decreasing of amount of receptors to insulin D. Increase of insulinoresistance E. Increase of plasma osmolarity The right answer : A 39. What from clinical symptoms are not typical for hypoglycemia? A. Diarrhea B. Hyperhydrosis C. Sense of the hunger D. Dizziness E. Palpitation The right answer : A 40. What basic intermediate substance which appears in the process of biosynthesis of insulin will represent insulinproduced function of pancreas in patient who intakes insulin? A. С-peptide B. Proinsulin C. Insulin like growth factor - 1 D. Glucagon E. Preproinsulin The right answer: A 41. A 59 years old man, with height 175 cm, weight 74 kg, (and his father has been suffering from diabetes mellitus for 86 last 24 years), is tested against the glucose tolerant test: glycemia on an empty stomach - 4,8 mmol/l, in 2 hours after glucose load (pp) (75 g) - 6,8 mmol/l. Choose a glucose tolerant test interpretation to the results from below: A. *NORMAL, B. Impaired tolerance to glucose, C. Type 1 Diabetes mellitus, D. Type 2 Diabetes mellitus, E. The glucose tolerant test indications are absent. 42. A woman being 24 complains to raised thirst, frequent urination and skin itch. She went to a local polyclinic. They did glucose tolerant test to her: glucose level on an empty stomach - 8,7 mmol/l, in 2 hours after glucose load (pp) (75 g) - 14,3 mmol/l. Choose a glucose tolerant test interpretation to the results from below: A. *Type 1diabetes mellitus, B. Type 2 diabetes mellitus. C. Impaired tolerance to glucose D. Normal. E. Allergetic dermatitis. 43. A 48 years old man, with height 179 cm, weight 94 kg, having done the glucose tolerant test on: glucose level on an empty stomach - 5,9 mmol/l, in 2 hours after glucose load (pp) (75 g) - 8,4 mmol/l. Choose a glucose tolerant test interpretation to the results from below: A. *Glycosylated hemoglobin, B. Level of total cholesterol, C. Level of triglyceride, D. Measurement of blood pressure, E. All listed above. 87 44. Patient, 30 years, suffers on diabetes mellitus Type1 during 13 years. He noticed the appearance of edema of the face, lower extremities, increase of arterial pressure, up to 180/110 mm. Sugar of blood on an empty stomach - 11,6 mmol/l. Doctor-endocrinologist considered these complaints as exposures of nephropathy. What researches do need to be conducted for the exposure of the first signs of worsening of function of buds? A.* Presence of microalbuminuriya; B. Ultrasound kidney; C. The level of glycosylated of HbAlc; D. Determining the level of blood urea; E. Determining the level of creatinine. 45. Patient, 56 years, an engineer, is ill diabetes mellitus Type 2 during 3 years, began to notice deterioration of visual acuity of variable character. Sugar of blood during conducting of test of tolerance to glucose of 9,9 mmol/l, 10,1 mmol/l, 8,7 mmol/m. To set the degree of weight of diabetes and complication: A.* Diabetes mellitus is type 2, moderate , phase compensation. Diabetic cataract I; B. Diabetes mellitus is type 2, moderate, phase of sub compensation. Diabetic retinopathy II; C. Diabetes mellitus, moderate, phase of compensation. Diabetic cataract; D. Diabetes mellitus type 2, moderate, phase of sub compensation; E. Diabetes mellitus type 2, easy motion, phase of sub compensation. Diabetic retinopathy . 88 46. Patient., 27 years, contacted doctor with the complaints about the increased thirst, increase of amount of urine, increased appetite, gradual emaciation for several months. During an inspection level of glucose on an empty stomach – 8,8 mmol/l. Test of tolerance to glucose – in 2 hours glycemia of 12,6 mmol/l. The most likely diagnosis? A. * Diabetes mellitus type 1; B. Diabetes mellitus type 2; C. Non- Diabetes mellitus; D. Violation of tolerance to glucose; E. Broken glycemia on an empty stomach. 47. What inspection does need to be conducted for the exposure of latent diabetes mellitus? A.* Glucose tolerance test; B. Daily glucosuria ; C. Determination of glycemic profile; D. Determination of glycosylated hemoglobin; E. Determination of c-peptide. 48. The ill M., 12 years old, after the carried windy pox in a heavy form there was the promoted thirst (drinks to 5 liters in a day), poliuriya , dry skin, pruritus. The grandfather of patient is ill diabetes mellitus. Level of glycemia – 15 mmol/l, acetone of urine +. To define a diagnosis. A.* Diabetes mellitus is type 1, stage of decompensation, ketosis ; B. Diabetes mellitus type 1, subindenifications; C. Diabetes mellitus 2 types, easy motion; D. Diabetes mellitus type 2, to middle weight, stage of indemnification; E. Diabetes mellitus type 1, heavy form, ketoacidosis. 89 49. A patient 19 years had diabetes mellitus the type 2 «masonic type». Why is this type of diabetes «adult type» in young people? A. * Presence of mutations of gene of glucokinasa; B. Stresses; C. Alcoholic intoxication; D. Drug addiction; E. Smoking. A. B. C. D. E. 50. For patient A., 50 years, obesity type 3. Inspects on diabetes mellitus. Table of contents of glucose in blood on an empty stomach - 6,0 mmol/l. Sugar in urine is absent. In urine tracks of ketone bodies. Glycosylated HbAlc – 6%. IRI insulin –– 180 mmol/l. What inspection must be done? A.* Glucose tolerance test; B. Daily gluсosuria; C. Daily proteinuria; D. Glycemic profile; E. Level of C-peptide. 51. A 49-year-old female, insulin dependent diabetic, been on treatment for 19 years, presents with urinary frequency but no dysuria or urgency. Her blood glucose is 18.6 mmo/l. Why patient has urinary problems? Hyperglycaemia* Hypoglycaemia Urinary tract infection Somatic neuropathy Autonomic neuropathy 52. 17-year-old female complains of polyuria, polydipsia, weight loss, and blurred vision. The level of fasting serum glucose is 15,9 mmol/L. What is your diagnosis? 90 A. B. C. D. E. A. B. C. D. E. A. B. C. D. E. Health. Impaired glucose tolerance. *Diabetes mellitus type 1. Diabetes mellitus type 2. Impaired fasting glycemia. 53. Patient J., 34 years old, has diabetes mellitus during 7 years, takes insulin (53 Units). Patient sometimes has diabetic nonproliferative retinopathy. Fasting glucemia is 11,3 mmol/l. What is your previous diagnosis? *Diabetes mellitus type 1, moderate degree. Diabetes mellitus type 1, severe degree. Diabetes mellitus type 1 mild degree. Diabetes mellitus type 2, moderate degree. Diabetes mellitus type 2, severe degree. 54. A 29-year-old female in her first trimester of pregnancy complains of polyuria, polydipsia. Fasting plasma glucose 8,7 mmol/L was found. She was treated with a diet and insulin which resulted in a reduction in plasma glucose to normal. Which method is the most important to establish the stage of compensation of diabetes mellitus? The level of fast glucemia. The level of postprandial glucemia. The level of glucosurea. *The level of glicolize Hb The level of the insulin in the blood. 55. Patient M., 22 y.o., has first time diagnosed DM. Fasting glucose is 15,2 mmol/l, glucosurea – 2,5 g/l, glicolize Hb – 10 %. What treatment is necessary? А. Glitazones 91 B. Diet therapy C. Biguanides D. Sulfonylurea medicines II generation E. Insulin therapy* 56. Patient is 13 years old. Felt ill rapidly. Complaints on thirst, polyurea, general weakness, during last 2 weeks lost 2 kg of weight. Fasting blood glucose level – 32 mmol/l, in the urine – 6 %, acetone (+). Your treatment? А. Diet therapy B. Phytotherapy C. Biguanides D. Sulfonilurea medicines E. Insuline therapy* 57. Patient 24 years old is ill with DM during 7 years. Uses injections of semi-long acting insulin. Variability of blood glucose level during day is 15,2 – 22,0 mmol/l, fasting glucose – 28,6 mmol/l. What treatment should be prescribed A. Intensive insulin therapy* B. Biguanides C. To increase dose of semi-long acting insulin D. To prescribe short-acting insulin twice a day E. Sulfonilurea medicines 58. Patient R. of 16 is ill with DM type 1. In the morning takes Actrapid 16 U, Protaphan 20 U, before dinner – Actrapid 6 U, in the evening - Actrapid 4 U, Protaphan 12 U. Height 179 sm, weight 60 kg. Glucose level: fasting – 6,4 mmol/l, 13.00 – 8,8 mmol/l, 18.30 – 9 mmol/l, 22.00 – 7,1 mmol/l. Glcosurea 5 g/l. What has to be changed in treatment? A. To increase evening dose of Protaphan 92 B. C. D. E. To increase morning dose of Protaphan * To increase dose of Actrapid before dinner To not change doses of insulin To increase morning and evening doses of Actrapid 59. Patient P. 42 y.o. suffers from DM type 1, takes insulin Farmasulin 30/70 before breakfast – 32 U, before supper – 16 U. Glucemia during day – 8.00 – 7,5, 13.00 – 12,0, 18.00 – 14,2, 21.00 – 16,0, 3.00 – 9,0 mmol/l. What changes of treatment have to be made? A. To change doses of Protaphan in the morning to 30 U, in the evening to 12 U B. To decrease evening amount of carbohydrates C. To decrease day amount of carbohydrates D. To change doses of Protaphan in the morning to 36 U, in the evening to 20 U * E. To change insulin 60. Patient C. 17 years old with DM takes intensive scheme of insulin therapy. Presents with nightmares, in the morning – headache, general weakness. Glucemis: 8.00 – 10,1, 12.00 – 6,6, 17.00 – 4,3, 21.00 – 3.7 mmol/l. What is wrong in the insulin therapy scheme? A. Too high dose of semi-long acting insulin in the evening * B. Not enough dose of semi-long acting insulin in the evening C. Not enough dose of semi-long acting insulin in the morning D. Too high dose of short acting insulin in the evening E. Not enough dose of semi-long acting insulin per day 61. Child, 11 years old presents with recidivating of furunculous. Fasting blood glucose is 7,9 mmol/1, glucose and acetone absent in urine. What is the most likely diagnosis? 93 A. Diabetes mellitus type 1 B. Chronic infections C. Diabetes mellitus type 2 D. Dermatopathi E. Diabetic ketoacidosis 62. Patient V., 26 years old, complains of an itch about the genitals, frequent urination, tiresome thirst, weight gain. These symptoms were evidenced over several months. Fasting blood glucose - 12,0 mmol/1, glucose in urine - 1,5 %. What is the most likely diagnosis? A. Diabetes mellitus type 1 B. Diabetes insipidus C. Diabetes mellitus type 2 D. Dermatopathi E. Chronic infections 63. Patient with persistent dermatomyositis examined glucose and appearance such results: the fasting level – 5,55 mmol/L and 6,68 mmol/L, during the day 7,85;9,11;11,13 mmol/L. Your conclusion. A. Diabetes mellitus B. Impairment of carbohydrate tolerance C. Normal D. Necessary to order additional laboratory tests. E. Needed an add examination 64. Patient M., 27 years, after recovering from the flu, complaining of thirst, frequent urination, and weight loss. Blood glucose is - 12,3 mmol/1, in urine - 3%, acetone - +. What is your previous diagnosis? A. Diabetes mellitus type 1, inadequate control, ketosis B. Diabetes insipidus C. Diabetes mellitus type 2 94 D. Diabetes mellitus type 1, adequate control E. Diabetes mellitus type 2, inadequate control, ketoacidosis 65. Which of the statement given below is correct relatively to GTT? A. Correlates with determination of potential abnormalities of glucose tolerance B. Helpful to choose the most appropriate treatment C. Used to differentiate type of diabetes D. Indicate stage of diabetes E. Useful in the seeking of early diabetic complications 66. Interpret GTT. Glycemia: I trial - 5.3 mmol/1, II trial - 7.8 mmol/1, III trial - 4.8 mmol/1 A. Normal B. Impairment of carbohydrate tolerance C. Diabetes mellitus D. Necessary to order additional laboratory tests. E. Additional laboratory investigations are indicated 67. Patient 30 years is being ill by Diabetes Mellitus for 10 years. He accepts the insulin in a dose 46 OD in two injections. There are frequent hypoglycemic commas, decline of the sight, pain in lower extremities in anamnesis. The micro aneurisms, hemorrhage, expansion of veins of the retina are determined on the eyeing bottom. The fasting level of glucose is 13.2 mmol/L. What is your previous diagnosis? A. diabetes mellitus type 1, diabetic background retinopathy, diabetic foot syndrome B. diabetes mellitus type 2, diabetic proliferative retinopathy, diabetic foot syndrome 95 C. diabetes mellitus type 1, diabetic proliferative retinopathy, diabetic neuropathy D. diabetes mellitus type 1, diabetic preproliferative retinopathy, diabetic neuropathy, diabetic nephropathy E. Non of this variant 68. A patient W., which is ill for 15 years by Diabetes Mellitus type 1, during last year determine weakness, hypertonia, swelling of lower limbs and face. In the general analis of urine: albumen – 0,99 g/L, glucose 11 mmol/L, leukocyte 3 – 5, creatinine, urea are normal. The fasting level of glucose – 11,5 mmol/L. Established the most suitable diagnose? A. Diabetes Mellitus type 1, adequate control, diabetic nephropathy, persistent proteinuria B. Diabetes Mellitus type 1, inadequate control, diabetic nephropathy, persistent proteinuria C. Diabetes Mellitus type 1, adequate control, diabetic nephropathy, renal impairment D. Diabetes Mellitus type 1, inadequate control, diabetic nephropathy, microalbuminuria E. Non of this variant 69. A patient 32 years, for the first time diagnosed the Diabetes Mellitus type 1, complained of hypersensitive in the both of legs, especially in the region of thighs, touch of the clothes caused unbearable pain. Connecting appearance of pain with symptoms of diabetes and lost of weight. Your preliminary diagnosis? A. Diabetes mellitus type 1, diabetic distal symmetrical sensorimotor polyneuropathy B. Diabetes mellitus type 1, diabetic angiopathy C. Diabetes mellitus type 1, diabetic autonomic neuropathy D. Diabetes mellitus type 1, diabetic amyotrophy 96 E. Non of this variant 70. Patient 42 years old is ill by Diabetes Mellitus type 1, entered permanent establishment with with complaints about the loss of the weight, growing the weakness. He ills by diabetes for a year. He treats by insulin in a dose 12 OD Humudari R of rapid actuion and 18 OD Humudari B of prolong action. Diabetes is stable, hypoglycemia is not present. At a review: considerable grow thin, especially muscles. Pulsation on the distal parts of arteries of the feet is good, the trophic changes are not exposed. Achilles' and knee’s reflexes are considerably slow from the both sides. Vibration, tactile, thermal sensitiveness are not damage. Your previous diagnosis? a. Diabetes Mellitus type 1 in the stage of decompensation b. Diabetes Mellitus type 1, asthenia of somatogene. c. Diabetes Mellitus type 1, peripheral polyneuropathy d. Diabetes Mellitus type 1, total motor polyneuropathy e. Diabetes Mellitus type 1, total sensor polyneuropathy 71. What food caloric content must the patient have if he suffers from diabetes mellitus and has adiposity of the second stage? a. caloric content must be reduced on 20-30% on the account of limiting of hydrocarbons and fats. b. dose starvation prescribing c. caloric content must be reduced on 10 % on the account of hydrocarbons limiting d. physiological diet e. caloric content must be increased on 20-30 % on the account of hydrocarbons 72. Which of the enumerated complications can be caused by taking therapeutic doses of sulfanilamide sugar reducing preparations? 97 a. dysbacteriosis b. allergy c. breaking of hemopoesis b. gastrointestinal breaking e. all mentioned above 73. Name preparations of rapid- acting insulin: a. protafan NPH b. humodar B c. acktropide d. lantus e. humulin 74. Name preparations of intermediate-acting insulin: a. protafan NPH b. monotard NPH c. humulin d. insuman basal e. all mentioned above 75. Which of the below mentioned preparations of sulfanilurine belong to the second generation: a. glibenclamide b. glipizid c. gliclazid d. glicvidone e. all mentioned above 76. What number of bread units is necessary for a patient with an excessive mass of body? a. 20-24 BU b. 15-19 BU c. 10-15 BU d. 24-28 BU e. 29-32 BU 98 77. The principal purpose of diabetes mellitus, 1 type treating is: a. glycemia on the empty stomach 5,1-6,5 mmol/litre b. glycosylated haemoglobin 6,1-7,5 % c. normal physical condition of the patient d. prophylaxis of specific diabetes mellitus complications e. all mentioned above 78. What is the mechanism of metformin acting connected with? a. with an amplified glucose absorption by skeletal muscles and with inhibition of the hepatic gluconeogenesis b. with the increase produced of the quantity of the insulin c. with stimulation of pancreas d. with stimulation of hepatic gluconeogenesis e. with reducing produced the quantity of the insulin 79. A 23 years old patient suffers from the diabetes mellitus, 1 type. Disease was detected for the first time. Complaints of dryness and thirst. Ketoacidosis is absent. What dose of insulin a day does he need to normalize his hydrocarbon metabolism? a. 0,3 unit/kg/ a day b. 0,1 unit/kg/ a day c. 0,5 unit/kg/ a day d. 1,5 unit/kg/ a day e. 0,7 unit/kg/ a day 80. The patient, 55 years old, has suffered from the diabetes mellitus of the 2 type for 10 years. Complains of dryness, thirst and frequent excretion of urine. Has lost 8 kg of his weight during 6 month. His father also suffered from the diabetes mellitus/ During examination: glycemia on the empty stomack – 11.2 mmol/litre. During 24 hours – from 15.2 up to 20 mmol/litre.glycosylated hemoglobin – 14%. The last 6 years 99 this patient was treated with glibenclamide: 1 tablet, 3 times a day. Your tactics of treatment: a. to add behuanides b to increase the dose of preparations of sulfanilurine c to prescribe insulinotherapy d to prescribe combined sugar reducing reparations e. to prescribe another preparations of sulfanilurine 81. Patient, 28 years, is ill with diabetes during 5 years, intakes insulin in daily dose 38U. What scheme of insulinoterapy is indicated for patient with stable clinical course of disease? A. Traditional B. Intensifyed C. Only with insulins of short action D. Only with insulins of the prolonged action E. Some method from transferred is possible The right answer: A. 82. Patient, 28 years, is ill with diabetes during 5 years, intakes insulin in daily dose 38U. He was hospitalized with signs of ketoacidosis. What scheme of insulinotherapy is indicated for patient with ketoacidotic state? A. Only with insulin of the short action B. Intensifyed C. Traditional D. Only with insulins of the prolonged action E. Some method from transferred is possible The right answer: A. 83. Patient, 28 years, is ill with diabetes during 5 years, intakes insulin in daily dose 38U. There were frequent hypoglycemic and ketoacidotic states in anamnesis. Level of 100 the glycated haemoglobin 10,5%. What scheme of insulinotherapy is indicated for patient with labile clinical course of disease? A. Intensifyed B. Traditional C. Only with insulins of short action D. Only with insulins of the prolonged action E. Some method from transferred is possible The right answer: A. 84. Patient, 28 years, asked for help to endocrinologist with the first diagnosed diabetes without the clinical signs of ketoacidosis. Define the approximate daily amount of insulin for this patient? A. 0,5 U/kg B. 0,7-0,9 U/kg C. 1,0-1,5 U/kg D. 50 U a day E. 2 U on each 1 mmol/l of fasting glycemia The right answer: A. 85. Patient (woman), 46 years, is ill with diabetes mellitus during 9 years, inakes Humodar B - 26 U in the morning and 18 U in the evening. She complains about weakness, languor in the morning after sleep, headache, hyperhydrosis in the night. Objectively: pulse-72/min., BP 125/70 mmHg. Cardiac borders are normal. Liver +4cm. Glycemia: 8.00 - 14 mmol/l; 12.00 - 9 mmol/l; 17.00 - 11 mmol/l; 2.00 - 3,1 mmol/l. What are the reason of these complaints? A. Overdose of insulin in the evening. B. Underdose of insulin in the evening C. Presence of hepatosis D. Climacteric syndrome 101 E. Underdose of insulin in the morning The right answer: A. 86. Youth, 19 years, is ill with diabetes mellitus during two years. Objectively: tongue is dry. There is angular stomatitis. Diabetic blush. Liver +4cm, soft. Fasting glycemia 12,3 mmol/l. Glucosuria - 25g/l. There is increased level of triglicerides in plasma. It is necessary for the prophylaxis of complications of diabetes: A. Rich of normal level of glycemia B. Prescribe angioprotectors C. Prescribe the А, Е, С D vitamins D. Prescribe aldosoreductase inhibitors E. Prescribe hypolipidemic drugs The right answer: A. 87. Patient is ill with diabetes mellitus during 28 years. Dose of insulin was decreased on 14U last year. Analysis of urine: protein - 1,7 g/l, sugar - 0,8 %, many WBC, cylinders. These indicated signs are the display: A. Diabetic nephropathy B. Insulinoresistance C. Decompensation of diabetes D. Pyelonephritis E. Syndrome of the chronic overdosing of insulin The right answer: A. 88. Patient is ill with diabetes mellitus type 1 during 28 years. What scheme of insulinotherapy uses by patient, if he intakes 10U of Protaphan and 6U Aktrapid in the morning, in the evening - 8U of Protaphan and 6U of Aktrapid? A. Traditional B. Intensifyed C. Basis-bolus D. DAFNE method 102 E. Untraditional The right answer : A 89. Patient is ill with diabetes mellitus type 1 during 28 years. What scheme of insulinotherapy uses by patient, if he intakes 12U of Aktrapid in the morning, 10U of Aktrapid in the afternoon, 8U of Aktrapid and 12U of Protaphan in the evening? A. Intensifyed B. Traditional C. Combined D. DAFNE Method E. Untraditional The right answer : A 90. A 25 years old woman, has went for the first time with complaints of strong thirst, dryness in a mouth, frequent urination, growing thin along with the raised appetite. These complaints have appeared 2 months ago after her stressful situation. During examination the diabetes mellitus was revealed. What treatment is necessary to be administered to the patient? A. *Insulin therapy. B. Medicines of sulphonylureas. C. Biguanides. D. Diabetic diet only. Е. The patient does not require special treatment. 91. A 47 years old patient has been suffering from type 1 diabetes mellitus for last 5 years. The diabetes is at compensation stage. Patient was been directed on sanatorium therapy. What medical factors of a resort will help to improve a condition of the patient? А. *Mineral water. В. Climatic factors. 103 С. Therapeutic mud. D. Physiotherapeutic training. Е. All listed above. 92. Patient В is 26 years old and he has been suffering from type 1 diabetes mellitus. Uses insulin from group of porcine insulins. 2 months ago there was atrophy of subcutaneous fat in the places of insulin injections, especially on hips (where the insulin injections were done more often). What recommendations will help in eliminating the insulin lipodystrophy? A. *To use human insulin, B. To change places of injections necessarily, C. To inject the needle strictly perpendicular to the skin. D. To heat up the insulin dose till it reaches the body temperature before its use E. All listed above. 93. Patient Р has been suffering from type 1 diabetes mellitus for last 8 years. She went to an endocrinologist for getting the directions to the sanatorium therapy. What laboratory examinations will give the information about the compensation stage of the diabetes? A. *Level of glucose in blood on an empty stomach and after meal (pp), B. Level of glycosylated haemoglobin, C. Level of total cholesterol and triglycerides, D. The body mass index, E. All listed above. 94. A patient entered the clinic with diabetes mellitus of type 1, heavy form in the stage of indemnification, which complicated nephropathy , myocardial dystrophy and stagnant cardiac insufficiency of the II item Preparation of what group is it expedient to plug in complex therapy of patient for the 104 prophylaxis of thromboses? A.* Low molecular weight heparin ; B. Fibrinolitics; C. Plasminogen activators ; D. Non-steroidal anti-inflammatory agent; E. Indirect anticoagulants. 95. A patient, 27 years, is under surveillance of therapist with a diagnosis diabetes mellitus type 1. He works in the production carpenter, moderate physical activity. What calculation of bread units on days does require patient in accordance with his loadings? A.* 30-35 kcal/kg in a day – 21 BU; B. 40-45 kcal/kg – 30 - 35 BU; C. 35-40 kcal/kg – 25-30 BU; D. 20 - 25 kcal/kg – 14 BU; E. Contraindicated work moderate. 96. Patient with diabetes mellitus type 1, 27 years old. At present in a period of subindentification. The weight of patient does not differ from ideal. Choose the necessary dose of insulin from offered. A.* 0,7 UN/kg of weight; B. 0,2 UN/kg of weight; C. 0,5 UN/kg of weight; D.1,0 UN/kg of weight; E. 1,5 UN/kg of weight. 97. For a patient, 22 years old , diabetes mellitus is set type 1. Glukozuria makes 150 g on daily. Choose the necessary dose of insulin on the level of glukosuria. À.* 30 UN (1 UN insulin on 5 g of day's glukosuria, 150:5=30 UN) B. 75 UN insulin (1 UN insulin on 2 g of day's glukosuria, 150:2=75 UN) 105 C. 15 UN insulin (1 UN insulin on 10 g of day's glukosuria, 150:10=15 UN) D. 150 UN insulin (1 UN insulin on 1 g of day's glukosuria, 150:1=150 UN) E. 300 UN insulin (1 UN insulin on 0,5 g of day's glukosuria, 150:0,5=300 UN) 98. For a patient, 24, diabetes mellitus type 1. Gets 40 UN insulin a day. Feeling is satisfactory. What criteria of the right calculation of dose? A. The absence of the syndrome of Samadhi; B. The absence of the syndrome of «phosphor»; C. The absence of the syndrome of hunger, sweating , cramps; D.* All listed; E. Nothing from listed. 99. The sick, 20 years old, after the carried Rubella is set diabetes mellitus of type 1 . Weight of patient higher than middle norm. From anamnesis of sick – sick likes sweets too much. The grand-dad of sick also suffers diabetes mellitus. Name a main provocative factor in development of this disease for a patient A. * Viral infection; B. Genetic; C. Violations in a diet; D. Age of patient; E. Obesity. 100. Patient L., 49 years old, height 163 cm, weight 76 kg, level of fast (on an empty stomach) glucose is 6,3-7,28,7 mmol/l. What is the possible diagnosis? A. Normal. B. Impaired glucose tolerance. 106 C. Diabetes mellitus type 1 D. Impaired fasting glycemia. E. *Diabetes mellitus type 2 101. An overweight 56 y-r-old woman complains of having to go to toilet more frequently. She says she does drink a lot of tea but that she is always thirsty and tired. She needs the energy. Prescribe investigations, please. A. Fasting blood glucose* B. Urinary ketones C. T3, T4 and TSH levels D. HbAlc levels E. C-peptide levels 102. A 31-year-old Hispanic female is hospitalized with polyuria, thirst, and blurred vision. Her is 161 cm (63 in) tall and weighs 121 kg (265 lb). Acanthosis nigricans is noted on his neck. Plasma glucose concentration is 17 mmol/l (323 mg/dL); serum ketones are negative and arterial blood gases are normal. The patients grandmother has type 2 diabetes mellitus. What is the most likely cause of this patient’s hyperglycemia? A. Type 1A diabetes mellitus B. Type 1 B diabetes mellitus *C. Type 2 diabetes mellitus D. Secondary diabetes E. Maturity-onset diabetes of youth 103. A 50 year old male has type 2 DM. Which one of the following statements about non-insulin dependent diabetes mellitus (type 2 DM, NIDDM) is NOT true ? A. Circulating islet cell antibodies are usually found* 107 B. There is not HLA association C. Ketosis is rare D. Relative resistance to insulin is present E. Obesity is common 104. Diabetic N., 50 years old, takes metformin 3000 mg/day. Was founded level of fasting glucose – 6,9 mmol/l, aglucosurea, angiopathy of lower extremities - functional stage, nonproliferative retinopathy. Choose the degree of severity of the diabetes mellitus, please. A. Mild. B. *Moderate. C. Severe D. Stabile. E. Progressive. 105. A 63-year-old woman presents with a fasting plasma glucose level of 9,8 mmol/l and no ketones on routine examination. She is asymptomatic and has no evidence of microvascular or macrovascular disease. She is 157 cm (62 in) tall and weighs 51 kg (110 Ib). She takes no medications and has no family history of diabetes. What treatment is necessary in this case? A. Diet therapy* B. Insulin therapy C. Does not need any treatment D. Biguanides E. Sulphonilurea medicines 106. Patient R., 45 years old, height 160 sm, weight 83 kg. 3 monthes ago was diagnosed DM type 2, used diet therapy, and lost 5 kg of body mass. Fasting glucose – 12 mmol/l. What is the next step in his treatment? А. Sulfonilurea medicines I generation 108 B. Insulin therapy C. Sulfonilurea medicines III generation D.Biguanides* E. Sulfonilurea medicines II generation 107. Patient P. 52 y.o., height 162 sm, weight 92 kg. He is ill with diabetes mellitus type 2 during 4 years. Uses diet therapy and metformin 3000 mg/day. Fasting glucose – 10,4 mmol/l, postprandial glucose – 12,8 mmol/l. What treatment has to be used in this case? А. To add sulfonylurea medicines I generation B. To prescribe short-acting insulin C. To prescribe semilong-acting insulin D. Biguanides E. To add sulfonylurea medicines iI generation* 108. Patient D. 65 years old, suffers from diabetes mellitus type 2, got viral hepatitis A. During last 2 years used Glibenclamid 15 mg/day. Fasting glucose – 13,6 mmol/l. What treatment has to be prescribed? А. To prescribe glimepirid (sulfonylurea medicine III generation) B. To add biguanides C. To increase dose of Glibenclamid to 20 mg per day D. To add Acarbosa E. To prescribe insulin therapy* 109. Patient F. 62 years old is ill with diabetes mellitus type 2. To receive compensation it is enough for him to use diet and maninil (glibenclamid). He is going to have operation of stomach resection. What has to be changed in the treatment of DM? А. To change maninil for glimepirid B. To change maninil for glurenorm 109 C. To prescribe short acting insulin* D. To prescribe long acting insulin E. To prescribe biguanides 110. Patient K., suffering from Diabetes Mellitus for the past 10 years, fell into a coma. His skin is dry, breathing is loud and there is an acetone smell around him. Pulse 130 /minutes, the arterial pressure 70/45 mmHg. The reaction of the urine to acetone is positive (+++), the glucose of the blood - 30 mmol/1. What type of coma is this? A) ketoacidosis ; B) hyperosmolar; C) lactic acidosis; D) hypoglycemic; E) brain. 111. Patient X., 67 years old. Duration of Diabetes Mellitus type 2 during 9 years. Patient keeps to the prescribed diet, receives regular treatment with oral drugs, and several times in the few last years was treated by sulfonylureas with maximal therapeutic doses. What complication should you consider? A. Hyperglycaemia B. Hypoglycaemia C. Diabetic ketoacidosis D. Hyperosmolality E. Lactic acidosis 112. Patient E, 24 years, is suffering from diabetes mellitus type 1 during 3 years. Several days ago she canceled the insulin. Patient lost consciousness in the evening. She is hospitalized. The skin is dry. Kussmaul's respiration, the odor of acetone from the mouth, tongue is dry, with fur of brown colour. Pulse 120 /minutes, the arterial pressure 80/45 mmHg. The abdomen does not react to the palpation. The liver is + 3 cm. 110 B. C. D. E. B. C. D. E. A. B. C. D. E. The reaction of urine to acetone is positive, the glucose of blood - 28 mmol/1. The preliminary diagnosis is: A. Diabetic ketoacidotic coma Lactacidemic coma Infectious - toxic shock Hepatic coma Hyperosmolar coma 113.The unconscious woman of 25 years is delivered to hospital. She is suffering from Diabetes Mellitus for the past 8 years. In the morning the patient could not wake. The skin is dry, the turgor is reduced, eyeballs are mild. Kussmaul's respiration, the odor of acetone from the mouth. The pulse - 130 /minutes, the arterial pressure - 100/65 mmHg. The liver is +2 cm. The glucose of blood- 25,5 mmol/1. The emergency treatment will consist of introduction: A.Rapid-acting insulin in the dose of 10 U hourly i/v Insulin of prolonged acting 10 U hourly i/v 40% glucose solution Reopolyglukin up to 1,0 L 0.9% NaCl solution 114. Patient C, 24 years with the 10-years experience of diabetes type 1 is unconsciousness. He has lost consciousness after intensive physical activity. Objective: a patient is pale, the skin is wet, tongue is wet too. Respiration is superficial, the arterial pressure - 140/70 mmHg. Tone of muscles and tendon reflexes are raised. Establish the diagnosis: Hypoglycemic coma Lactoacidotic coma Hyperosmolar coma Diabetic ketoacidotic coma Cerebral coma 115. The patient with diabetes type 1, the severe form, is 111 B. C. D. E. F. B. C. D. E. compensated. After game of football he has lost consciousness. Objective: a patient is pale, the skin is wet, tongue is wet too. Respiration is superficial, the arterial pressure - 110/70 mmHg. Tone of muscles and tendon reflexes are raised, meningeal signs are absent. The emergency treatment will consist of introduction: 40-80 ml 40% glucose solution Reopolyglukin up to 1,0 L Rapid-acting insulin in the dose of 10 U hourly i/v 0.45% NaCl solution 0.9% NaCl solution 116. One of the main causes of hypoglycaemia is A. Diarrhoea B. Stress C. Weight loss D. Weight gain E. Unaccustomed exercise 117. The patient of 63 years suffers from insulindependent form of diabetes. On the background of acute infringement of the cerebral circulation the high hyperglycemia56 mmol/1, severe dehydration, hyper-chloremia, hypernatremia has developed. The ketonemia and acetonuria are absent. Establish the correct diagnosis: A hyperosmolar coma Diabetic hyperketonemic coma Acute renal failure Hyperlactacidemic coma Chronic renal failure 118. Point out to the features which are characteristic for the 3 stage of the diabetic nephropathy: a. proteinuria b normoalbuminuria 112 c. hyperfiltration d. mikroalbuminuria e. hyperperfusion 119. Point out to the features which are characteristic for the diabetic proliferative retinopathy: a. hemorrhages into the vitreus body b. rubeosis c neovascularization of the disk of the optic nerve d. exfoliation of the retina e. all mentioned above. 120. What changes on the eye bottom must take place in case of the preproliferative diabetic retinopathy to make lasercoagulation necessary. a. all mentioned below b. sinuosity of the vessels c. uneven expansion of the venulae d. soft exudates e. There are a lot of microaneurysma and retinal hemorrhages. 121. To the neuropathic form of the syndrome of diabetic foot belong: a. acral micosis b. pale skin c. absence of pulsation on the feet’s arteriae d. cyanosis of skin e. dry skin, areas of hyperkeratosis 122. The duration of treatment for a patient having the third stage of ulcerous lesions must be: a. from 2 weeks to 1,5 month b. from 2 to the 3th months c. from 6 to 12 months d. from 4 to 5 months 113 e. from 5 to 6 months 123. The patients who belong to the risk category with the neuropathy and the ischemia of extremities need examination: a. once in 6 months b. once in 9 months c. once in 1-3 months d. once in 12 months e. once in 24 months 124. Treatment the diabetic autonomous neuropathy includes: a. adequate sugar reducing therapy b. sulfur-containing preparations c. neurotropic vitamins d. using elastic high socks e. all mentioned above 125. The basic principles of treatment of the trophic lesions of lower extremities for patients suffering from diabetes mellitus are: a. optimization of the metabolic control b. antibakterial therapy c. local treatment of the ulcer d. all mentioned above e. unloading of the wounded extremity 126. Conservative treatment of the diabetic retinopathy is accomplished on the account of: a. keeping to dietotherapy b. all mentioned c. adequate sugar reducing therapy d. correcting the hemodynamics of retina e. correction the hyperlipidemia 114 127. To treat patients suffering from the diabetic nephropathy on the stage of microalbuminuria the following must be done: a. correction of hydrocarbonic metabolism b. correction of the remal hemodynamics c. correction of the arterial tension d. correction of the hyperlipidemia e. all mentioned above 128. Patient (woman), 26 years. She is ill with diabetes mellitus type 1 during 5 years and intakes insulinotherapy. She complains about hyperhydrosis, sense of hunger, palpitation in bedtime after last hospitalization during last half a year. She put on yhe weight on 4kg during last 3 months. Glycemia: 8.00 - 12mmol/l; 12.00 - 9mmol/l; 17.00 - 8.5mmol/l; 2.00 2.9mmol/l. Specify possible complication of insulinotherapy? A. Somoji's syndrome B. Lipodystrophy C. Intolerance to insulin D. Insulin edemata E. Insulinoresistance The right answer : A 129. Patient, 48 years, is ill with diabetes mellitus during 8 years. He intaked metformin 850mg twice a day. The last three months he notes increase of intensity of diabetic complaints, fasting glycemia - 9-12mmol/l. Patient lost 6kg of body mass during these time. This is decreased level of Cpeptide in blood. What medical tactic is indicated to this patient? A. Prescribe insulin B. To double the daily dose of metformin C. To appoint thiazolidindiones D. To cancel metformin and prescribe sulphonylureas 115 E. The state of patient is not related with diabetes The right answer: A. 130. Patient, 48 years, is ill with diabetes mellitus during 8 years. He intaked metformin 1000mg twice a day. The last three months he notes increase of intensity of diabetic complaints, fasting glycemia - 9-12mmol/l. Patient lost 2kg of body mass during these time. This is normal level of C-peptide in blood. What medical tactic is indicated to this patient? A. To add insulin secretion stimulatos B. To double the daily dose of metformin C. To appoint short action insulin D. To cancel metformin, to prescribe a-glucosidase inhibitors E. The state of patient is not related with diabetes The right answer: A. 131. Patient, 48 years, is ill with diabetes mellitus during 8 years. He intaked metformin 1000mg twice a day. Fasting glycemia – 8-9 mmol/l. He prepares for surgery of cholelitiasis. What tactic of treatment of diabetes during surgical treatment is need for this patient? A. To transfer on short action insulin B. To double daily dose of metformin C. To prescribe sulphonylureas D. To cancel metformin, to prescribe a-glucosidase inhibitors E. To leave treatment without changes The right answer: A. 132. Patient, 48 years, is ill with diabetes mellitus during 8 years. He intaked metformin 1000mg twice a day. Fasting glycemia – 8-9 mmol/l. He was hospitalized with diagnosis macrofocal pneumonia. Which adverse events may 116 A. B. C. D. E. be in treatment by metformin? A. Lactatacidosis B. Hypoglycemic coma C. Abscess formation in lungs D. Pneumothorax E. Pulmonary embolism The right answer: A. 133. Diabetes mellitus was diagnosed in patient, 48 years. Fasting glycemia – 8-9 mmol/l. Which from listed states insulinotherapy is not indicated to this patient in? A. First diagnosed diabetes. Obesity III degree B. Diabetes mellitus type 1, moderate severity, compensation. Pregnancy C. Diabetes mellitus type 1, moderate severity, decompensation. Diabetic ketoacidosis D. Hematologic diseases E. Diabetes mellitus type 2. Acute myocardial infarction The right answer: A. 134. Diabetes mellitus was diagnosed in patient, 48 years. Fasting glycemia – 8-9 mmol/l. Which from listed states oral sulphonylureas are indicated to this patient in? Diabetes mellitus type 2, moderate severity. Diabetic angioretinopathy 1degree Diabetes mellitus, mild form, compensation Diabetes mellitus type 1, compensation. Diabetic nephropathy, nephrotic stage Diabetes mellitus type 2, moderate severity, compensation. IHD, acute myocardial infarction, heart failure 1 functional class Diabetes mellitus type 1, moderate severity, compensation in cholecystectomy 117 A. B. C. D. E. The right answer: A. 135. Diabetes mellitus was diagnosed in patient (woman), 39 years. Fasting glycemia – 8-9 mmol/l. Which from listed states oral sulphonylureas are indicated to this patient in? Extraction of a tooth Cytopenic states of any genesis Acute hepatitis Lactation and pregnancy Severe exacerbation of chronic infection The right answer: A. 136. What is the main indication to prescribing of sulphonylureas? A. Diabetes mellitus type 2 in adults B. Juvenile diabetes mellitus C. Labile clinical course of diabetes mellitus D. Diabetes mellitus type 2 in pregnant E. Diabetic ketoacidosis The right answer : A 137. What from these preparations increase utilization of glucose in peripheral tissues? A. Metformin B. Trental C. Glibenclamide D. Nicotine acid E. Аcarbose The right answer : A 138. A 65 years old man (of 169 cm height and 84 kg weight) has come to polyclinic to an oculist with complaints to aggravation of vision. Type 2 Diabetes mellitus was revealed after examination. What medicine for treatment of Diabetes Mellitus are the most rational for this patient? 118 A.* Biguanides. B. Insulin. C. Medicines of sulphonylureas. D. Thiazolidinediones. E. Inhibitors of glucose absorption UNthe gastrointestinal tract (α-glucosidase inhibitors). 139. A patient of type 2 diabetes mellitus has been taking Metformin (Dymethylbiguanide) for 3 years, drank alcohol during a friendly meeting. After this he felt accrued general weakness, nausea, muscle pain, palpitation. There was a unitary vomiting. What is the most probable reason of these symptoms? A. *Development of hidden lactate-acidosis. B. An alcoholic poisoning. C. Overdose of biguanides. D. An insufficient dose of biguanides. E. Development of a reactive pancreatitis. 140. Woman D, being 68 years old, has been suffering UNtype 2 diabetes mellitus and receiving Gliclazide for last 8 years. Level of glycemia is increased, the patient has lost weight. What an endocrinologist’s tactic can be in this case? A. *To discontinue Gliclazide and to administer with insulin. B. To increase the dose of Gliclazide. C. To administer Biguanides along with Gliclazide. D. No change in the treatment. E. To administer Thiazolidinediones along with Gliclazide. 141. A patient with type 2 diabetes mellitus orally takes Glimepiride 5 mg daily and has got a good results. Patient has got cholelithiasis (a biliferous disease). Symptoms of mechanical jaundice arose 5 days ago. The conservative 119 therapy of jaundice has not given its results yet. Cholecystectomy should be done on the patient. What an endocrinologist’s tactic can be for use of Glimepiride in this case? A. *To discontinue the drug during the treatment at surgical department. B. To increase the dose of Glimepiride up to 8 mg a day. C. To decrease the dose of Glimepiride up to 3 mg a day. D. To administer biguanides with it. E. No change in the treatment. 142. A patient, 45 years, is observed during 3 years concerning diabetes mellitus type 2. What must be criteria of indemnification (in accordance with offered the European group of policy of treatment of diabetes)? A. Absence of glucosuria , absence of ketosis , and the hypoglycemia states; B. Glycemia on empty stomach : adequate level - 5,1– 6,5 mmol/l, satisfactory - to 7,8 mmol/l, glycemia after a meal: adequate level - 7,6–9 mmol/l, satisfactory - to 10,0 mmol/l; C. Level glycosylated HbAlc – to 6,1-7,5 %, table of contents of cholesterol - to 6,5 mmol/l; D.* All of the above-mentioned; E. Nothing UNthe above-mentioned. 143. A 55-year-old type 2 diabetic patient has lost weight and has had good control of his blood sugar on oral agents. He has a history of mild hypertension and hyperlipidemia. He asks for advice about an exercise program. Which one of the following statements is correct? A. A stress test should be recommended prior to beginning an exercise program B. Exercise should be avoided because it may cause foot trauma C. An active lifestyle cannot slow the complications of diabetes 120 D. Vigorous exercise cannot precipitate hypoglycemia E. Start an exercise program an any cases 144. A newly diagnosed type 2 diabetic patient asks for clarification about dietary management. Which of the following is good advice? A. Less than 10% of caloric intake should be saturated fat B. Restrict carbohydrates and eat a high-protein diet C. Avoid sucrose altogether D. Caloric intake should be very consistent from one day to another E. Eat a high-carbohydrates diet 145. At sick Н, 44 years, are adiposity of 2 degrees, an arterial hypertensia of II stage. Has addressed to the gynecologist concerning an itch in perineum. As a result of inspection the diabetes 2 types is revealed. There are initial displays of an angiopathy and a polyneuropathy. Choose the most rational therapy for the given patient. А.* Diabeton MR - 30 mg once a day; B.Humalog - 10 UN in day; C.Novorapid - 20 UN in day; D.Humodar - 30 UN in day; E. Sofor of 500 mg 2 times a day. 146. At patient К, 50 years, are a diabetes 2 types the easy form, growth - 165 sm, weight - 70 kg. Choose, that corresponds to principles dietotherapy for the given patient. A. Limitations easy digestible carbohydrates; B. Valid receipt of fats, fibers, carbohydrates; C. Except sugar, sweets, honey, мороженное; D. Use of sugar substitutes - sorbite, ксилит, fructose, steviosid; E *All aforesaid. 121 A. B. C. D. E. A. B. C. D. 147. At patient М, 50 years, the diabetes 2 types of average weight is revealed. There is postprandeal hyperglycaemia. Choose a preparation from offered for monotherapy and elimination of this complication А.* It is novo-norm; B. Metformin; C. Glibenklamid; D. Insulin; E. Gljurenorm. 148. Patient B., 60 years, has the type 2 of DM for 14 years. Complaints on headache, itching of the skin, appearing of the edema on the legs and face. He has hypertension during 6 years. In the urine test: proteinurea is 2,99 gm/l, glucosurea is 20 gm/lt. the level of creatinine is 0,27 mmol/l, urea – 14,3 mmol/l. Fasting serum glucose level is 12,1 mmol/l. What oral hypoglycaemic agent have you to choose for the patient? maninil chlorpropamid metformin *glurenorm acarbose 149. Diabetic H., 44 years old, admitted to the endocrinology department with complains of weakness, polydypsia, dyspnea. Objective examination reveals dry skin and tongue, Kuss – Maul’s breath. What investigation you have to prescribe? Daily glucosurea Glucose tolerance test. The level of postprandial glucemia. *The level of acetone in urea 122 A. B. C. D. E. E. The level of bilirubinemia 150. Diabetic F., 52 years old, takes maninil 3,5 mg twice a day. After activization of chronic bronchitis began to take biseptol 480 mg (2 tabl. twice a day). He complains of increased appetite, tremor of fingers, headache. What is your previous diagnosis? Allergic reaction Lactic acidosis Diabetic ketoacidosis *Hypoglycemic status. Hyperosmolar coma 151. Patient P., 34 years, was hospitalized. She is unconscious. Her relatives told you, that she has type 1 DM and felt herself bad 3 days ago after influenza, when she decreased the dose of insulin. She had weakness, fatigue, nausea and vomiting. Examination: dry skin and tongue, hyporeflexia. Laboratory findings: blood glucose level – 29 mmoll/l, acetone in urine is positive. What is your previous diagnosis? A. Hypoglycemic coma B. *Diabetic ketoacidosis C. Infectious endocarditis D. Hyperosmolar coma. E. Lactic acidosis 152. A 32-year-old female, insulin dependent diabetic, presents with failure to pass urine. Which complication of diabetus can be found? A. Hyperglycaemia B. Hypoglycaemia C. Urinary tract infection D. Kidney calculi E. Autonomic neuropathy* 123 A. B. C. D. E. A. B. C. D. E. 153. Patient F., 24 years old, has diabetes mellitus during 14 years complains of tachycardia, which can’t treated by β-blockers. During physical examination was found blood pressure: 190/110 mm of Hg in horizontal position and 80/ 60 mm of Hg in vertical position. What is your previous diagnosis? Diabetic peripheral neuropathy Thyrotoxicosis Hypertensive disease Myocarditis *Diabetic cardioneuropathy 154. Diabetic R., 46 years old complains of changes of the foot: it becomes shorter and wider. During examination was found painless swelling of the feet without edema or signs of infection, external rotation, and flattening of the longitudinal arch. Which complication of diabetus can be found? *Diabetic arthropathy. Diabetic neuropathy. Diabetic angiopathy. Diabetic foot. Diabetic myelopathy 155. Patient K., 31 years old, has diabetes mellitus during 7 years, takes insulin (54 Units) in 2 injections. Was found such changes on retina: microaneurtisms, dilation of vessels, haemorrhages, soft and hard exudates. What stage of retinopathy does patient have? Background retinopathy. *Preproliferative retinopathy. Proliferative retinopathy. Retina is normal. Nonproliferative retinopathy. 124 A. B. C. D. E. A. B. C. D. E. 156. Patient B., 41 years, has the type 1 of DM for 16 years. Complaints on periodical hypertension (during the last year), appearing of the edema on the legs and face. In the urine test: proteinurea is 0,98 gm/l, glucosurea is 30 gm/l. Creatinine, urea within the pale of norm. Fasting serum glucose level is 16,1 mmol/l. What is your diagnosis? *DM type 1, severe degree, stage of the subcompensation. Diabetic nephropathy, III stage. DM type 1, moderate degree, stage of the subcompensation. Diabetic nephropathy, III stage DM type 1, severe degree, stage of the compensation. Diabetic nephropathy, III stage DM type 1, severe degree, stage of the subcompensation. Diabetic nephropathy, IV stage DM type 1, moderate degree, stage of the subcompensation. Diabetic nephropathy, IV stage 157. Diabetic R., 49 years old, complains of symmetrical sensory loss, pain at night and during the rest, hyporeflexia, decreased response touch, burning of heels and soles. The skin becomes atrophic, dry and cold. What is your previous diagnosis? *Diabetic distal polyradiculoneuropathy. Diabetic truncal polyradiculoneuropathy. Diabetic truncal monoradiculoneuropathy. Diabetic myelities. Diabetic myelopathy. 158. Woman 59 years old, has been illed by Diabetes Mellitus during 20 years. Treated oneself by the oral preparations that reduce the level of sugar. There is diabetes in the state of decompensation. Does not adhere to the diet. Stomach-aches and swelling are disturbed, emptying is liquid, 125 a. b. c. d. e. f. g. h. i. j. their incontinence. A skin is dry, with easy hyper pigmentation. The tongue is red, ”geographical”. A stomach hurts at palpation. The lower edge of liver comes forward on 4 centimeters from under a costal arc. Your previous diagnosis? Diabetic enteropathy Cholecystitis Diabetic hepatosis Hepatitis Colitis 159. Patient 42 years old is ill by Diabetes Mellitus type 1, entered permanent establishment with with complaints about the loss of the weight, growing the weakness. He ills by diabetes for a year. He treats by insulin in a dose 12 OD Humudari R of rapid actuion and 18 OD Humudari B of prolong action. Diabetes is stable, hypoglycemia is not present. At a review: considerable grow thin, especially muscles. Pulsation on the distal parts of arteries of the feet is good, the trophic changes are not exposed. Achilles' and knee’s reflexes are considerably slow from the both sides. Vibration, tactile, thermal sensitiveness are not damage. Your previous diagnosis? Diabetes Mellitus type 1 in the stage of decompensation Diabetes Mellitus type 1, asthenia of somatogene. Diabetes Mellitus type 1, peripheral polyneuropathy Diabetes Mellitus type 1, total motor polyneuropathy Diabetes Mellitus type 1, total sensor polyneuropathy 160. Patient 38 years old is ill by Diabetes Mellitus type 1, complains about impotence and the loss of the weight, progressing the weakness. . He ills by diabetes for 12 years. Metabolic compensation of disease is good. Independently actively watches on the glucose level in the blood.Ttreats oneself by insulin in the 4th injections in a dose 36 OD per day. 126 a. b. c. d. e. f. g. h. i. j. The symptoms indicated earlier began to disturb during a year. At a review: large loss of the weight. The step is slow. The abdominal and cremasters reflexes are sickly. Vibration, tangent sensitiveness are reduced. Hyperesthesia of the skins is on the thighs. Your previous diagnosis? Diabetes Mellitus type 1, autonomous neuropathy Diabetes Mellitus type 1 in the stage of decompensation Diabetes Mellitus type 1, chronic sensory neuropathy Diabetes Mellitus type 1, peripheral polyneuropathy Diabetes Mellitus type 2, in the stage of decompensation 161. Patient 56 years old is ill by Diabetes Mellitus type 2, complains about dizziness, darkening in the eyes at the getting up. These phenomena accompany perspiration on the face, neck, thorax. The reception of the meal does not give the facilitation. He ills by diabetes for 42 years. The last year he adopted Mannil in a dose 1 or 2 pills per day. Compensation of the disease is satisfactory. The indicated complaints coincided with resistance tachycardia. AP – 140/95 mm.hg., standing – 90/60 mm.hg. Your previous diagnosis? Diabetes Mellitus type 1. Diabetic vegetative neuropathy of a heart Diabetes Mellitus type 1, climacteric syndrome. Dismetabolic cardiopathy Diabetes Mellitus type 1. Diabetic miocardiopathy Diabetes Mellitus type 1,atherosclerotic cardiosclerosis Non of this variant. 162. Patient 52 years old, is ill by Diabetes Mellitus type 1, complains about about the diarrheas which disturb her mainly at night, is carried the imperative character. The amount of emptying arrives at 30 or 40 for a night.She feels discomfort from the side of digestion organs during 2 years. Disturbed swelling in a epigestrial area, periodic obstipations and 127 a. b. c. d. e. a. b. c. d. e. emptyings, that was interpreted as chronic pancreatitis. Adopted enzymic preparations without the special success. He ills by diabetes for 18 years. She treats by the insulin monotard in a dose 46 OD in the morning. The fasting level of the glucose in scopes from 11 to 13 mmol/L. Without regard to emptyings she doesn’t have the loss of weight. Your previous diagnosis? autonomous enteropathy ulcerous colitis syndrome of the malabsorbtion syndrome of irritation of the colon sharpening of the chronic pancreatitis 163. At a patient with the heart attack of myocardium the level of the glucose is 8.2 mmol/L. 2 years passed from the beginning of the disease. What is the most expedient inspections to appoint the patient for estimation of the state of carbohydrate exchange? Determination level of the Hb-alc. proteins Test of the tolerance to glucose Determine the fasting level of the glucose during 3 days Determination of day's glucoseurine Determination level of the glucose in the blood for a day 164. A patient K., 36 years old, which complains on the headache, eyestrain, loss of appetite. He is ill during 18 years on the diabetes mellitus 1. During the examination: pale, swell face, heart increase in the left sight on 1,5 sm. Pulse – 70/min, rhythmical, pressure 180/100 mm.hg.. Stomach is soft, liver doesn’t increased. Pasternackyj symptom’s is negative with both sight. Urination – accelerating, mainly in the night. Analysis of the blood: Hb 98 g/l, er. 3* 1012 /L.lk. 8,2 *109 /l, urea 15,8 mM/l. Cholesterin 7,6 mM/l, albumen – 81 g/l, creatynine 177 mcM/l, glucose 8,2 mM/l. 128 F. G. H. I. J. A. B. C. D. E. Analysis of urin : specific gravity 1010, albumen 0,6 g/l, er. 1-3, l. 10-15, glucose 2 g/l, ECG: hypertrophy of the left ventricle. Your preliminary diagnosis? Diabetes mellitus 1, hard form in the stage of decompensation. Diabetic nephropathy. Diabetes mellitus 1, high pressure disease, stage 3. Chronic glomerulonephritis. Diabetes mellitus 1, hard form in the stage of compensation. Chronic pyelonephritis, high pressure disease. Non of this variant. 165. Which regime of insulinotherapy is the most correct for a patient, who is ill by diabetes mellitus and which is complicated by encephalopathy? A. Injection by insulin of long – term action in the morning and addition injection by insulin short – term action before taking food. B. One take injection by insulin of long – term action. C. Repeated injection by insulin of rapid – term action. D. Two injection by insulin per day. E. Non of this variant. 166. Which is the most effective method of prophylactic proliferous retinopathy? Laser photocoagulation. Courses of anticoagulants. Anticoagulants straight action. Courses of angioprotectors. Anabolic steroids. 167. In a boy of 5 years, after measles vaccination a temperature of the body is 37,8 0C, a big thirst, polydipsia, polyuria. Weakness is increasing during 6 hours from the 129 A. B. C. D. E. beginning of the disease, noisy breathing, sleepy. Your preliminary diagnosis? Diabetes mellitus. Postvaccination syndrome. Typical postvaccination reaction. Postvaccination allergy. Non of this variant. 168. In case of hypoglycemic coma following remedies must be immediately prescribed: a. short-acting insulin b. biguanides c. sulfanilurine preparations d. glucose 169.To help the patient out of the hyperosmolar coma besides insulins the following remedies must be prescribed using intravenous droppering: a. hypotonic Na chloride solution b. isotonic Na chloride solution c. 4% Na hydrocarbonate d. 10% Na chloride solution 170.With the hypoglycemic coma the following can be revealed: a. Kussmaul’s respiration b. acetone odor in the expired air c. blood omolarity increases d. normal state of the eye apples e. glucosuria 171.A 9 year old girl has been suffering from diabetes mellitus for 3 years. The patient has fallen ill with acute respiratory infection. Her state became worse during the last two days, the temperature reached 38 C, the thirst and the frequency of urination increased. General weakness enlarged. 130 Nausea and vomiting appeared. General condition is grave. Consciousness is dizzy. Rubeosis. The tungue is dry. Heart tones are rythmic, accelerated. What is your diagnosis? a. hypoglycemic coma b. ketoacidotic coma c. hyperosmolar coma d. pneumonia 172.To the number of the main insulin effects we refer: a. stimulating glucagone synthesis b. lithogenesis stimulating c. stimulating proteon synthesis d. all mentioned above 173.The patient K, of 56 years old, has suffered from the diabetes mellitus for 8 years. Owerweight of her body is 8 kg. The state of her health become much worse after she had enterocolitis (vomiting, diarrhea). She was urgently hospitalized. Consciousness is absent. Expired air does not contain the odor of acetone. Breathing perfunctory accelerated, skin and mucous coats dryness. Lowering the eyeballs’ tone. The diagnosis will be confirmed with determining: a. the level of sugar in the blood b. the level of sugar in the urine c. osmolarity d. all mentioned above 174.With a differential diagnostics between the hyper and hypoglycemic comas the following should be taken into account: a. the beginning of the coma b. apple eyes tone c. Kussmaul’s respiration d. acetone odor in the air e. all mentioned above 131 175. The patient M. suffers from the diabetes mellitus from the age of 6. When brought into the department, her state was grave. A day before, after the diet breaching vomiting and nausea appeared. Because of the lack of appitite, mother injected half a dose of insulin. The girl drank cold tea. The following day the morning dose of insulin was missed and the girl was brought into the endocrinological department. The child is somnolent, her consciousness is intricate. There is stiff respiration in the lungs. Heart sounds are rhythmic, accelerated. Her liver is palpated 4 cm lower than the costal arch. Your preliminary diagnosis is: a. hyperosmolar coma b. lactacidemic coma c. hypoglycemic coma d. ketoacidotic coma 176. For the lactacidemic coma is not characteristic: a. Kussmaul’s respiration b. tachicardia c. hypotonia d. oliguria, anuria e. hyperthermia 177. For helping the patient out of the lactacidemic coma it is necessary to inject intravenously: a. physiological salt solution b. Na bicarbonate solution c. 5% glucose solution d. all mentioned above 178. What from these states are indicated for prescription of insulinotherapy in patients with diabetes mellitus type 2? A. All it's true B. Proliferative angioretinopathy 132 A. B. C. D. E. A. B. C. D. E. C. Ketoacidotic state D. Myocardial infarction E. Aorto-coronal shunting The right answer : A 179. Patient, 24 years, is ill with diabetes mellitus during 8 years. He uses insulinotherapy. He was hospitalized unconscious. Which from listed actions is need for treatment of hypoglycemic coma? IV stream introduction of 40% glucose IV droply introduction of 10% glucose IV droply introduction of 40% glucose IV droply introduction of 5% glucose IV droply introduction of 40% glucose and 6-8U of insulin The right answer: A. 180. Patient, 54 years, is ill with diabetes mellitus during 8 years. He uses insulinotherapy. He was hospitalized with glycemia 45 mmol/l unconscious. Which from listed symptoms are not typical for hyperosmolar coma? Ketoacidosis Local neurological symptomatic High hyperglycemia Dehydration Hypopnoe, tachypnea The right answer: A. 181. Patient, 63 years, is ill with diabetes mellitus type 2. High hyperglycemia, sharp dehydration, hyperchloremia, hypernatriemia were developed during acute stroke. Ketonemia and acetonuria are not present. Define true diagnosis: A. Hyperosmolar coma B. Acute renal failure C. Hyperketonemic diabetic coma D. Chronic renal failure 133 E. Hyperlactatacidemic coma The right answer: A. 182. Coma developed in man, 25 years, that is ill with diabetes mellitus during 8 years. Objectively: a skin is dry, turgor is reduced, Cussmaul's breathing's present, BP - 105/60 mmHg, HR -116/min, smell of acetone in mid air. What is the type of coma in this patient? A. Ketoacidotic B. Hyperosmolar C. Lacatacidotic D. Hypoglycemic E. Cerebral The right answer: A. 183. Coma developed in man, 28 years, with bronchopneumonia. Objectively: HR - 122/min, extrasystoles. BP - 80/45 mmHg. Skin is dry, turgor is reduced. Breathing is deep, noisy, liquid. Strong smell of acetone in mid air. Liver +5cm. Glycemia -32 mmol/l. рН of blood - 6,9. What solution is most directed to operate on normalization of metabolism? A. 4,2% solution of sodium hydrocarbonatis B. 5% solution of glucose C. 0,9% solution of sodium chloride D. 1% solution of potassium chloride E. Rheopolyglukin The right answer: A. 184. Woman, 18 years, is ill with diabetes mellitus during 5 years. She intakes 36U of insulin a day. The state became worse sharply after start of pneumonia : thirst increased, abdominal pain developed, nausea, vomits, somnolence increased considerably. Patient refused to meal in the evening, did not get evening dose of insulin, and she lost consciousness in the morning . Objectively: unconscious, a 134 skin is dry, turgor is reduced. Tongue is dry, breathing is noisy and deep, strong smell of acetone from a mouth. Body temperature - 36,6 °С, pulse - 100/min, BP - 90/50mmHg. In urine positive reaction on an acetone is present. Glycemia 33mmol/l. What is previous diagnosis? A. Ketoacidotic coma B. Hyperosmolar coma C. Lactatacidemic coma D. Hepatic coma E. Cerebral coma The right answer: A. 185. Patient was hospitalized unconscious. He is ill with diabetes mellitus during 5 years. He intakes prolonged action insulin in dose 24U in the morning and 18U in the evening. Suddenly he lost consciousness. A skin is moist, tone of muscles of extremities is promoted. Tone of eyeballs is normal. Ps - 96/min, BP - 120/80mmHg. Cardiac tones are normal. Breathing is rhythmic. Language is moist. Менінгеальні symptoms are absent. What emergency therapy is need? A. Introduction of 40% glucose solution B. Introduction of short action insulin C. Introduction of 4% sodium hydrocarbonatis solution D. Introduction of hydrocortizone E. Introduction of adrenalin subcutaneously The right answer: A. 186. Whatever feature is not typical for myocardial infarction in patients with diabetes mellitus? A. It is accompanied by development of hypoglycemia B. Q-miocardial infarction C. Absent ECG-signs of myocardial infarction D. Absent increase level of markers of myocardial necrosis 135 E. Absent development of complications of myocardial infarction The right answer : A 187. What is the typical sign of Kimelstil-Wilson's syndrome? A. Nephrotic syndrome B. Strong hypertension C. Proteinuria, cilinderuria D. Leukocyturia E. Microalbuminuria The right answer : A 188. Specify the early marker of diabetic nephropathy: A. Microalbuminuria (30-300mg/day) B. Proteinuria (more than 300mg/day) C. Hypertension D. Decreasing of glomerular filtration rate less 10ml/min E. Glucosuria (more than 10g/l) The right answer : A 189. A family doctor had been called to a patient with type 1 diabetes mellitus. Apparently: the patient is adynamic (a condition of deep sleep), he comes out of it only under intensive stimulus, noisy respiration, the skin is dry, retracted abdomen. Members of family told, that patient’s condition has gone worse for last 2 days. What is the tactic of the family doctor here? A. *Urgent hospitalization of the patient to a hospital. B. To organize a treatment at home, in order to observe rehydratation. C. To prescribe to the patient plentiful drink. D. To seek for an endocrinologist’s consultation at the patient’s home (for taking the decision about his further 136 treatment). E. To make patient’s examination the next day. 190. A 72 years old man, has come to a surgeon in polyclinic complaining a pain in the lower extremities before which he felt numbness and crawling of bugs under the skin there over and at night especially having cramps in gastrocnemius muscle. Diabetes mellitus was found out during examination. What complications of diabetes mellitus can take place over here? A. *Diabetic neuropathogenesis. B. Diabetic macroangiopathogenesis. C. Diabetic microangiopathogenesis. D. Plexopathogenesis. E. Radiculopathogenesis. 191. A woman being 59 years old, has been suffering UNdiabetes mellitus for last 8 years, complains nail-bed hemorrhages on both first toes with no mechanical injury with raised fragility of nails along with the pain in lower extremities. What can be the most probable reason of these symptoms? A. *Diabetic microangiopathogenesis. B. Atherosclerotic angiopathogenesis of vessels of lower extremities. C. Fungal lesions of nail. D. The raised fragility of blood vessels due to the lack of vitamin РР. E. Pain connected with impaired innervations due to osteochondrous lumbosacral radiculitis. 192. In a woman of 23 gestational diabetes mellitus is developed during pregnancy. An endocrinologist prescribed diabetic diet and insulin therapy to her. Diabetes was in the stage of compensation during pregnancy. What may be tactics 137 of supervision after labour? A. *Insulin treatment is not necessary but making the glucose tolerant test annually. B. To continue treating with insulin. C. To discontinue insulin but to appoint medicine of sulphonilureas. D. To discontinue insulin but to appoint biguanides. E. To administer biguanides along with insulin. 193. At the man of 25 years who is ill with a diabetes of 8 years, illness complication, a coma: a condition heavy, a skin dry, turgor skin it is lowered, an acetone smell, breath Kussmaul’s, blood pressure 105/60, pulse-116/minute. What kind of a coma can be suspected? A. Lactacidemia B. Hyperosmolar coma C. *Ketoacidosis D. Hypoglycemic coma E. Brain coma 194.The ill M., 12 years old, after the carried windy pox in a heavy form there was the promoted thirst (drinks to 5 liters in a day), poliuriya , dry skin, pruritus. The grandfather of patient is ill diabetes mellitus. Level of glycemia – 15 mmol/l, acetone of urine +. To define a diagnosis. A. * Diabetes mellitus is type 1, stage of decompensation, ketosis ; B. Diabetes mellitus type 1, subindenifications; C. Diabetes mellitus 2 types, easy motion; D. Diabetes mellitus type 2, to middle weight, stage of indemnification; E. Diabetes mellitus type 1, heavy form, ketoacidosis. 138 A. B. C. D. E. 195.A 30-year-old nursing student presents with confusion,sweating, hunger, and fatigue. Blood sugar is noted to be 40 mg/dL. The patient has no history of diabetes mellitus, although her sister is an insulin-dependent diabetic. The patient has had several similar episodes over the past year, alloccurring just prior to reporting for work in the early morning. On this evaluation, the patient is found to have high insulin levels and a low C peptide level. The most likely diagnosis is Factitious hypoglycemia Reactive hypoglycemia Early diabetes mellitus Insulinoma Diabetes insipidus 196. In hospital the woman of 25 years without consciousness is delivered. Mother of the patient has told, that the daughter is ill with a diabetes since the childhood. Daily receives 40 UN insulin. Several days ago has autocratically stopped treatment. There was a thirst, weakness, appetite has decreased. In the morning the patient could not wake. A skin dry, turgor it it is lowered, eyeballs soft. Breath Cussmaul’s, is an acetone smell. A tachycardia 110 1 minute the BP of 100/45 mm hg, the stomach soft; a liver on 3 sm below edge of a costal arch. Sugar of blood of 17,5 mmol/l. Urgent therapy should include: A. *Fractional treatment by insulin in a dose 10 UN in an hour B. Dehydration - furosemid C. Insulin 100 UN IV D. Reopolyglucin to 1,0 l E. Prednisolon 60-90 ml IV 197. The woman of 27 years is found out without consciousness by a conductor in a train compartment. Objectively: a skin dry, tongue dry, a smell of fruit, a short 139 wind, breath deep, noisy. HR 120/ 1 minute, the BP 80/50 mm hg, the stomach is strained, at пальпации the painful. In blood leukocytes 17,0 . 10 9, glucose of 21 mmol/l, creatinin 0,28 mmol/l, рН 7,2. In the urine received catheter, reaction with nitroprussid sodium +++. Appoint most effective treatment in the given situation: A. Insulin of average duration of action 100 UN IV B. Insulin of fast action 100 UN IV C. Insulin of fast action in a dose 6-10 UN/hour IV D. 4 % a solution of bicarbonate of sodium 400,0 E. Glucagon 1,0 mg s/c or IM UNIT 2 1. Most common presentation of endemic goitre is: A. Hypothyroid* B. Diffuse goitre C. Hyperthyroid D. Solitary nodule E. Subacute thyroiditis 2. A 42-yr-old woman presents thyrotoxicosis with decreased Radioiodine uptake. Put diagnosis, please. A. Subacute thyroiditis* B. Thyrotoxicosis C. Hashimoto's thyroiditis D. Grave's disease E. Toxic multinodular goiter 3. A 41-yr-old patient presents with sore throat followed by midline tender swelling with pain, red hot skin on the anterior part of the skin. Put diagnosis, please. A. Subacute thyroiditis B. Acute thyroiditis* 140 C. Thyroglossal cyst D. Toxic thyroid nodule E. Chronic thyroiditis 4. A 72-yr-old-female was admitted to the hospital in unconscious status. Myxedema coma was diagnosed. What treatment can you prescribe? A. Hydrocortisone B. IV fluids C. T33 injections D. All of the above* E. None above them 5. Thyroid carcinoma associated with hypocalcemia is A. Follicular carcinoma B. Medullary carcinoma* C. Anaplastic carcinoma D. Papillary carcinoma E. All of them 6. A 42-yr-old woman presents subacute thyroiditis All the following are true of DeQuervan's Thyroiditis EXCEPT A. Pain B. Increased ESR C. Increased radioactive iodine uptake* D. Fever E. Increased lymphocyte level 7. Needle biopsy of solitary thyroid nodule in a 31-yr-old woman with palpable cervical lymph nodes on the same sides demonstrates amyloid in stroma of lesion. Likely diagnosis A. Medullary carcinoma thyroid* B. Follicular carcinoma thyroid C. Thyroid adenoma D. Multi nodular goiter 141 A. B. C. D. E. A. B. C. D. E. A. B. C. D. E. DeQuervan's Thyroiditis 8. A 52-yr-old woman presents to her GP for fatigue, depression and weight gain. She also complains of constipation and poor memory. On examination, she has a smooth peaches and cream complexion, thin eyebrows and a large tongue. What treatment of thyroid disorder can you recommend? Diuretics Propanolol Carbimazole Thyroxine* Propylthiouracil 9. 63. A 17-yr-old girl presents with lethargy and weight gain. She is depressed and sensitive to cold. She would like something done about her excessive weight. The other children abuse her. Prescribe investigations, please. Dexamethasone suppression test CAT scan of the skull Water deprivation test T3, T4 and TSH* Serum TRH 10. A previously well 71-yr-old woman has been noticed by her daughter to be increasingly slow and forgetful over several months. She has gained weight and tends to stay indoors with the heating even in warm weather. Put diagnosis, please. Cerebral malignancy Hashimoto's thyroiditis Subacute thyroiditis Hypothyroidism* 142 E. Graves' disease 11. Habitant of the Ivano-Frankivsk region of age of 28 years, pregnancy. In marriage 7 years, there were a few pregnancies and all ended by fetal loss. During a review the goiter of 1 B grade is determined, thyroid gland is soft. During the inspection of plasma of blood, the level of thyriod hormones are normal. Middle iodineuria at the habitants of this region is 50 mkg/l. The treatment of iodine deficit state first of all usually include: A. Iodized salt B. Consumption of meal rich in iodine C. L-thyroxinum 75 mkg per day D. IodBalance 200 mg per day E. Antistruminum 1 tab. 2 times per a week 12. Pregnant women, 28 years old, habitant of the Carpathian region. In anamnesis – 10 years ago was treatment of endemic diffuse nontoxic goiter of 1 B grade. Objective review: thyroid gland enlarged 2 grade, smooth, normal texture and homogenous. Periorbital puffiness, Stellwag's, Dalrymple's, Rosenbach's signs are negative. Level of thyroid hormones are normal. What are principles of treatment? A. Potassium iodide 200 mg per day B. Potassium iodide 150 mg per day C. Consumption of meal rich in iodine D. Only iodized salt E. Diet, rich in iodine and potassium iodide 200 mg per day diet, rich in iodine 13. Patient U., 26 years old, complains of swallowing, weakness, feeling of a “loop” round the neck. In anamnesis – 10 years ago was treatment goiter. Objective review: thyroid gland enlarged III grade, normal texture, homogenous. Periorbital puffiness, Stellwag's, Dalrymple's, Rosenbach's 143 A B C D E signs are negative. Result of ultrasound examination: a thyroid gland is increased, total size is 36 cm³, echogenicity is not changed. Level of thyroid hormones are normal. Median of iodine excretion with urine 100 mkg/l. Substantiate diagnosis: A. Nodular goiter B. Endemic diffuse nontoxic goiter of 3 grade C. Diffuse nontoxic goiter of 2 grade D. Sporadic diffuse nontoxic goiter of 2 grade E. Diffuse euthyroid goiter of 2 grade 14. A previously healthy 19 year old female complains of a fast heart rate, weight loss, and fatigue over the past 2 months. Her family history is significant for a grandmother and aunt with Hashimoto thyroiditis. Objective review: temperature - 37,0° C, pulse - 110 beats/min, blood pressure - 120/50, cardiac tones normal, skin is moist, warm, a mild tremor. Thyroid gland enlarged, smooth, normal texture and homogenous. High level of T3 and T4, undetectable TSH. Thyroid stimulating immunoglobulin assay is positive. Your diagnosis? Diffuse toxic goiter Hashimoto thyroiditis Diffuse nontoxic goiter Nodular goiter Sporadic diffuse nontoxic goiter 15. A 24 years old woman with chronic schizophrenia is referred for evaluation of abnormal thyroid function tests after experiencing an acute exacerbation of psychosis. She has no prior history of thyroid dysfunction but several family members have been affected by autoimmune thyroid disease. Over the preceding three months she has lost 8 kg and has noted insomnia, tachycardia, heat intolerance, and irregular menses. Physical examination demonstrates an anxious and restless young 144 woman with resting tachycardia (rate 120 beats/min), tremor of extended fingers, bilateral lid lag and “stare”, warm moist skin, brisk reflexes, and an impalpable thyroid gland. Thyroid function tests are as follows: free T 4 - 4,2 ng / dL (normal 0,8 – 2,0 ng / dL), TSH < 0,01 μU / L (normal 0,4 – 4,5 μU / L), thyroglobulin – 5 ng / mL (normal < 40 ng / mL). Which diagnosis is most likely? A Nervous exhaustion B Adenoma of thyroid gland C Struma ovarii D Diffuse toxic goiter E Nodular goiter 16. Patient A., 38 years old. Operated a diffuse toxic goiter. After the operation the state a patient became worse, palpitation, shortness of breath, diarrhea, fever. Objective review: fever is 38.6°С, pulse - 160 beats/min, blood pressure 85/40. Preliminary diagnosis: A. Thyrotoxic crisis. B. Thyrotoxic hepatitis C. Pneumonia D. Adrenal crisis E. Acute pancreatitis 17. The patient M. is 55, appealed with complaints about the increase of thyroid. From the inspections the diffuse increase of thyroid is exposed to 2 grade, function is abnormal: high level of T3 and T4, undetectable TSH. The treatment of diffuse toxic goiter first of all usually include. Choose agent, which inhibits synthesis of thyroid hormones: A. Thiamazole B. Potassium perchlorate C. Potassium iodide D. Iopanoic acid 145 E. Dexamethasone 18. A 50 year old man presents with enlargement of left anterior neck. He has noted increased appetite over past month with no weight gain, and more frequent bowel movements over the same period. Physical examination: temperature of 37,4 °С, the heart rate is 92 and the blood pressure is 110/50. There is an ocular stare with a slight lid lag. The thyroid gland is enlargement of 3 grade and asymmetric to palpation, nodule in left lobe of the thyroid gland. Result of ultrasound examination: a thyroid gland is increased, total size is 40 cm³, there is a 3 x 2.5 cm firm nodule in left lobe of the thyroid. Level of thyroid hormones are abnormal: high level of T3 and T4, undetectable TSH. Which diagnosis is most likely? A Adenoma of thyroid gland B Nodular goiter 3 grade, thyrothoxicosis C Multinodular goiter D Diffuse toxic goiter E Nodular goiter 19. Patient V., 26 years old, during 3 months was ill diffuse toxic goiter III, to treatment – thyrozol 30 mg per day. After grippe the patient complaints: palpitations, tremor, high fever, diarrhea. Objective review: thyroid gland enlarged, smooth, normal texture homogenous. Abdominal pain, vomiting. Tachycardia – 140 beats/min, blood pressure 140/50. Fever – 40 ° C. Establish your diagnosis? A. Diffuse toxic goiter in decompensation: Thyrotoxic crisis. B. Nodular goiter C. Toxic goiter in pregnancy D. Adenoma of thyroid gland E. Diffuse toxic goiter in compensation 20. A 53 year old woman came to the polyclinic. She had no symptoms but gave a history of a lump in her neck 146 being noticed by her primary care physician during a routine ‘well-woman’ check. There was no family history of thyroid disease and she had a blameless past medical history. She had not noticed any change in her voice, or difficulty swallowing or breathing. Examination was entirely normal, except thyroid gland enlarged II grade, normal texture, homogenous for a 3 × 2 cm single nodule in the left lower thyroid gland. Blood tests showed that her total T4, free T3, TSH - normal, and thyroid autoantibodies were not present in serum. What is the probable diagnosis? A. Nodular goiter B. Endemic diffuse nontoxic goiter C. Diffuse nontoxic goiter D. Sporadic diffuse nontoxic goiter E. Diffuse euthyroid goiter 21.Which of given below symptoms is characteristic for hypothyrosis: a. bradycardia b. body mass lessening c. increased basal metabolism d. all mentioned above 22.Name of the most dangerous complication of the thyrotoxic crisis: a. acute violation of the cerebral blood circulation b. acute renal insufficiency c. acute cardiac and coronary insufficiency d. acute respiratory insufficiency 23.Name a thyrostatic remedy: a. thyroidine b. thyrocomb c. mercazolil d. thyroxin 147 e. thriiodothyronine 24.What violation of the cardiac rhythm is characteristic for a thyrotoxic heart? a. respiratory arrhythmia b. sinus bradycardia c. paroxysmal ventricular tachycardia d. ventricles’tremor e. glimmering arrhythmia 25.Woman of 42 came to the polyclinic for the primary examination. Her weight was decreased. She had tremor of the upper extremities. The woman was excited, dissatisfied with everything and she behaved herself outrageously. Suddenly she lost consciousness. Tachypnea and cardiac rhythm violation were evident. Her arterial pressure was 190/110 mm, her pulse was 160 beatings per minute, her temperature was 38.8 C. The most veritable diagnosis is: a. thyrotoxic crisis b. hypothyroid crisis c. hypertensive crisis d. sympathoadrenal crisis 26.Patient M. of 39 years old having a thyrotoxic goitre the second day after the subtotal resection of the thyroid gland the temperature rose up to 41 C, the puls was 180 beatings per minute, the glimmering arrythmia has developed, the tremor of the extramities appeared. How could the development of the thyrotoxic crisis have been prevented? a. to remove the phenomena of the thyrotoxicosis before the operation b. to refuse from the operative treatment c. to prescribe iodine preparations d. to prescribe thyrostatic preparations during the postoperative period 148 27.The thyrotoxic crisis can be developed as a result of: a. inadequate treatment of thyrotoxicosis b. addition of an infectious disease if patients are suffering from the thyrotoxicosis c. cancellation of taking tyrostatic preparations d. operative intervention e. all mentioned above 28. A solitary node in the left lobe of the thyroid gland of a 25 year old man was detected. Which of the investigations mentioned below is to be made in the first turn? a. T3,T4 determination b. TTH determination c. fine-needle aspiration biopsy of the node d. the USD of the thyroid gland 29. Afther the subtotal resection of the thyroid gland the patient began to suffer from increasing weakness, drowsiness and fatigability. Oedema of eyelids appeared. Hypothyrosis was suspected. What is it necessary to determine? a. TTH b. T3, T4 c. the level of antythyroid antibodies d. the level of the radioactive iodine absorption 30. He patient P, 60 years old, was brought into the therapeutical department with pneumonia. Has suffered from the hypothyrosis for almost 10 years. Took L-thyroxin in a dose of mkg. The last month stopped taking preparations. It was detected that her general state is grave. Her reflexes are inhibited and the oedemas of the face and extremities are well seen. Her body’s temperature is less than 34 C. she has hypotonia and weakened respiration in the lungs. There is crepitation on the left. Your diagnosis is: 149 a. b. c. d. hypothyroid coma thyrotoxic crisis sympathoadrenal crisis meningoencephalitis 31. Patient (woman), 25 years, plans pregnancy and appealed to endocrinologist to recommendations in relation to the use of iodine with a meal. Specify a daily necessity in an iodine for the biosynthesis of thyroid hormones for adult healthy person: A. 100-150μg B. 14-17mg C. 1-2g D. 98-104ng E. 123-228μmol The right answer: A. 32. Woman, 27 years, appealed to endocrinologist with endemic goiter 2 degree, euthyrosis. What is the tactic of treatment in this patient? A. To prescribe L-thyroxin in complete dose during 612 months, after diminishment of size of gland - maintenance dose of L-thyroxin, if it's normalization of gland function iodated salt B. To conduct preoperative preparation with iodides and follow radical surgical treatment C. To prescribe medical dose of iodine-131 D. Complex treatment by polyvitamins with microelements The right answer: A. 33. Woman 27 years, appealed to the doctorendocrinology with a diagnosis endemic goitre of a 2 item, еутиреоз. What at this sick does it follow to choose tactic of treatment during pregnancy? 150 A. L-thyroxine B. Thyamazol C. Metizol D. Merkazolil E. Lugol's solution The right answer: A. 34. Enlargement of thyroid gland was defined in woman, 25 years, during examination. She lives in Carpatian mauntain region. Abnormalities in internal organs was not fixed. Thyroid gland is diffusely enlarged to II degree, it's softly-elastic, smooth, painless. Levels of thyroid hormones are normal. US: thyroid gland is enlarged, echogenicity is not changed. Note the most credible diagnosis: A. Endemic diffuse euthyroid goiter, II degree B. Autoimmune thyroiditis, euthyrosis C. Nontoxic diffuse euthyroid goiter, II degree D. Nodular goiter E. Chronic thyroiditis The right answer: A. 35. Patient (woman), 48 years, complains about irritability, hyperhydrosis, tremor of hands, palpitation, weight loss, progressing weakness, violation of sleep. She is ill during one year. Objectively: skin is moist, warm, thyroid gland is diffusely enlargement to ІІ degree, it has elastic consistency; positive Cocher, Grefe, Stelvag's symptoms. Pulse - 118/min, BP - 150/60mmHg. Cardiac tones are increased, systolic noise is on apex. Preliminary diagnosis: A. Diffuse goiter, II degree, thyrotoxicosis B. Chronic autoimmune thyroiditis C. Diffuse goiter, II degree, euthyrosis D. Diffuse goiter, II degree, vascular dystonia The right answer: A. 151 36. Asymmetric enlargement of thyroid gland (II degree) is exposed in patient, 27 years. The gland is sickly palpatory with irradiation into lower jaw. Temperature of body - 38°С. Patient was ill with flu week ago. Blood count increased ESR. Note preliminary diagnosis: A. Subacute thyroiditis B. Diffuse toxic goiter C. Toxic adenoma of thyroid gland D. Autoimmune thyroiditis E. Fibrous thyroiditis The right answer: A. 37. Patient (woman), 32 years, complains about permanent irritability, palpitation, pain in eyes, watering, weight loss - 10kg during 4 months. Objectively: skin is warm, moist, mild exophthalmos, Grefe, Cocher, Mebius' symptoms are positive. A thyoid gland is diffusely enlarged, unsickly. Pulse - 108/min, BP - 140/66mmHg. Shallow tremor of fingers on hands. Your diagnosis? A. Diffuse toxic goiter B. Acute thyroiditis C. Medular carcinoma of thyroid gland D. Neurasthenia E. Subacute thyroiditis The right answer: A. 38. What complication can after usage medicines of iodine during long period of time? A. Iodine-induced thyrotoxicosis B. Hypothyroidism C. Autoimmune thyroiditis D. Diffuse-nodular goiter E. All it's true 152 A. B. C. D. E. A. B. The right answer : A 39. What from medicines is the most effective for treatment of sterility in patients with hypothyroidism? A. Thyroxin B. Vitamin E C. Progsteron D. Estrogens E. Chorionic gonadotropin The right answer: A 40. What from these is the most typical for endemic goiter? A. Low excetion of iodine with urine (less 50mg/day) B. Promoted absorption (more 50%) by the thyroid gland І131 during 24 hours C. Normal level of total Т4 and reduced free Т4 D. Reduced level of TSH E. Promoted level of АB to ТPO The right answer: A 41. A 62-yr-old woman presents with stiff joints, myopathy and constipation. Plain radiographs reveal a right calculus and, evidence of osteitis fibrosa cystitica. Put diagnosis, please. Parathyroid adenoma* Pheochromocytoma Prolactinoma Corticosteroma Follicular carcinoma thyroid 42. A 49-yr-old woman complains of perioral paraesthesia, carpopedal spasm and generalised seizures. Put diagnosis of metabolic disturbances, please. Hypokalaemia Hyperkalaemia 153 C. Hypocalcaemia* D. Hypercalcaemia E. Hyponatraemia 43. On a routine blood examination a 42-yr-old woman is found to have very high serum calcium level She has complained recently of bouts of abdominal pain and recurrent UTI. On physical examination you find an enlarged thyroid gland. Put diagnosis, please. A. Aytoimmune thyroiditis B. Hyperthyroidism C. Parathyroid carcinoma* D. Secondary hyperparathyroidism E. Hypoparathyroidism 44. A 56-yr-old presented with following reports on a routine screen: Calcium-2.86 mmol/l, phosphate—0.8, ALP—111, PTH—raised, 25-OH vitamin D—low-normal. Put diagnosis, please. A. Primary hyperparathyroidism* B. Secondary hyperparathyroidism C. Tertiary hyperparathyroidism D. Hypoparathyroidism E. Hyperparathyroid with ectopiq PTH 45. A 36-yr-old woman presents with weight loss, muscular weakness, oligomenorrhoea, diarrhoea and blurring of vision. On examination, there is exophthalmos and proximal myopathy. Put diagnosis, please. A. Subacute thyroiditis B. Hypothyroidism C. Hashimoto's thyroiditis D. Graves' disease* 154 E. Follicular carcinoma 46. A 32-yr-old pregnant woman presents to her physicion with anxiety. On examination she is nervous woman with exophthalmos, warm peripheries and atrial fibrillation. What treatment of thyroid disorder can you recommend? A. Calciferol B. Propanolol C. Carbimazole D. Thyroxine E. Propylthiouracil* 47. A 49-yr-old woman presents with fever, tachycardia restlessness, hypertension and vomiting. On examination she has diffuse swelling of the thyroid gland and strabismus with diplopia. Put diagnosis, please. A. Subacute thyroiditis B. Graves' disease* C. Hashimoto's thyroiditis D. Thyroid carcinoma E. Hypothyroidism 48. A 24-yr-old young woman is warm, even when resting. She turns the central heating off, opens the windows and annoys her family. Her pulse rate is high and her skin is moist. Put diagnosis, please. A. Toxic adenoma B. Hyperthyroidism* C. Parathyroid carcinoma D. Hypoparathyroidism E. Follicular carcinoma 49. A 46-yr-old woman presents with tachycardia, 155 A. B. C. D. E. A. B. C. D. E. atrial fibrillation, sweating, double vision and swelling above her ankles. She has lid lag on examination. Prescribe investigations, please. Fasting blood glucose Neck USG ESG T3, T4 and TSH levels* Basal plasma protein 50. A female, 62 years old, suffers from pernicious anaemia for which she has received 1 mg cyanocobalamine intramuscularly every 3 month for the last 10 years. At a routine visit the patient is found with a puffy swollen face due to a non-pitting edema. Her skin is dry and cold, the heart rate is 55 beats/ min, her hair is sparse, and she complains of constipation and fatigue. A series of blood tests reveals the following: high levels of microsomal autoantibodies against the thyroid gland and autoantibodies against her parietal cells. The TSH concentration in the plasma is high, whereas the T3, T4 are low. The haematological variables are satisfying. What is the probable diagnosis? Hypothyroidism Cardiac insufficiency Pernicious anaemia Autoimmune (Hashimotos) thyroiditis, hypothyroidism Nephrotic syndrome 51. A 28 year old woman with recent tiredness and difficulty concentrating had experienced a decline in memory over the last several months. She also noted decreased frequency of bowel movements and an increased tendency to gain weight. She felt chilled without light sweater, even in warm weather. In the anamnesis - hypothyroidism in her mother and older sister. Objectively: She had a slightly puffy 156 A. B. C. D. E. face and her eyebrows were sparse, especially at the lateral margins. The thyroid gland is not palpated. Heart rate – 58 beats/min, BP is 100/60 mmHg. Tones of heart of low sonority. The deep tendon reflexes were normally contractive, but showed delayed relaxation. What laboratory tests would you order to evaluate this patient? A. Ultrasound thyroid gland B. Test for anti-thyroid antibodies (anti-thyroglobulin and anti-microsomal) C. Blood tests: levels T3, T4 and TSH D. ECG E. All methods 52. At patient M., 29 years old, the asymmetric increase of thyroid gland of the II stage is exposed, a gland is painful at palpation, pain irradiates in a left ear and upper jaw. Temperature of the body 38,7 °C. A week ago carried a tonsillitis. Most reliable that patient has: Fibrotic thyroiditis Diffuse toxic goiter Autoimmune thyroiditis Subacute thyroiditis Toxic adenoma of thyroid gland 53. The patient N., 26 years old complains of the increase of body mass, weakness, constipations, worsening of memory, some lowering of the voice and increase in fatigue. After delivery, she nursed the infant for 1 week. She then stopped nursing, but galactorrhea and amenorrhea continued for the next 5 months. In the anamnesis - she had menarche at age 16 and had regular periods. She married at age 24 and was not able to conceive. After receiving therapy for 7 months for treatment of extensive endometriosis, she became pregnant and delivered after 36 weeks' gestation. Her sister had autoimmune 157 thyroiditis. Objectively: a skin by touch is dry. A thyroid gland is enlarged II grade, smooth surface. Pulse – 58 beats/min, diminished sonority of tones of heart. The laboratory results: high levels of microsomal autoantibodies against the thyroid gland, the levels of TSH and prolactin were elevated, low levels of T3, T4. What is the diagnosis? A. Autoimmune thyroiditis, hypothyroidism B. Diffuse nontoxic goiter C. Autoimmune thyroiditis without violation of the thyroid gland function D. Hypothyroidism E. Syndrome of Van – Vik – Ross – Geness 54. A female, 72 years old in the grave condition hospitalization on emergency. Objectively: Temperature of the body - 35,8 °C. Blood pressure – 80/50 mmHg, pulse - 56 beats/min, diminished sonority of tones of heart, breathing - 12 /min. A skin is pale, cold, moderate edema of face and extremities. The hairs are liquid, thin, on a head areas of alopecia. Most reliable that patient has: A. Addisonic crisis B. Myxedema coma C. Lactacidotic coma D. Hypoglycemic coma E. Hypocalcemia 55. A patient B., 59 years old complains of the presence of nodule on the front surface of neck. Became ill 3 years ago. A nodule was enlarged in sizes, the timbre of voice changed, feeling of pressure appeared. Objectively: in the right lobe of thyroid a nodule is palpated 5 cm in a diameter, painless. The functional state of thyroid is not changed. What is the diagnosis? A. Nodular euthyreoid goiter 158 B. C. D. E. A. B. C. D. E. Nodular hyperthyroid goiter Cancer of thyroid gland Chronic lymphomatous Hashimoto thyroiditis Chronic fibrous Ridel's thyroiditis 56. At a patient M., 45 years old at the palpation of thyroid gland a nodule 2,0 х 3,2 cm is exposed in a left lobe, compacted, moderately painful during palpation. "Cold" nodule at scyntygrafia with I131. What inspection is most expedient for clarification of diagnosis? Ultrasound of thyroid gland Aspirational biopsy Determination of the TSH, T3 and T4 level in a blood Immunodetection Computed tomography scan (CT-scan) of thyroid gland 57. A patient S., 29 years old, for three days after thyroidectomy for thyroid gland cancer has complains of paraesthesia, muscle fibrillations, convulsions in extremities. What is the possible diagnosis? A. Secondary hypoparathyroidism B. Hypothyroidism C. Distant metastases D. Myeloma E. Primary hypoparathyroidism 58. A patient Z., 58 years old has been operated on for multinodular goiter III grade. For one month after the operation cramps in upper extremities had appeared, which persisted for 1-2 min. and accompanied with numbness in face. Cramps occur 2-3 times a day, commonly at a daytime. Pulse is 86 beast/min, rhythmic; blood pressure is 100/70 mmHg. Visceral organs are not damaged. Trousseau's, Hvostek's I and II, Schlesinger's, Hoffmann's symptoms are positive. What is the diagnosis? 159 A. B. C. D. E. A. B. C. D. E. Postoperative hypoparathyroidism Postoperative hypothyroidism Postoperative hypoparathyroidism and hypothyroidism Pseudohypoparathyroidism Epilepsy 59. A patient Q., 60 years old, having urolithiasis for 15 years, has coral kidney stone size 22x30mm in right kidney. Menopause has been obtained at 52 years. Laboratory tests: serum calcium - 2,95 mmol/L, serum phosphate - 0,6 mmol/L, creatinine, urea are normal. Bone X-ray examination: systemic osteoporosis, subperiosteal resorption of bones, cysts, spine deformation. Determine possible diagnosis: Pedjet disease Primary hyperparathyroidism Secondary hyperparathyroidism Postmenopausal osteoporosis Primary hypoparathyroidism 60.Determine operative treatment of the nodal goitre. a. node size is up to 1 cm b. node size is up to 2 cm c. node size is up to 3 cm d. senile age of the patient 61.What form of the hypothyrosis is the increasing level of the TTH characteristic for? a. primary b. secondary c. tertiary d. peripheral 62. The myxedema coma is characterized by: a. deep hypothermia b. hypothermia c. tachicardia 160 d. hypertension 63. The following frequency of the cardiac contractions is characteristic for an easy grade of gravity of the diffusive toxic goitre: a. up to 100 beatings per minute b. up to 120 beatings per minute c. more than 120 beatings per minute d. more than 140 beatings per minute 64. How much time must the thyrotoxicosis treatment take? a. 1 month b. 2-4 months c. 4-6 months d. 1-1,5 year 65. Which of the below mentioned preparations is used in the complex treatment of thyrotoxicosis? a. β-adrenoblockers b. ATE’s inhibitors c. statins d. diuretic preparation 66. With what TTH level of L-tyroxin shouldn’t be prescribed? a. up to 3 mu/litre b. up to 5 mu/litre c. up to 7 mu/litre d. up to 10 mu/litre 67. A 65 year old patient addressed the doctor complaining of tachycardia, uneven heart beating, shortness of breath, oedema of shins and feet, pain behind the sternum. Has been ill nearly 2 months. Detected: pulse 130 beatings per minute. The pulse is allorrhythmic with weak filling. Cardiac tones are muffled. 161 Glimmering take place. Her belly is soft, tachyarrhythmia, the liver +3 cm., the thyroid gland is diffusively enlarged up to the second grade. It is elastic. Tremor of the fingers when arms are stretched out. Grefe symptom is positive. Oedema of lower extremities takes place. Make the diagmosis. a. diffuse toxic goitre b. stenocardia of the strain c. hypertonic disease d. rheumatism 68. A 48 year old patient addressed the doctor complaining of eye balls oedema, cutting and lacrimarrhage in the eyes, progressive decreasion of the vision . Has been ill nearly 1 year. Detected: pulse 72 beatings per minute. The pulse is rrhythmic with normal filling. AP 115/80 mm. Pathlogy of all the rest organs and systems is absent. Oedema of eyelids, unilateral exophtalm takes place. Eye-chinks are not closed. Initial atrophy of the optic nerves on the ophtalmoscopy take place. Make the diagmosis. a. autoimmune ophthalmopathy b. thyrotoxicosis c. autoimmune thyroiditis d. hypothyrosis 69. Name factors which promote the development of the hypothyroid coma: a. stoping of taking thyroid preparations b. overcooling c. surgical intrusion d. all mentioned above 70. Excited state, paresthesias on the face, extremities, dyspnea, retrosternal pain, periodically convulsions of shin 162 muscules appeared in woman, 46 years, after subtotal resection of thyroid gland. HR- 89/min. Breathing rate 20/min. BP - 150/100mmHg. What medicines are indicated for prophylaxis of sharp exacerbation of state? A. Preparations of calcium B. Neuroleptics C. Tranquilizers D. Nitrates E. Beta-blockers The right answer: A. 71. Inspiratory dyspnea, constringent retrosternal pain, parasthesias on face and extremities appeared in woman, 52 years, after strumectomy on the occasion of the cancer of thyroid gland. What symptom appearance is the most credible? A. Chvostek's B. Babinsky's C. Stelvag's D. Mebius' E. Grefe The right answer: A. 72. Treatment with L-thyroxin in dose 50μg a day was prescribed in woman, 56 years, in connection with postoperative hypothyroidism. Level of TSH is normal. Constringent retrosternal pain with irradiation to left shoulderblade after physical activity appeared. AT -150/100mmHg. What changes of therapy are need after the removal of pain? A. To reduce the L-tyroxin dose B. To add beta-blockers C. To prescribe nitrates D. To replace L-thyroxin on triiodthyronin E. To prescribe thyrotom The right answer: A. 163 73. Woman, 36 years, with primary hypothyroidism intakes 50μg L-thyroxin a day. What inspection is indicated for estimation of efficacy of dose of medicine? A. Definition of TSH level B. Definition of T3 level C. Definition of T4 level D. Definition of thyroglobulin level E. Definition of АB to ТPО The right answer: A. 74. Elderly woman was hospitalized in the grave condition. Objectively: BP - 90/60mmHg, pulse - 56/min. Breathing rate - 12/min. Temperature of body - 35,8 °С. Skin is pale, cold, very dry. Hairs are liquid, thin, on a head are places of alopecia. Cardiac tones are dull, noises are absent. Stomach is enlarged, the peristalsis is not auscultated. Edemata are present on shins and round eyes. Glycemia - 3,2 mmol/l. What the most reliable pathology does patient have? A. Hypothyroid coma B. Addisonic crisis C. Hypoglycemic coma D. Hypercalciemic crisis E. Lactatacidotic coma The right answer: A. 75. Patient (woman), 52 years, complains about increase of body mass, weakness, constipations, worsening of memory. The indicated symptoms slowly grew during the last 1,5 year. Objectively: skin is dry, there is moderate edema of face and extremities, cardiac borders are enlarged, tones are dull, pulse - 66/min., BP -110/70mmHg, thyroid gland doesn't palpated. In a blood antibodies to thyroglobulin (+) and thyroid 164 peroxydase (+++) are present; level of TSH - 15,2 mU/l. US of thyroid gland: sizes are diminished, structure is heterogeneous. Define diagnosis: A. Autoimmune thyroiditis, hypothyroidism B. Autoimmune thyroiditis, euthyrosis C. Endemic goiter D. Subacute thyroiditis E. Diffuse untoxic goiter The right answer: A. 76. Patient (woman), 44 years, complains about general weakness, periodic pain in cardiac area, somnolence. She's ill during 3 years. She treated without an effect in a cardiologist and neurologist. Objectively: height 166cm, weight 70kg. Skin has ordinary color and it's moist by touch. A thyroid gland is diffusely enlarged (II degree), resiliently-elastic consistency, with a smooth surface, mobile. Pulse - 96/min. BP 140/70mmHg. Cardiac tones are impaired, systolic noise is present on the apex. Breathing is vesicular. Pathology of abdominal cavity organs isn't detected. Additional data: blood count and urine analysis are without pathology. US of thyroid gland: a gland is enlarged, a capsule is impact, a contour is clear, echogenicity of tissue is reduced, alternation of echopositive and negative areas. There is presence of antibodies to thyroid peroxydase. Define clinical diagnosis: A. Autoimmune thyroiditis B. Diffuse toxic goiter (II degree) C. Cancer of thyroid gland D. Fibrous thyroiditis E. Diffuse euthyroid goiter The right answer: A. 77. What level of blood pressure is the most typical for uncomplicated form of diffuse toxic goiter? 165 А.Increased systolic and decreased diastolic BP B. Increased systolic and diastolic BP C. Increased diastolic and normal systolic BP D. Diastolic BP is increased and systolic BP is decresaed E. Decreased systolic and diastolic BP The right answer : A 78. Specify the main mechanism of antithyroid effect of glucocorticoides: А. Inhibition of formation of thyroidstimulated antibodies B. Change of sensitivity of thyrotrophes to thyroliberin C. Decreasing of converting of Т4 to Т3 D. Decline of synthesis of steroid hormones E. All it's truel The right answer : A 79. Specify features of clinical displays of diffuse toxic goiter inelderly? А. Prevalence monosymptomatic forms with visceropathy B. Expressed ophthalmopathy C. Expressed neuro-psychologic disorders D. Expressed weight loss E. Frequent diarrhea The right answer : A 80.A 38-year-old female complains of palpitations, fatigue, and insomnia. On physical exam, her extremities are warm and she is tachy cardie. There is diffuse thyroid gland enlargement and proptosis. There is a thickening of the skin in the pretibial area. Which of the following lab values would you expect in this patient? A. Decreased TSH, increased total thyroxine B. Increased TSH, total thyroxine, total T3 166 C. Increased T3 uptake, decreased T3 D. Decreased TSH, normal T4 E. Decreased TSH, decreased T4 81. At the patient of 48 years the diffuse increase in a thyroid gland, an exophthalmus, weight loss at 4 kg for 2 months, a hyperhidrosis, the HR - 105/mines, BP - 140/70 mm hg a feces normal is revealed. What therapy is recommended in that case? A. A thyroxin B. Radioactive iodine C. A propranolol D. Solution Lugol’s E. *A mercasolil 82. At the patient of 56 years with a diffuse toxic goitre the vibrating arrhythmia from state of emergency - 110/mines is observed, an arterial hypertensia, BP – hg. What treatment along with mercasolil it is necessary to recommend 165/90 mm in this case? A. Radioactive iodine B. *A propranolol C. A novocainamid D. Verapamil E. A nifedipine 83. The man of 57 years marks palpitation, a hyperhidrosis, dream infringement., accruing weakness, loss in weight. It was treated concerning the IDH without effect. Objectively: t ° = 36,8°С, the HR 128 in minute, pulse 112 in a minute, arrhythmic, BP of 160/70 mm hg. The skin warm, damp. A tremor of fingers of the extended hands. Tones of heart are strengthened, systolic noise over apex. The Thyroid gland is not palpated. What from listed research is the most important for specification A. Ultrasonic research of a thyroid gland 167 B. A lipid spectrum of blood C. Test with the dosed out physical activity D. *Researches of level of thyroid hormones in blood E. Ultrasonic research of heart 84. The patient of 24 years who is ill with a diffuse toxic goitre within 5 years, complains of irritability, fatigue, tearfulness, a hyperhidrosis, palpitation. At survey: an insignificant exophtalm, shine of eyes, a positive symptom Grefe. The thyroid gland is diffusely increased to П-Ш st.; a tremor of fingers of hands, tongue. Рs - 100 in a minute. In a disease pathogenesis has leading value: A. Hypoproduction of a thyroid-stimulating hormone B. Hypoproduction of hormones of a thyroid gland C. Hyperproduction of antibodies to a thyreoglobulin D. Hyperproduction of antibodies to a microsomal thyreoglobulin E. *Hyperproduction of hormones of a thyroid gland 85. The patient of 27 years complains of irritability, insomnia, a hyperhidrosis, a weight loss at the kept appetite. About th: the skin warm, damp, weight of a body is lowered. The thyroid gland is diffusely increased, a uniform bilaterial exophtalm. A tachycardia, чсс 110 in minute. Level of the TSH in whey of blood below norm. Your diagnosis? A. An euthyroid goitre B. *A hyperthyrosis C. A thyroid gland cancer D. A hypothyroidism E. An acute thyroiditis 86. The woman of 52 years after surgery operation concerning a thyroid gland cancer had inspiratory dyspnea, compressing pains behind a breast, paresthesia in face, finitenesses. What occurrence of a symptom most is probable? 168 A. Babinsky B. *Hvostek’s C. Shtelvag’s D. Moebius E. Grefе 87. At the boy of 15 years last 3 months attacks of spasms of chewing muscles, muscles of hands, spasms muscles flexural. Spasms painful, symmetric. Positive symptoms Hvostec’s, Trusso. Calcium in blood of 1,8 mmol/l. The most probable diagnosis: A. Spasmophilia B. Epilepsy C. Hyperparathyroidism D. A tetanus Е * Hypoparathyroidis UNIT 3 A. B. C. D. E. 1. In persons with congenital adrenal hyperplasia resulting from inherited defects of adrenal steroid C-21 hydroxylase, excessive androgen production is the result of autonomous adrenal production of steroids autonomous pituitary production of ACTH *failure of production of an adrenal product necessary for negative feedback on pituitary ACTH secretion extraglandular formation from large amounts of nonandrogenic adrenal steroids positive feedback on pituitary ACTH secretion by abnormal adrenal products 2. A 41-years-old woman with obesity, dermal striae, and hypertension is referred for endocrinologic evaluation of possible cortisol excess. The woman receives a midnight 169 A. B. C. D. E. A. B. C. D. E. A. B. C. dose of 1 mg of dexamethasone; a plasma cortisol level drawn at 8 a.m. the next day is 386 nmol/L (14µg/dL). At this point in the evaluation the most appropriate diagnostic maneuver would be CT scanning of the pituitary gland abdominal scanning measurement of 24-h 17-hydroxycorticosteroid excretion in urine *measurement of a 24-h urine free cortisol a 2-day high-dose dexamethasone suppression test 3. 14. A 37 – year old man has symptoms of intermittent palpitation, anxiety, excess perspiration. examination reveals a blood pressure reading of 210/110 mm.hg and glucosuria. basal metabolic rate is + 50 %, fasting glucose 6,05 mmol/l, and an i/v histamine test produced a rise in blood pressure reading to 270/160 mm. hg, whereas a cld pressor test was negative. The best diagnosis is: Atherosclerotic heart disease Diabetic nephropathy *Pheochromocytoma Hypertensive disease Adrenal cortical carcinoma 4. A 48-year-old woman is diagnosed with Addison's disease (adrenal insufficiency), and treatment with Cortisol and fludrocortisone acetate (Florinef) is started. One month later she still complains of weakness and fatigue. Which of the following would suggest that the dose of Florinef should be increased? Ankle edema Increased blood pressure *Hyperkalemia 170 A. B. C. D. Hypernatremia E. Increasing pigmentation 5. A young (36-yr-old) man is found to have hypertension. Pheochromocytoma is suspected because of paroxysmal episodes of symptoms, with blood pressure elevations up to 210/120 mm Hg during the paroxysms. Which of the following findings would further support a diagnosis of pheochromocytoma? A. Central obesity, with a dorsal fat pad B. Hypoglycemia C. Low serum potassium level D. *Postural hypotension E. Marked fall in blood pressure with spironolactone treatment 6. A 42-yr-old woman complains of fatigue, weight loss and intermittent abdominal pain. She looks well tanned, although it is winter. She has postural hypotension and buccal pigmentation. What is the cause of fatigue? A. Anaemia B. Hypothyroidism C. Addison's disease* D. Hemochromatosis E. Tuberculosis 7. A 57-year-old man complains of fatigue, weight loss, and joint pains. Laboratory studies reveal that his serum Cortisol level is undetectable. Which of the following findings would suggest most strongly that he has primary adrenal insufficiency rather than secondary adrenal insufficiency due to hypopituitarism? Serum sodium and potassium levels are normal. Serum free thyroxine levels are low. Blood pressure is low, and decreases further when the patient 171 D. E. stands up. The patient has noted severe weakness and dizziness during upper respiratory infections. *The patient has noted some darkening of the exposed areas of his skin. 8. A 26- year old female patient presents with hypertension, Na – 146 meq/I, K – 1,4 meq/l, normal serum creatinine most probable diagnose is: A. pheochromocytoma B. *Conna’s syndrome C. Cushing’s syndrome D. renal parenchymal disease E. renal vascular disease 9. A 68-yr-old woman presents to her GP with weight loss and depression. On examination she is noted to have buccal pigmentation and pigmented scars. She appears dehydrated. Her BP is 100/60 mm Hg. What treatment can you recommend? A. Desmopressin B. Long-term replacement glucocorticoids and mineralocorticoids* C. Carbimazole D. Propylthiouracil E. Thyroxine 10. A 27 year old lady has put on weight (16 kg over a period of 3 years), and has oligomenorrhoea followed by amenorrohoea for 8 months. The blood pressure is 170/100 mm of Hg. Which of the following is the most appropriate investigation? A. Serum electrolytes B. Plasma cortisol* C. Plasma testosterone and ultrasound evaluation of pelvis 172 D. T3, T4 and TSH E. Plasma prolactin 11.A 55 year old woman presented with classical clinical features of Cushing's syndrome including hypertension, diabetes mellitus, central obesity and easy bruising. She denied previous depression or heavy alcohol intake and was not receiving any steroid containing medication. Questions: 1. List the causes of Cushing's syndrome and discuss what initial investigations you would perform? 2. In the light of these results, what additional tests would you perform? 3. What is the likely cause of this patient's recurrent Cushing's and what further investigations should be performed? 12. A 64 year old woman was seen in the Endocrine clinic because she had noticed increasing hirsutism for 6 years but worse over the last year. She had also noted increasing fatigability, and some left-sided abdominal pain. Her voice had become deeper but she had a normal appetite and no weight loss. She had a past history of mild hypertension and had had one child. She smoked 10 cigarettes a day. Examination revealed marked hirsutism with temporal recession of the hairline and a beard. The blood pressure was 200/110 mmHg. Questions: 1. What initial investigations would you perform? 2. Following the receipt of these results, what further investigations would you perform? 13. Patient M., 34 years old, presents with complaints of instant weight gain, back pain, headache, decreased potency, periodical dryness of the mouse, sense of fatigue. 173 Objective status: height - 169 cm, weight - 102 kg. Fat tissue localized mainly on the chest, belly and hips. On the shoulders, belly and hips are discovered striae with a dark blue color. Cordial tones were muffled, blood pressure 170/95 mmHg. Fasting blood glucose – 6,8 mmol/L. Questions: 1. Substantiate diagnosis. 2. Principle of treatment approach. 14. Patient B., 46 years old, complains of onsets of high blood pressure, which appear spontaneously, accompanied by headache, disturbance of eyesight, sense of anxiety and fear, trembling of extremities, overwhelming sweating, irritability, shortness of breath, nausea, vomiting, pain over the belly and chest, pallor or ruddy face. All of which rapidly end with profuse urination and sweating. The history of this illness is that all symptoms presented within 6 months. Questions: 1. What is the most likely diagnosis? 2. Principle of treatment approach. 15. A 35 year old man presented with elevated blood pressure (188/112 mmHg, seated) at a yearly physical exam. Previous exams noted blood pressures of 160/94 mmHg and 158/92 mmHg. On questioning, he admitted episodes about twice a month of apprehension, severe headache, perspiration, rapid heartbeat, and facial pallor. These episodes had an abrupt onset and lasted 10-15 minutes. Objective status: 30 min after the initial blood pressure measurement, the seated blood pressure was 178/110 mmHg with a heart rate of 90 beats/min. The blood pressure after 3 min of standing was 152/94 mmHg with a heart rate of 112 beats/min. The optic field of vision showed moderately narrowed arterioles with no hemorrhages or exudates. Result of laboratory tests: routine hematology and 174 chemistry studies were within the reference ranges and a chest film and EKG were essentially normal. Questions: 1. What is the most likely diagnosis? 2. Principle of treatment approach. 16. Patient U., 64 year old presented to the emergency department after experiencing generalized weakness, nausea, constipation, and an intermittent cough for about 3 weeks. He subsequently admitted anorexia and weight loss (approximately 10 kg) over the past month. The patient denied fever, chills, chest pain, abdominal pain, diarrhea, and dysuria. He had begun treatment with antibiotics 1 week prior to presentation for a sinus infection. Objective status: temperature 36.2° C; pulse -106 beats/min, irregularly irregular; respiratory rate - 20 / min; blood pressure -138/84 mmHg. The patient was thin with no palpable lymphadenopathy, and no edema or cyanosis. Laboratory studies: Chemistry - sodium: 108 mmol/ L (normal 133 – 145 mmol/L), potassium: 5,1 mmol/L (normal 3,5 – 5,0 mmol/L), chloride: 76 mmol/L (normal 96 – 108 mmol/ L). Fasting plasma glucose – 2,4 mmol/L. Cortisol: Baseline - 7.6 μg/dL, 30-min - 7.9 μg/dL (normal 6.0-28.0 μg/dL), 60-min μg/dL - 8.6 μg/dL (normal >20 μg/dL). Questions: 1. What is the most likely diagnosis? 2. Principle of treatment approach. 17. Patient D., 38 years old, presents with complaints of general weakness, weight loss, dizziness, nausea, vomiting, diarrhea, pain in the cord. Objective status: height - 163 cm, weight -53 kg. Skin grayish – brown. Tones were rhythmical, muffled. Blood pressure – 90/60 mmHg, pulse – 90 beats/min. Questions: 1. Describe the methods used to diagnose chronic 175 adrenal insufficiency? 2. Principle of treatment approach. 18. Patient R.H., 35 year old husband and father of three children. Over the past six months he has experienced headaches and palpitations of increasing frequency and severity. In addition, he has had periods of intense anxiety and panic attacks. His wife has noticed that R.H.’s face is often pale and that he sweats more. Upon examination by his physician, R.H. was found to be severely hypertensive and in atrial fibrillation. R.H.’s physician suspected that he might have a pheochromocytoma and ordered a battery of tests, which confirmed his diagnosis. Questions: 1. Name the hormones secreted by the adrenal medulla. 2. How are hormones of adrenal glands classified? 3. Why are patients with a pheochromocytoma often hyperglycemic? 4. Describe the methods used to diagnose a pheochromocytoma. 5. How are pheochromocytomas usually treated? 19. A 27 year old woman presents with depression, insomnia, increased facial fullness and recent increase in facial hair. She had also had an episode of depression and acute psychosis following uncomplicated delivery of normal baby boy 9 months previously. Her menses have been irregular since their resumption after the birth (she is not breast-feeding). Objective status: the heart rate was 90 beats/min and the blood pressure was 146/110 mmHg. Her face was puffy with an increase in facial hair and ruddy complexion. There was no obesity, peripheral wasting, or striae. Laboratory studies: serum electrolytes, white cell count, and hemoglobin and hematocrit 176 1. 2. 1. 2. were all within normal limits. Questions: What do you think the patient's primary problem is? What laboratory tests would you order to evaluate this patient? 20. A 40 year old woman presents with a 6 month history of increasing fatigue. For the past three months she had suffered recurrent upper respiratory infections, poor appetite, abdominal cramps and diarrhea. During this time, she lost 11 kg. She had also noted joint pains, muscle weakness, a dizzy spells following exercise, and she had not menstruated for the past 3 months. Objective status: height - 165 cm and weight – 46 kg. Her heart rate was 86 beats/min and the blood pressure was 120/65 mmHg when she was supine. After one minute of quiet standing, the heart rate was 120 beats/min, the blood pressure was 90/58 mmHg, and she became dizzy. Questions: What do you think the patient's primary problem is? What laboratory tests would you order to evaluate this patient? 21.On the CT of the 38 years old patient a round formation with the diameter to 25 mm was found in the right adrenal. The patient has the stable arterial hypertension 180/110, shin oedematose. Trophic skin disorders and stretchings are absent. Potassium level in the blood is 2.8 mml/l. Formulate a preliminary diagnosis. a. pheochromocytoma b. Itsenko-Kushing’s syndrome c. Konn’s syndrome d. Itsenko-Kushing’s disease e. innate hyperplasia of adrenic cortex 22.Patient, 28 years old, moonlike plethoric face, 177 dysplastic adiposity with predominant fat fibre on the body, stretchings on the front abdominal wall. On the USD and CT there is a solitary formation in the left adrenal with the diameter to 40 mm. Formulate a preliminary diagnosis. a. pheochromocytoma b. Itsenko-Kushing’s syndrome c. Konn’s syndrome d. Itsenko-Kushing’s disease e. innate hyperplasia of adrenic cortex 23.Stretchings appearance under the syndrome of Itsenko-Kushing’s is conditioned by: a. hemorrhages b. scratching c. trophic skin modifications d. venous excrescences e. melatonin accumulation 24.Medicinal therapy of primary hyperaldosteronism consists in prescribing: a. veroshperone 200-400 mg/day b. hydrochlorothiazide 100-200 mg/day c. indapamide 2,5-5 mg/day d. loratadine 25-50 mg/day e. furosemide 100-200 mg/day 25.At the lesion of adrenals by a tubercular process with development of insufficiency of their function this insufficiency is defined as: a. primary b. secondary c. tertiary d. peripheral e. any 26. For the primary hyperaldosteronism are 178 charecteristic: a. high levels of potassium and aldosterone in the blood b. low levels of potassium and aldosterone in the blood c. high level of potassium and low aldosterone in the blood d. low level of potassium and high aldosterone in the blood e. levels of potassium and aldosterone do not change 27.To diagnose Konn’s syndrome it is possible to make: a. small dexamethasone test b. test with furosemide c. large dexamethasone test d. test of stimulation ACTH e. test of prednisolone oppression 28.To the methods of visualization of adrenals do not belong: a. scintigraphy b. punctural biopsy c. USD d. CT e. MRT 29.Chronic adrenal insufficiency is accompanied: a. by the decrease of hyperglycemia and natremia b. by the increase of hyperglycemia and natremia c. by the decrease of hyperglycemia and increase of natremia d. by the increase of hyperglycemia and decline of natremia e. levels of sodium and potassium do not change 179 30.Arterial hypertension develops with all adrenal illnesses, except: a. pheochromocytoma b. Kushing’s syndrome c. Konn’s syndrome d. adrenal insufficiency e. Kushing’s disease 31.For a patient with the kidney form of unsaccharine diabetes it is expedient to apply following diuretic in therapy: a. furosemide b. spironolactone c. hydrochlorothiazide d. torasemide e. mannite 32. Woman, 39 years, complains about headache, weakness and parasthesias in extremities, polyuria. Objectively: cardiac tones are dull, HR - 94/min, BP 190/105mmHg. Glycemia - 5,5 mmol/l, sodium of plasma 148 mmol/l, potassium of plasma - 2,7 mmol/l. In urine: relative density - 1012, protein -0,1 g/l, pH is alkaline, WBC3-4 in area vision, RBC - 2-3 in area visiov. Specify the most credible diagnosis: A. Primary hyperaldosteronism B. Amyloidosis C. Diabetes insipidus D. Chronic glomerulonephritis E. Hypertension The right answer: A. 33. Patient, 36 years, is ill during 1-1,5 years. He complains about weakness, anorexia, nausea. Weight loss is 10 kg during last year. Skin is hyperpygmented. Folds of skin mammillas are dark. Pulse - 60/min. BP -80/50mmHg. There is 180 high level of ACTH in blood plasma. Your previous diagnosis? A. Chronic adrenal insufficiency B. Diabetes mellitus C. Hypopituitarism D. Chronic gastritis E. Chronic hepatitis The right answer: A. 34. Pain in a lumbar area with irradiation in left leg appeared in man, 28 years. Objectively: height - 186cm, weight - 92 kg. BP - 170/100mmHg, pulse - 84/min., Breathing rate 18/min. Distributing of fatty tissue on upper type, for except extremities. Purple strias present on stomach and thighs. X-ray: ostoporosis of vertebral bodies, compression fracture LIV. In blood: RBC - 5,5х1012/l, НЬ -190 g/l, WBC - 9х109/l. Calcium -3,3 mmol/l. What is the most credible reason of fracture? A. Cushing's syndrome B. Addison's disease C. Compensatory policytemia D. Primary hyperparathyroidism E. Alimentary-constitutional obesity The right answer: A. 35. Patient (woman), 46 years, much was in the sunshine in rest period at seaside, without regard to prohibition of doctors. In this time she weight loss, but didn't draw attention that "a tan sticks very quickly" and also pigmentation of mamillas and smoke-coloured spots on the mucus shell of lips appeared. Define diagnosis: A. Аddison's disease B. Medicinal disease C. Cancer of mammary glands D. Mastopathy 181 E. Allergic dermatitis The right answer: A. 36. Patient (woman), 36 years, complains about frequent episodes of increase of BP to 260/120mmHg during the last months. There are not patients with hypetension in family . Intensive headache, palpitation, concern appear in the morning. BP 260/120mmHg, HR - 115/min. During attack: glycemia - 7.3 mmol/l, leukocytosis, expressed polyuria appeared after decreasing of blood pressure. Prominent the most effective antihypertensive drugs for this patient: A. alfa-blockers B. beta-blockers C. Calcium antagonists D. ACE inhibitors E. Diuretics The right answer: A. 37. Patient (woman), 42 years, after the physical overload lost consciousness. BP 40/20mmHg. In anamnesis she is ill with bronchial asthma during 5 years, constantly she uses glucocorticoides. Last week she didn't use medications. Objectively: skin has ordinary color and normal humidity, cardiac tones are dull, pulse 100/min, weak filling, rhythmic. Glycemia - 3,0 mmol/l, sodium - 117 mmol/l, potassium - 6,0 mmol/l. Set a previous diagnosis: A. Acute adrenal insufficiency B. Pheochromocytoma C. Cardiogenic shock D. Asthmatic status E. Hypoglycemic coma The right answer: A. 38. Woman, 35 years, felt dizziness, weakness after the 182 emotional overstrain. There are anorexia, weight loss, nausea, diarrhea in anamnesis. Blood pressure 60/40mmHg in vertical position. Pulse 102/min, small, rhythmic. Glycemia 3,1 mmol/l. Hyponatriemia. Hypercalciemia. Hyperpigmentation of skin. Previous diagnosis: A. Bleeding in the abdominal region B. Vascular dystonia C. Pheochromocytoma D. Chronic adrenal insufficiency E. Diabetes mellitus, hypoglycemia The right answer: A. 39. A patient, 36 years old, has been suffering from arterial hypertension for 4 years. Physician prescribed different hypotensive medicines without result. Conn’s syndrome was diagnosed after supplementary examination. What are basic clinical and laboratory symptoms of this syndrome? А. *Resistant arterial hypertension, hypokaliemia, nycturia, В. Periodical increase of blood pressure after stressful situation, normokaliemia, С. panic attacks, frequent urination after they, normal level K+ in blood, D. Palpitations, nervousness, headache, good effect from sedatives, Е. Arterial hypertension, good effect after treating by hypotensive medicines. 40. A 43 years old woman, complains us about elevated blood pressure, including paroxysmal high blood pressure, palpitations, anxiety often resembling that of a panic attack, headaches often, pallor sometime, diaphoresis during and after paroxysms. Pheochromocytoma was suspected. What are basic symptoms of this disease? 183 А.*Catecholamines (adrenalin, noradrenalin), vanililmandelic acid in urine, B. Increased levels of potassium, sodium, chlorides in blood, C. General cholesterol, triglycerides, beta lipoproteins found in blood, D. Increased levels of calcium, phosphates in blood, calcium in urine, Е. Increased levels of 17-OKS in urine and ACTH in plasma. 41. A 42-year-old alcoholic male is being treated for tuberculosis, but he has not been compliant with his medications. He complains of increasing weakness and fatigue. He appears to have lost weight, and his blood pressure is 80/50 mmHg. There is increased pigmentation over the elbows. Cardiac exam is normal. The next step in evaluation should be: A. Early morning serum Cortisol and cosyntropin stimulation B. CBC with iron and iron-binding capacity C. Erythrocyte sedimentation rate D. Blood cultures E. Level of serum K+ 42. A 25-year-old woman is admitted for hypertensive crisis. In the hospital, blood pressure is labile and responds poorly to antihypertensive therapy. The patient complains of palpitations and apprehension. Her past medical history shows that she developed hypotension during an operation for appendicitis. Hct: 49% (N=37-48). WBC: 11 x 103 mm (N=4.3-10.8). Plasma glucose: 160 mg/dL (N=75-115) Plasma calcium: 11 mg/dL (N=9-10.5). The most likely diagnosis is: A. Pheochromocytoma B. Renal artery stenosis 184 A. B. C. D. E. C. Essential hypertension D. Insulin-dependent diabetes mellitus E. Cushing syndrome 43. A 36-year-old woman has a 3-year history of oligomenorrhea that has progressed to amenorrhea during the past year. She has observed loss of breast fullness, reduced hip measurements, acne, increased body hair, and deepening of her voice. Physical examination reveals frontal balding, clitoral hypertrophy, and a male escutcheon. Urinary free Cortisol and dehydroepiandrosterone sulfate (DHEAS) are normal. Her plasma testos terone level is 6 ng/mL (normal is 0.2 to 0.8). The most likely diagnosis of this patients disorder is A. Arrhenoblastoma B. Cushing syndrome C. Polycystic ovary syndrome D. Granulosa-theca cell tumor E. Thyroid carcinoma 44. At patient Р., who treat from septic condition, whom expressed weakness, adynamia, has suddenly appeared, vomiting, a diarrhoeia. Sopor. Deficient pulse, 110 /minutes, ВР 60/40 mm hg. On an electrocardiogram: a tachycardia, decrease in a voltage of all waves. Laboratory data: hyponatremia, hypochloremia, hyperpotassemia, hypoglycemia. Specify the reason of development of such condition: Hypothalamic crisis; Acute insufficiency of adrenal glands; Hypoglycemic coma; Panhypopituitarism Acute heart attack of myocardium 45. Patient D, 42 years, after a physical overstrain has lost consciousness. BP has decreased to 40/20 mm hg In the 185 anamnesis long (5 years) reception of glucocorticoids, in connection with presence of a bronchial asthma. Last 4 days glucocorticoids does not accept. About th: it is braked, a skin of usual colour, normal humidity, tones of heart deaf persons, pulse 100 in minute, weak filling, rhythmical. Glucose level in blood - 3,0 mmol/l, sodium - 117 mmol/l, potassium - 6,0 mmol/l. Your preliminary diagnosis. A. Cardiogenic shock; B. Adrenal crisis; C. Acute adrenal insufficiency; D. Hypovolemic shock; E. Hypoglycemic coma 46. At sick К, 29 years, with the compensated diabetes of 1 type have appeared frequent hypoglycemia, a nausea, intestines frustration, skin hyperpigmentation (bronze colour), ВР - 70/50 mm hg., Hb 100 g/l. Than decrease ВР can be caused? A. Chronic insufficiency of adrenal glands B. Diabetic enteropathy C. Diabetic gastropathy D. Overdose of antidiabetic preparations E. Development diabetes insipidus 47. Patient U, 18 years, is taken to hospital by an acute care unconsciousness. According to native it is known, that last two weeks it was observed. Rise in temperature of a body to 39º. With, that was regarded, as flu signs. Four hours there was a sharp pain in a stomach, decrease ВР. At survey: epileptic convulsion, meningeal syndrome. From additional researches: level increase potassium in blood to 8 mmol/l, level cortisol- 18 mkg / 100 ml of plasma. On an electrocardiogram – high waves T. Computer tomography - sign hemorrhage infarction of adrenal 186 glands. What diagnosis is most probable? A. Acute insufficiency cortex of adrenal glands B. Hyperosmolar coma C. Hyperlactacidemic coma D. Hyperthyroid coma E. Hypoglycemic coma 48. The patient 43 rivers it is delivered in a grave condition. According to the husband, is ill with Addison's disease. Constantly accepted 5 mg prednisolone. Last week a preparation did not accept, as there was a pain a stomach, appetite has worsened, yesterday did not eat in connection with a nausea and vomiting. The patient in soporose state. A skin and mucous hyperpigmentation. Turgor skin and muscles the lowered. Tones of heart are muffled, accelerated, ВР-60/40 mm hg, НR-96/minute Sodium in blood - 130 mmol/l, potassium - 5.5 mmol/l. What deficiency of a hormone plays the basic role in complication development? A. Aldosterone; B. Adrenocorticotropic hormone, ACTH C. Adrenaline; D. Noradrenaline; E. Cortisol. 49. At sick with suspicion on pheochromocytoma during the period between attacks the BP - within norm; it is marked tendency to a tachycardia. In urine of a pathology is not present. It decided to spend provocative test with histamine. What preparation should be had for rendering of the emergency help at positive result of test? A. Mesaton B. Pipolphen C. Nifedipine D. Phentolamine 187 E. Prednisolone 50. At 38 summer women paroxysmal raises the BP of 240/120 mm Hg, thus the nausea, vomiting, a tachycardia, raised perspiration, comes to light hyperglycemia. After an attack the plentiful discharge of urine. При sonography kidneys is revealed an additional education adjoining to the top pole of the right kidney, probably, concerning an adrenal gland. What laboratory test will help to specify the diagnosis? A. Level definition rennin in blood B. Definition of insulin and C-peptid in blood C. Definition of speed glomerular filtration on endogenous creatinin D. Definition thyroxin and ACTH in blood E. Definition excretion catecholamine and vanillic mandelic acids with urine 51. At the patient 35th years muscular weakness, spasms of the feet, moderated polyuria. Medicines did not accept. Weight of a body of 75 kg at growth of 170 sm, the BP of 160/100 mm hg. Blood glucose on an empty stomach 3.5 mmol/l, potassium blood 2.4 - 2.5 mekv/l; sodium of 140 mekv/l; creatinine 0.08 mmol/l. Fluctuations of relative density of urine 1005-1015; glycosuria and proteinuria is not present, a deposit in norm. What probable cause of illness A. Diabetes insipidus B. Hyperaldosteronism C. Insulinoma D. Insufficiency of adrenal glands E. Hyperparathyroidism 52. At sick 45 years the central type of adiposity of 2 degrees, purple-red striae on a stomach, hips, increase the BP to 200/120 mm hg, glycemia on an empty stomach - 9,3 mmol/l, infringement menstrual cycle is marked. After big 188 dexamethasone test excretion 17- oxycorticosteroids, 17-CS has not changed. The most probable diagnosis: A. Corticosteroma. B. Cushing’s disease. C. A metabolic syndrome. D. Syndrome Shtejn-Levental’s. E. Carcinoid syndrome. UNIT 4 A. B. C. D. E. A. B. C. D. E. 1. A 49-yr-old man is admitted for investigation and glycosuria. His wife comments that his appearance has changed over the last few years and "everything seems to have got bigger". He also cojnplains of tingling in his left hand and excessive sweating. Prescribe investigations, please. Dexamethasone suppression test Serum cortisol T3, T4 and TSH levels Look at old radiographs* Water deprivation test 2. A 31-year-old male complains of polydipsia she drinks nearly 12 L of water during the day, polyuria, weakness. Several weeks ago he has cerebral trauma. Postprandial glucemia level is 6,3 mmol/l. In analysis glucose is absent, there is low urine osmolality. What is your diagnosis? psychogenic polydipsia hypopituitarism diabetes insipidus* syndrome of inappropriate ADH secretion diabetes mellitus 189 3. A 31-year-old man is married during 8 years but hasn’t children. Wife is healthy. He is 188 cm tall, his weight is 76 kg. Ginecomastia. Secondary sexual signs are developed purely. Penis is normal size, testes are small, firm. Azospermia is present. Which of the following karyotypes is most likely to be present? A. 45 XO B. 46 XY C. 46 XX D. 47 XXY* E. 47 X0 4. A young (29-yr-old) women comes with secondary amenorrhea and galactorrhoea. MRI shows a tumor of < 10mm diameter. What kind of treatment can you recommend? A. Hormonal therapy for withdrawal bleeding B. Radiotherapy C. Chemotherapy D. Bromocriptine* E. Surgery 5. A 31 year old women presents a history of amenorrhea and impaired vision of five month's duration. Physical examination is normal except for pale optic discs and diminished visual acuity. The most likely diagnosis is: A. Pituitary adenoma* B. Craniopharyngioma C. Hypothalamic glioma D. Benign intracranial hypertension E. Encephalitis 6. All are components of Laurence-Moon-Biedl syndrome EXCEPT: A. Mental retardation 190 B. C. D. E. Digital anomalies Hypogonadism Asthenic body built* Obesity 7. A 21 year old woman is seen for primary amenorrhea. She is short, has poorly developed breasts and a webbed neck. Her chromosome pattern is most likely to be: A. XY B. XO* C. XXY D. XYY E. XXYY 8. A 62 - year-old female with polydypsia. Water deprivation test was performed and the values obtained are as follows: (Baseline : Na+-138 mEq, K+-3.6m Eq, Osm. plasma-274/L Osm, urine-151 mosm. After vasopressin: Na+-143) mEq, K+-3.9 mEq, Plasma osm 271 mosm/L, Urine Osm-489 mosm/L). Diagnosis is: A. Cranial diabetes insipidus* B. Psychogenic polydypsia C. Nephrogenic diabetes insipidus D. SIADH E. Diabetes mellitus 9. A 22 year old woman presents with complaints of primary amenorrhoea. Her height is 152 cms, weight is 50 kg. She has well developed breasts. She has no pubic or axillary hair and no hirsuitism. Which of the following is the most probable diagnosis? A. Turner's syndrome B. Stein-Leventhal syndrome C. Premature ovarian failure 191 D. Complete androgen insensitivity syndrome* E. None above them 10. A 37 year old man presents with vomitings and confusion. On examination Na+ 119 m mol/L, K+ 4.2 m mol/L, Uric acid 2 mg/dl. Patient is no edematous. The diagnosis is: A. Cerebral toxoplasmosis with SIADH* B. Hepatic failure C. Severe dehydration D. Congestive heart failure E. None above them 11. A 32 years old woman complains of evaluation of chronic thirst, polydipsia, and polyuria. The symptoms have been present for many years and seem to increase during her pregnancies. Several years before, she consulted another physician who “tested my sugar”, performed a dehydration test, and told her not to worry because she simply “drank to much”. She is otherwise healthy and takes no drugs. She was adopted at an early age and knows nothing of her biological family. Objective status and result of laboratory are within normal limits. However, her 24 – hour urine volume and osmolality are 6,5 liters and 1002 respectively. A fluid deprivation test increases her urine osmolality and plasma sodium to maxima of 465 mmol/kg and 141 meg/L, respectively, after four hours. It also increases her thirst. At this point, the best approach would be to: A. Assess the response of urine osmolality to the injection of desmopressin B. Order MRI of the brain before and after gadolinium infusion C. Measure plasma vasopressin before and after an infusion of hypertonic saline 192 D. Start treatment with desmopressin and reevaluate her symptoms in two weeks E. Reassure her that she simply drinks excessively and should restrict her fluid intake 12. An 18 year old young man is tall and has narrow shoulders, a large pelvis, an adult woman pattern of hair distribution, and oxyphonia. Mental retardation is also present. Based on these symptoms, the provisional diagnosis of Klinefelter's syndrome was made by a doctor. What genetic method can confirm the diagnosis? A. Cytogenetic B. Pedigree analysis C. Population-statistical D. Biochemical E. Study of twins 13. During the analysis of the buccal mucosa epithelium of a male patient two Barr bodies in each nucleus of most cells were found and in neutrophil nuclei two "drumsticks" in each were found. What syndrom is it typical of? A. Patau syndrome B. Turner's syndrome C. Klinefelter's syndrome D. Down's syndrome E. Edward's syndrome 14. An 18 year old girl complained to a doctor of the absence of menstruation. The patient had such features: height -140 cm, a short neck with typical folds ("neck of sphinx"), wide shoulders, a narrow pelvis, absence of secondary sexual characters, underdeveloped ovaries. What was the provisional diagnosis of the girl? A. Turner's syndrome 193 B. C. D. E. A. B. C. D. E. Patau syndrome Morris's syndrome Klinefelter's syndrome Down's syndrome 15. A 24 years old man is refferred because incidentally discovered pituitary adenoma. He relates that he was involved in an automobile accident three weeks earlier. Magnetic resonance imaging performed to evaluate the consequences of head trauma revealed a 7 mm area in the right side of the pituitary that did not emit as bright a signal following gadolinium as did the rest of the pituitary. On questioning, he said he had been felling generally well. Specifically, he had experienced headaches for a week after the accident but not before or since. He had not noted a change in libido, potency, or appearance. Objective status: height -160 cm; weight -75,5 kg, blood pressure was 118/82 mmHg; pulse - 72 beats/min and regular. He didn’t appear acromegalic or to have Cushing’s syndrome. He had no lid lag or lid retraction. His thyroid gland was not palpably enlarged. Serum rolactin concentration was 12 ng/mL (normal<15 ng/mL). Which one of the following tests should you order next? Thyroid Stimulating Hormone (TSH) Visual fields Luteinizing Hormone (LH) Follicle Stimulating Hormone (FSH) Insulin-like growth factor-1 (IGF – 1) 16. Patient F., 57 years old, complains of general weakness, headache, weight gain, menstrual irregularity, hirsutism, frequent urination and thirst. Objective status: height - 157 cm, weight - 92 kg. Localization of fat tissue is disproportional, being found mostly on the face, neck and trunk with relatively thin extremities. Face is ruddy, “moon face”. There are dark 194 A. B. C. D. E. A. B. C. D. E. red striae under her arms, on the inner surfaces of the hips and on the belly. Skin is normal, moist, pulse - 70 beats/min, blood pressure – 180/100 mmHg. Over her upper lip and chin large amount of hair growth (patient shaves). Laboratory tests: 17-OHCS is elevated, oral glucose tolerance test (OGTT) – 6.2-10.8-9.4 mmol/l, glucose in urine – 0.5%. What is the most likely diagnosis? Diabetes mellitus Arterial hypertension Itsenko-Cushing syndrome, steroid diabetes Obesity Steroid diabetes 17. A 45 year old man was referred by his dentist for prognathism and malocclusion. Shoe size had increased from 6 to 8 in the last six years and the patient complained of excessive sweating, headache, but denied change in sexual activity. Objective status: the patient had a prominent jaw, spacing of the teeth, thick skin and the thyroid was enlarged 2 times normal size, and felt firm and finely nodular. The rest of the examination was normal except for large, spade-like hands and feet. Laboratory evaluation revealed: fasting growth hormone increase 1,5 times normal, fasting glucose was 6,5 mmol/L. The most likely diagnosis is: Gigantism Obesity Acromegaly Physiologic Growth Hormone variation Hypothyrosis 18. Patient S., 33 years old, complains of a tiresome thirst (drinks 8-12 l/day), mouth dryness, frequent urination, weight loss and headache. In the anamnesis - tree months ago patient 195 A. B. C. D. E. A. B. C. D. E. suffered a craniocerebral injury. Objective status: height – 174 cm, weight – 53 kg, skin is dry, turgor decreased. Blood pressure – 110/60 mmHg, pulse –70 beats/min. Result of laboratory tests: specific gravity 1002-1004, oral glucose tolerance test – 4.7-7.1-5.5 mmol/l. X-ray of the cranium – normal, eye fields – normal. What diagnosis would you suggest? Diabetes mellitus Diabetes insipidus Panhypopituitarism Sheehan’s syndrome Hypothyrosis 19. Patient Y., 34 years old, complains of absence of libido, dizziness, feeling of cold, constipation, frequent urination, paresthesias, worsening of eyesight and memory, increased disability, thirst with 2-4 l/day of fluids being consumed. The disease commenced approximately 2 years previously, with an ophthalmological consultation because of a narrowing of eye fields and a consequent diagnosis of pituitary tumor. Patient has under gone a radiation therapy to ablate the tumor. Objective status: height – 168 cm, weight – 57 kg, skin is dry and cool. Blood pressure – 85/50, pulse – 56 beats/min. 17-OHCS is decreased, T3, T4, TSH, LH levels are decreased. What is the most likely diagnosis? Hypothyrosis Diabetes insipidus Panhypopituitarism Addison’s disorder Pituitary tumor 20. A patient R., 54 year old complains of headache, increased sweating, periodical palpitation, elevation of blood pressure, severe weight gain (30 kg in 3 years), sexual failure. 196 A. B. C. D. E. The patient considers himself to be ill for 3 years. He connects his disease with having craniocerebral trauma. Objective status: height is 187 cm, weight is 128 kg. Body mass index 42 kg/m2. There are numerous pink thin stria on the hips and abdomen, the skin is highly wet. The left heart border is displaced to 2 cm laterally. Cardiac sounds are dull, there is an accent of the second heart sound above the aorta, blood pressure is 180/110 mmHg on the left arm and 160/100mmHg on the right arm. Lungs are without pathology. Abdomen is enlarged because of adipose tissue, painless. Liver is near the border rib's are. Thyroid gland is not enlarged. What is the clinical diagnosis? Hypothalamic obesity, class III Cushing disease, obesity, class II Alimentary constitutive obesity, class II Obesity class III Arterial hypertension 21. A 53 year old patient who is suffering from diabetes melitus, after acute cerebral blood circulation disorders feels thirst, dryness in the oral cavity, polyuria to 10-12 litres/ day. Blood glucose during a day is 4,6-6,9-6,0 mmol/litre, urine glucose is absent, relative density of urine is 1002 c.u. Such changes can testify: a. decompensation of diabetes b. psychical disorders c. autonomous nueropathy of urinary bladder d. joining of unsaccharine diabetes e. joining of pyelonephritis 22.A patient of 24 years old who had a stress has thirst, dryness in the oral cavity, polyuria to 10-12 litres/day. Glucose of blood during a day is 4,6-6,9-6,0 mmol/litre, urine glucose is absent, relative density of urine – 1002 c.u. thirst, dryness in 197 the oral cavity appeared, polyuria to 10-12 litres/day. relative density of urine is 1002-1003 c.u. This symptomatics is conditioned: a. by insufficiency of aldosterone b. by insufficiency of vasopressin c. by insufficiency of prolactin d. by insufficiency of thyrotropin e. by insufficiency of adrenalin 23.Hypersecretion of somatotropic hormone in adult age, besides acromegaly development, often leads to: a. hypocorticism b. hyperaldosteronism c. saccharine diabetes d. pheochromocytoma e. insulinoma 24.Galactorrhea appearence requires differential diagnostics among: a. prolactinoma and polycystosis of ovaries b. prolactinoma and menopause c. prolactinoma and aldosteroma d. prolactinoma and endometriosis e. prolactinoma and hypothyreosis 25.For a 38 year old patient with the syndrome of constant galactorrhea-amenorrhea the preasence of adenomas of hypophysis was established. Conturs of the Turkish saddle are not changed, intracranial hypertension is absent. Preparation for treatment of this patient is: a. bromcriptine b. bromcamphora c. bromelain d. bromantan 198 e. bromhexine 26.The next dynamics of STH in reply to the oral loading of glucose 1,75 grammes/kg will be characteristic for acromegaly: a. decline of STH b. absence of changes c. increase of STH d. the level of STH is not estimated e. all mentioned above 27.A diagnostic criterion of hyperlipidemia under the metabolic syndrome X is: a. level of blood triglycerides is below 1,7 mmol/l b. level of blood triglycerides is below 2,2 mmol/l c. level of blood triglycerides is higher 1,7 mmol/l d. level of blood triglycerides is higher 2,2 mmol/l e. level of blood triglycerides is higher 3,5 mmol/l 28.Adiposity is determined as abdominal when the circumference of waist is: a. for men more than 102 cm, women more than 88 cm b. for men more than 90 cm, women more than 70 cm c. for men more than 88 cm, women more than 102 cm d. for men more than 80 cm, women more than 60 cm e. for men more than 116 cm, women more than 102 cm 29.The piculiarity of metabolic syndrome for women is: a. more expressed adiposity b. absence of hypertension c. henoidal adiposity d. inclination to suicide e. The symptomatics is more closely connected with menopause than with adiposity 30.The dense knot in the thyroid gland of the 16 year old boy was found. The size of the knot is 10 mm. Local 199 lymphatic nodes are not increased. The function of thyroid gland is not disturbed. Choose the tactics of treatment. a. prescribing of mercazolyl b. prescribing of propiltiouracyl c. prescribing of iodide potassium d. prescribing of thyroxyne e. surgical treatment 31.The development of papillary cancer of the thyroid gland is mainly connected with: a. radiative irradiation b. hypothyrosis c. thyrotoxicosis d. iodine deficit e. iodine surplus 32.The development of medullary cancer of the thyroid gland is mainly connected with : a.radiative irradiation b. hypothyrosis c. thyrotoxicosis d. iodine deficit e. iodine surplus 33.The advantage of a thin-neeled aspirative biopsy of the thyroid gland under the USD control can be everything except: a. possibilities of gland’s function estimation b. possibilities of making the biopsy when the size of knots is 10 mm and less under the condition of their palpation c. economical justification d. the absence of risk for the patient e. possibility of mastering the method 34.The following factors: a. presence of local lymphadenopathy 200 b. clear contours of the knot with hydrophyl rim c. head and neck irradiation in the anamnesis d. rapid rates of knot growth e. age up to 20 do not belong to those which specify the high probability of thyroid gland cancer. 35.The probability of thyroid gland cancer is high if it is discovered that the patient has: a. multinodal goitre b. fluctuant and soft nodes c. nodal form of Khashimoto thyroiditis d. a solitary node with clear contours e. a solitary node with washed out contours 36.The malignant formation of thyroid gland during histological research of punctation is shown by: a. presence of plenty of watery colloid b. multinuclear cells c. preasence of macrofages with foamy cytoplasm d. cyst or cystic degeneration of the node e. presence of lymphocytes 37.For the histological investigation of medullary cancer the following is not characteristic: a. presence to the amyloid b. the positive coloration for calcetonine c. fusiform cells with eccentric nuclei d. intranuclear includings e. presence of colloid in plenty amount 38.To the methods of thyroid gland visualization do not belong to: a. ultra-sound research b. proton-emission tomography c. computer tomography 201 d. magneto-resonant tomography e. punctional biopsy 39.To the malignant diseases of thyroid gland belong to a. degenerated colloid node b. diffuse untoxic goitre c. multinodal goitre d. adenocarcinoma e. colloid cyst 40. Disorders of carbohydrate metabolism in patients with acromegaly can be conditioned: A. Contrainsular influence of growth hormone B. Decreasing level of half-life of insulin C. Disorders of insular apparatus function D. Activation of sympathoadrenal system E. Increasing of estrogens secretion The right answer : A 41. What is the typical picture of data of adrenals CT in patients with Cushing’s disease: A. Both adrenals are enlarged, regular shape B. One adrenal gland is enlarged, unregular shape, others is diminished C. One adrenal gland is enlarged,unregular shape, others has normal size D. Both adrenals are diminished, regular shape E. Both adrenals aren’t changed The right answer : A 42. At Which is mental development in patients with hypophysial nanism, as a rule? A. Normal B. Slowed C. Accelerated D. It is slowed in puberty 202 E. It is slowed in babyhood The right answer : A 43. Patient, 37years, complains about thirst, polyuria (to 10l a day), weakness. He relates a disease with carried cranialcerebral trauma. Pathology of internal organs was not detected. Laboratory data: blood count - without pathology; urine analysis: relative density - 1001, protein - absent, sugar absent, leukocytes - 2-3 in area vision. Previous diagnosis? A. Diabetes insipidus B. Acute renal failure, stage of polyuria C. Diabetes mellitus D. Primary hyperaldosteronism E. Chronic glomerulonephritis The right answer: A. 44. Man, 48 years, pitman, appealed to the doctor with progressing headache, changes of appearance. In anamnesis chronic tonsillitis. He practises upon an alcohol, tobacco. Objectively: height - 178cm, weight - 92kg. There are enlarged superciliary arcs, nose, ears, tongue. Prognatism. There are enlarged hands and feet. A skin is thickened, greasy. Liver +2cm. Stagnant optic papilla. Fasting glycemia - 7,2 mmol/l. What the most credible reason of development of this state? A. Tumour of brain B. Chronic infection C. Abuse by an alcohol D. Terms of labour E. Chronic overeating The right answer: A. 45. Woman, 32 years, complains about a weakness, apathy, sensitiveness to the cold, amenorrhea, which are developed for a year after the births complicated by the strong bleeding. Objectively: height 165cm. Weight 54kg. Pulse - 203 56/min. BP -90/50mmHg. A skin is pale, dry. Fasting glycemia - 3,3 mmol/l. What complication of sharp bleeding is the most probably? A. Pituitary insufficiency B. Primary insufficiency of the adrenals C. Primary hypogonadism D. Hypothalamic syndrome E. Adenoma of pituitary The right answer: A. 46. Man, 19 years, hospitalized with complaints about polydipsia, polyuria - to 9 litres a day, weakness, anorexia. The first symptoms of disease appeared after the carried flu 2 weeks later. Glycemia - 4,2 mmol/l. What additional examination must be conducted for definition of clinical diagnosis? A. Analysis of urine by Zimnitsky B. Common analysis of urine C. Analysis of urine on acetone D. Analysis of urine by Nechiporenko E. Blood count The right answer: A. 47. Patient (woman), 24 years, put of the weight on 7kg during last year, she has frequent headaches, general weakness. Objectively: accumulation of fatty tissue is more expressed on a neck, thorax, abdomen. There are red strias on the lateral surfaces of abdomen. Menses absent, there is hairiness to masculine type. BP - 160/100mmHg, HR - 92/min. Additional data: X-ray-examination of cranium - expressed osteoporosis, extended sella turcica. Clinical diagnosis? A. Cushing's disease B. Cushing's syndrome C. Pubertal dyspituitarism D. Primary hypoparathyrosis 204 E. Stein-Levental's syndrome The right answer: A. 48. Patient, 38 years, complains about permanent headache, thirst, promoted fatigue, increase BP and gain in weight (46 kg during 3 years). The disease is related with flu. Objectively: height - 176cm, weight -143kg, distributing of fatty tissue: mainly on trunk, thighs, abdomen, red strias. BP 180/100mmHg. Your previous diagnosis? A. Cushing's disease B. Hypothyroidism C. Hypothalamic syndrome, neuroendocrine form D. Cerebral obesity, III degree E. Alimentary-constitutional obesity, ІІІ degree The right answer: A. 49. Patient, 25 years, appealed to endocrinologist with complaints about problems in sexual life (violation of potency, infertility). Objectively: eunuchoid type of constitution, height 191cm, weight 78 kg, sense of glandular tissue is present palpatory. Sexual organs are formed by masculine type. Testicles are packed, diminished in a size. Sexual chromatin 32%. Cariotype 47ХХY/46ХY. Specify a diagnosis? A. Kleinfelter's syndrome B. Clean dysgenesia of gonads C. Terner's syndrome D. Primary hypogonadism E. Mayer-Rokitansky-Kustner's syndrome The right answer: A. 50. What from clinical signs is not typical for the Conn’s syndrome? A. Hyperkaliemia B. Muscular weakness C. Alkaline reaction of urine 205 D. Hypokaliemia E. Hypernatriemia The right answer : A 51. When during days is it better to prescribe glucocorticoid therapy in patients with Addison’s disease? A. In the morning after sleep B. From 9 to 12 o'clock a.m. C. Before every use of meal D. After dinner E. Before sleep The right answer : A 52. What influence has catecholamines on metabolism of sodium in organism? A. Increasing of reabsorption B. Increasing of excretion C. Decreasing of rabsorption D. Without influence on metabolism of sodium E. B and C are true The right answer : A 53. A 27 years old man, has height 121 cm. Complaints are absent. Apparently: body parts are proportional, pathology of the internal organs is absent. What can be the reason of he is being small? А. *Pituitary pathology, В. Hypothalamus pathology, С. Thyroid gland pathology, Д. Genetic disorders, Е. Normal variant. 54. The patient of 34 years shows complaints on polydipsia (drinks more than 5 l of a liquid a day), pollakiuria, a polyuria, fatigue, a weight loss. Has transferred neuroinfection. Dryness of integuments, a mucous oral cavity. 206 Urine: the density - 1004, is not present some fiber, sugar, leukocytes - 3-5 in the field зр., эр. - 0-1 in the field зр. Creatinin - 0,13 mmol/l. Specify the most probable pathogenetic mechanism of the given A. *Decrease distal reabsorption waters at a lack antidiuretic hormone. B. Decrease proximal reabsorption waters at a lack antidiuretic hormone. C. Osmotic diuresis. D. Increase glomerular filtration because of polydipsia E. Decrease distal reabsorption waters owing to an atrophy canal epithelium. 207 Навчальне видання Методичні вказівки до практичних занять із дисципліни «Ендокринологія» для іноземних студентів спеціальності 222 «Медицина» денної форми навчання (Англійською мовою) Відповідальний за випуск Н. В. Деміхова Редактор О. О. Кучмій Комп’ютерне верстання Н. В. Деміхової Формат 60х84/16. Ум. друк. арк. 12,09. Обл.-вид. арк. 10,67. Видавець і виготовлювач Сумський державний університет, вул. Римського-Корсакова, 2, м. Суми, 40007 Свідоцтво суб’єкта видавничої справи ДК № 3062 від 17.12.2007. 208